You are on page 1of 301

MRCOG part 2 September 2015 Recalls

• INCIDENCES/FIGURES:
1. UAE was done for a 30 yrs old woman. % of those who will require re-intervention?
[25%]
2. Abroutpio recurrence %
[?4.4% after one time]
3. OASI incidence.
[?2.9%]
4. Risk of miscarriage after 3 consecutive miscarriages
[?40%]
5. What range is coded as very rare
[<1/10,000]
6. In what percent of PHA a cause can be found?
[options: 40-50%, 50-60%,……].
7. Risk of recurrence of shoulder dystocia above population in a woman with PH of dystocia.
[In folds: 10]
8. % of Ureteric injuries that can be identified during laparoscopy (intra-operatively):
[1:3]
9. Incidence of post dural tab/puncture; or duration of puncture headache?
[?0.5-2.5%, 7-10d]
10. Incidence of moderate to severe OHHS
[3-8%]
11. Risk of ovarian cancer in BRCA-1 carrier
[?40%]
12. Pt with PH of severe pre-eclampsia <28 wks, what is the chance of recurrence?
[1:2 (or 55%)]
13. Incidence of twins after successful IVF cycle?

1|Page
[25% (1:4)]
14. what percentage of couples will succeed to conceive within the 1
st
yr?
[>80%]
15. Laproscopy ,serious complications
[2:1000]
16. A woman with atypical endometrial hyperplasia is planned for TAH. What is the risk of finding
a concomitant cancer?
[?40- 50%,…]
17. What is the incidence of OAB among the elderly ladies??
[?15%.]

• UROGYNE:
18. What is the most common presentation of vault prolapse ?
[options: bulge in vagina, sexual dysfunction, stress incontinence, constipation, voiding
dysfunction]
19. Cystoscopy [I don’t know how did they get the photos]: Your management?

[?Refere to urologist (? tumor)]


20. Cystoscopy, diagnosis? here Woman has 3 yrs history of urgency frequency, 2 or 3 times
culture positive, now no growth. Options were chronic cystitis, IC, schistosomiasis,……
[petechial haemarrhages =IC ?]

21. A woman presented for cystocele correction. She also complained of stress incontinence. O/E:
cystocele grade( ) , no demonstratable sress incontinence. Next step?
[options: urodynamics, video urodynamics, urodynamics with measuring of residual
volume,ant.repair…..]

2|Page
22. 52 yrs women with recurrent UTI. UG shows microscopic haematuria and no growth. Next
step?
[?Cystoscopy]
23. The most suitable drug :?Elderly lady with OAB symptoms, ?mainly frequency, ?urodynamics
confirmed DI:
[? darifenacin]

• MMR/MBBRACE:
24. Commonest overall cause of MMR in 2008-2008 report?
[cardiac]
25. Long scenario, at the end asked for the commonest cause of neurological death in last MMR
report.
[? SAH]
26. How eclampsia kills according to new repor (? the most common cause of death in PET)?
[options : ICH, ARDS, uremia,…]
27.
• GYNE:
28. 9 years old girl brought C/O fever, nausea vomiting + pain in left side of pelvis.
[options: laproscopy, analgesia+ observation, treat PID , inform safeguarding services, refere to
surgeon,…]
29. A woman with IUD in-situ came with 5 wks amenorrhea & +ve PT. U/S: fundally locted sac no
fetal echo seen. 1
st
hcg around 600, repeated 48hrs around 1300. Most likely diagnosis?
[options: viable intrauterine, ectopic, PUL,…]
30. A woman with mild vaginal bleeding and mild lt lower abdominal pain. U/S shows a sac –
intrauterine- measuring around 20×20mm and a left ovarian cyst around 2cm. HCG around 900
iu. Most likely diagnosis?

3|Page
[same options as above]
31. Anti-D dose after kleihauer test -done after blunt trauma- showing ≈ 4ml bleeding at 20wks GA
[?500 u]
32. EMQ: refere letter from a GP with results of HIV & 1
st
TM screening: (thanks to d.mutaz)
Birth date .../...1975
Uss : CRL= 54 mm head _ 11-12 w abd =12-13 wks
Hcg =... normal
PAPPA .24 MOM
RESULT : 1/ more than 350 for down
1/ 12 for edward
1/ 12 for pataue
There is a hint of possibility of edward and patau Syndrome..
Q1. this pt has also a high titre of HIV, what is your management? [?refer to FM unit]
Q2. You notice that the birth date of pt . Is different from the date written in the referral letter (
1972) how you manage this case? [? repeat all the investigations]
33. ;;;;;
34. A lady was referred with results of 1
st
TM screen. All withen normal except ?↑(?↓) PAPP-a.
Management?
[options:???? Serial growth scans, uterine Doppler at 20wks,…]
35. A lady with 1
st
TM screening result for down’s: ?1/15. She has a very high HIV viral load.
Management?
[options: AC, CVS, placental biopsy, refere to FM unit,..]

4|Page
36. 74 y post ? coital [or PMB? ] bleeding. ET 3mm + some fluid in cavity.
[Options: D&C, hysteroscopy, hysteroscopic guided biopsy, pipelle, do nothing]
37. ?5th post op(TAH) day hematuria
[?options]
38. 10th post op day hematuria
[?options]
39. What abortion clause: young girl,…..(social reasons)
[C]
40. What abortion clause: anencephalic fetus
[E]
41. Suture material: for mass closure of periton & rectus sheeth after midline incision in obese lady
[ ?PDS 2]
42. Suture material: for repair of perineal skin.
[?2/0 vicryl]
43. Suture material: for re-anastomosisof faloppian tubes(reversal of sterilisation)
[?vicryl 4-0, 4-0 prolene with reverse cutting needle, ,..???]
44. Suture material: to close lap.port site
[j shaped needle]
45. Instrument used for: clamping the uterine artery/pedicle
[pictures rather than names: ?semicurved zepilin]
46. Instrument used for: ?deviding/dissecting the ureter during Wertheim surgery.
[?lahey’s ( loong forceps + small curve at the end)].
47. Instrument used for: handling/clamping the bowel during radical surgery
[? babcok]
48. Pst-op complications: Pt. after abdominal surgery presented few days later feeling unwell,
febrile, left hypochonrial pain radiated to the left shoulder. O/E stony dull percussion other
wise no other finding
[? Subphrenic abscess, pneumonia, bowel obstruction]

5|Page
49. Post-op complications: Pt. After 4 hrs of retrieval of oocytes develope severe tachycardia and
hypotention
[?vessel injury]
50. Post-op complications: Pt. After 3 days of oocyte retrieval develooe abd pain e guarding and
rebound. O/E distended abdomen, ?sluggish bowel sounds
[? Bowel inj, paralytic ileus,…]
51. Post-op complications: 24 hrs post hysterectomy .. feeling unwell .tachycardia + mild fever ..
cathetr shows 50ml of dark red blood
[? Review fluid balance, return to theater,….]
52. Post-op complications: 5 days post hysterectomy she came with fever and haematuria
[?UTI]
53. Post-op complications: diabetic patient on PCA developed SOB. O/E pinpoint pupils. Bp 80/60.
Action?
[options: naloxon (opiate overdose), adrinalin, atropine, glucose]
54. Post-op complications: 7 days after hysterectomy she came with tender abdomen, left loin
pain and haematuria.
[?ureteric injury]
55. What is the main complication to notify: Laparoscopy for endometriosis
[? Options: ureter/bowel inj, bleeding, BT,……]
56. What is the main complication to notify: Hysterectomy for multiple fibroids
[? Options : as above] ?BT

57. Management options of [or prophylaxis from] vault prolapse with different 3 scenarios: An
elderly with comorbidities [? colpocleises], 2
nd
one with short vagina [ ?Sacrocolpopexy], & the
3
rd

6|Page
in whom vault was reaching the introitus during VH [? SSF]
58. ;;;;
59. ;;;;;
60. What is the mech of action of tranixamic acid?
[?tissue plasminogen activator- inhibitor (ie anti-fibrinolytic)]
61. What is the main mech of action of Cocs?
[inhibition of ovulation]
62. What pair of hormones have very similar composition?
[? TSH/HCG]
63. What sign do you expect to find: obese girl BMI is 40 , mother DM type 1 .. has irregular cycle
and hirsutism ..
[?Acanthosis N]

• ONCOLOGY:
64. 55 years old undergone ?LLETZ cGIN with incomplete margin. Action?
[? hysterectomy]
65. After teated CIN, repeat HPV after 6 months was negative. Time of next smear?
[? 3 yrs]
66. Smear was borderline. HPV was +ve. After 6m, smear again borderline but HPV –ve
[? Routine recall]
67. Colposcopy showed ?leucoplakia, mosaicism, punctuations. Next step?
[options: biopsy , cone, LLETZ,…..]
68. 45 years old woman underwent MRI for (whatever reason). Incidentaly, a 6 cm simple ovarian
cyst was found. Management?
[???options]
69. while investigating an old post menopausal 67 years lady for bilateral complex ovarian cysts,
CA-125 was found to be 75. Management?
[????options]

7|Page
70. A patient with stage 2b Ca Cx who refused treatment. What is the most relevant complication
to notify?
[? Renal failure, metastasis, bleeding,…..]
71. A woman with a huge u.fibroid that caused heavy periods, anemia, preesure
effect(?hydronephrosis). What is the most relevant complication to notify if she refused
surgery?
[same options as above]

• INFIRTILITY/ENDOCRINE/FAMILY PLANNING
72. Infertile couple,Women with normal investigations, husband with azospermia. Likely cause?
[klienfilter?]
73. Two embryo transfer in 1st cycle at what age/condition?
[?≥40]

74. EMQ asking about management of different presentations of PCO: 1


st
is a young lady not in a
relationship, main complain is irregular periods. She has normal BMI but her BP was 140/90.
[options: high dose progesteron, Cocs, metformin, Cocs+metformin, clomit citrate,….]
75. As above but lady is p3, with↑ BMI.
76. Cause of 2
nd
ry A
0
: 8 months after delivery with ↓FSH/LH & ↑↑ PRL(2000)
[options: pregnancy, sheehan’s, prolactinoma,…]
77. What is the commonest cause of POF
[?autoimmune, genetic,…]

8|Page
78. Lady on Cocs developed diarrhea for 1 day in 3
rd
wk. Advise?
[options: reassure & no action, continue+1wk extra cc, ommit pill free interval,…]
79. Lady on DMPA, came 3 wks post-due. Action?
[?give DMPA+ extra CC for 1 wk]
80. An epilyptic lady. What contraceptive measure is (?not) suitable?
[options?????]
81. Sexualy active women 19 yrs old asking for Cocs. When to start ?
[ options: start immediately, start & continue barriers for 7 days, start in the next cycle day 1-5 e
out barriers,…..]
82. From the list, what is the hormonal profile for Cat-3 WHO anovulatory disorders?
[hyper..hypo:↑FSH/LH, ↓E ]
83. A lady on Cu IUD for 3 yrs. Her routine Cx smear showed actinomyces. What to do?
[options: leave in-situ, remove,…..]
84. A woman on continuous combined HRT, came after 2m C/O spotting. Advice?
[options?????]
85. A girl on Coc pill ,came after 2m with irregular bleeds. Advice?
[options: increase estrogen from25 to 35, reassure, change pil,….]
86. The most suitable drug: a woman needs a drug for PMS. She has a PH of Ca breast & currently
on tamoxifine. B6 failed to improve her symptoms.
[? Options:?????SSRI?]
87. From the following, what is considered 1
st
line therapy for severe PMS:
[options: Cocs, SSRI, Amytriptaline, CBT,..]

9|Page
• STATISTICS:
88. Someone is studying about PMS, wants to know the prevalence, made questionnaires &
distributed among young girls. What type of study is this?
[cross-sectional]
89. 4 cell table given.. what is the possitive predictive value?
[a/a+b]
90. Because of full beds & increased admissions of PTL, a hospital is thinking of a test for PTL .
For
PTL to be more accurately diagnosed, what is the best predictor of the test?
[?specificity, that is, a test with the least false +ve]
91. Details of a? RCT were given (one group given m.dopa, other group given another drug.) RR &
P
value were given & they asked for interpretation. There was another Q. (? NNT or something).
[Options were :Convencing evidence of bennifits.. Convencing evidence of harm .. non ethical
trial... floughded trial e an trustable results ...]

• OBS:
92. 32 yrs , last pregnancy ended in c/s at 28wks with GA due to severe PE, birth wt was 650g.
NOW she is pregnant at ?24 Wks. ? what intervention to reduce the risk of PE/IUGR?
[Option varies from aspirin, folic, scan, vitamin c, nothing] *May be the question was that she
specifically asked for aspirin, whether it’ll be of value at this GA or not.
93. ?As above but GA 12 wks
94. A lady was referred from midwife for anxiety.
[options: CBT, refere to psychologist,….]
95. Type I Dm with nephropathy now 12 weeks. Interventions?
[options???]
96. DM + IUGR at 34wks. Management?
[options???? Dopplers,….]
97. Diabetic lady 36 weeks became unwell. Investigations showed ketosis & glucose is raised .

10 | P a g e
Management?
[? admit for fluids and insulin sliding scale]
98. From the following list, what biochemical figures are going with DKA?
[? ↓oxygen +↑ co2 +↓ bicarb]
99. Cystic hygroma on scan. what is the likely cause/association
[?turner]
100. Short femur length on scan. Likely cause. (or may be next step of action)
[?Down syndrome, many others,….]
101. Mother has anti-kell titer of 1:64, father is heterozygous for kell. Next step?
[options: repeat titers 2wkly, test maternal cFFDNA, refere to FM specialist, …..]
102. Rh-ve lady with 20 week pregnancy. U/S: Hydropic fetus.Ab titer of 1/64. Next step?
[?refere her to fetal medicine specialist.]
103. Thallasaemia ?major with Hb 98%, came for preconception session
[options: offer echo, offer MRI, offer chelation, give oral iron, give iron infusion, give packed RBC,
offer TOP,..….]
104. Thallasaemia ? intermedia + anemia. Management?
[options: as above]
105. Thallasemia, GA 8wks, C/O SOB at rest, ejection fraction↓
[options: as above] TOP?
106. Mode and time of delivery MCMA twins
[c/s at 32wks]
107. MCMA twins+ demise of twin 1 at 30wks: Next step in surveillance?
[?options: US, MRI, Mg sulphate,…….]
108. Diaphragmatic hernia on scan. Next step?
[karyo]
109. Woman 40yrs, p3, her grandmother has DVT. What thromboprophylaxis?
[options????? Was it ante or postnatal???]
110. After Instrumental delivery, came after 3 days with fever+urinary incontinence. Likely cause?

11 | P a g e
[? Options: UTI, fistula,..]
111. Cause of anemia: detected on 3
rd
TM. Woman found to have ↑BP.
[? hemolysis]
112. Cause of anemia: Woman on dialysis with history of HMB
[?↓ erythropoietin, chronic disease, blood loss,….]
113. APH: P3, last one c/s, amount?, GA 33 wks, came c/o mild bleeding & lower abd pain. Now
bleeding setteled. CTG normal. ?uterus irritable
[Options: Observe, steroids, ? Tocolytics+ observe, c/s, group & save, transfuse & observe,………]
114. APH: Another scenario, GA was 22 wks, breech.
[same options as above]
115. APH: Another scenario, GA was 29 wks, ? irritable uterus?
[same options as above]
116. Classification of the instrumental deliveries?
[options????]
117. What “maternal” drug can cause “neonatal” hypoglycemia.
[?labetalol]
118. Cause of death: multiparus women delivered fast and post partum developed severe
shortness
of breathing ..chest pain e clotless vaginal bleeding arrested and died?
[?AFE]
119. Cause of death: Tall pregnant pt . develope retrostenal chest pain radiating to scapula +SOB.
while complaining she suddenly collapsed & died.
[? Aortic dissection]
120. Pregnant with longstanding type-1 DM, present e retrostenal chest pain radiated to the jaw
[?MI]
121. What is the most common(?most risky) factor responsible for VTE recurrence ?
[options, prothrombin gene, anti thrombin 3 dif, FVL,…]

12 | P a g e
122. What treatment for a pregnant lady with severe falciparum malaria?
[i/v artesunate]
123. A pregnant lady presented with a breast lump. What is the 1
st
line investigation?
[options: FNA, mammography, breast U/S, CT,….]
124. An obese women with BMI 50, had a PH of difficult intubation & postpartum admission due to
infection. Now for CS. After induction of anathesia; she develop bronchial spasm
&hypotension & ↓ Po2 . What happened?
[options : air embolism, fat embolism, AFE, gastric aspiration, anaphylaxis, tension p.thorax,…]
125. From the following, what is the highest RF for P.previa?
[options: previus scar, advanced age, smoking,…]
126. From the following drugs, what could be a risk factor for abruption?
[? cocaine]
127. A lady came for the 1
st
episode of RFM. No risk factors in history. FH present on Doppler. Plan?
[?CTG; if normal-reassure]
128. An asthmatic lady for which her asthma became less controlled with short acting β agonist &
inhaled steroids. Next step?
[?inhaled long acting β agonist]
129. What test (or? Placental enzyme) that can predict pre-eclampsia?
[? Options: ↑PAPP-A, ↑ VEGF, ↓HCG, ↓sFIT,..]
130. From the following history, what is the main risk factor for stroke?
[?migraine]
131. What are the long term effects of pre-eclampsia?
[options: HTN, MI, stroke,…]

13 | P a g e
132. PPH: A 70 kg, 30m after delivery developed PPH. Now BP started to drop sh’s tahycardic &
slightly disorientd. From this information, the estimated loss would be?
[options: <500, 500-1000, 1000-1500, 1500-2000, 2000-2500,…..]
• LABOUR:
133. Baby went to shock after ventose delivery, HB 3g/dl, O/E diffuse ?regular shaped head
swelling. Cause?
[options: subgelial haematoma, cephalohematoma]
134. CTG: shows sudden deceleration down to 90 bpm,extended to the rest of ctg. Action?
[??? Cat-1 c/s (need to remember Cx status)]
135. CTG: induction of labour ..mild pain early may be 2cm ..ctg shows many early decelerations..
asking about the cause of the abnormaility.
[?Head compression. Myocardial ischemia was given???]
136. CTG: PPROM 34(or 37) wks. CTG sowed tachycardia then the ctg improved e good variability
and accelerations.
[? IOL, I/V fluids, oral antibiotics]
137. CTG: Pt. With epidural sited 5 min ago shows fetal brady cardia.
[? Reposition the women and observe for few more minutes].
138. CTG: 3hrs 2
nd
stage, head -2 ,mild caput, CTG shows ?late decelerations
[?c/s, forceps at room/theater,FBS,…..]
139. CTG: as above but head at spines:
[options: as above]
140. CTG: 2
nd
stage, CTG showed bradycardia, head +2, DOA
[?forceps in the room; up to 3pulls (NOT 4)]
141. Breach in labour, 6cm,membrane intact ..you feel cord presentation ..you decide c/s .. theatre

14 | P a g e
occupied. Action?
[? Terbutalin]
142. Breech in labour, decelerations at 4cm
[? c/s ]
143. PG + breech in labour. After hours; contractions are not adequate. Sacrum at -2. Action?
[options: oxytocin augmentation, cat1 c/s, cat2 c/s, cat3 c/s]
144. Breech + PPROM at 22 wks. Management?
[?expectant, expectant + steroids,…]
145. Shoulder dystocia,first maneuver?
[Mcrobert]
146. Claustrophobic with previous scar, ? asking for home delivery
[?options: counseling, CBT, refere to psych,..]
147. post partum: hallucinations, want to harm baby
[?admit to mother baby psychiatric unit, admit under mental health act,…]
148. The description of ? OP? DOP position was give. What diameter?
[?occipitofrontal for DOP, suboccipitofrontal for OP]
149. Ideal way to ascultate FHR in 1
st
stage?
[ every 15 minutes for 60 seconds.]
150. What measure can reduce the risk of perineal tear?
[options: Hands on technique, routine episiotomy, supine position, pelvic exercise,….. ]

151. What measure from the following can reduce the risk of instrumental delivery?
[?support in labour.]
152. ? Late booker, parous, GA 37 wks, “?footling” breech. Management?
[???offer ECV, offer electivec/s,…]
153. Labouring VBAC, sudden PV bleeding + tachycardia

15 | P a g e
[dehiscence]
• INFECTIONS:
154. What suitable antibiotic: Mastitis
[?flucloxacillin]
155. What suitable antibiotic and dose: GBS carrier, now in labour
[?benzyle penicillin 3g(5 M.U) then 1.5g(2.5 M.U) 4-hourly until delivery]
156. What suitable antibiotic: ?GAS sepsis
[options:]
157. Pt present with post partum genital sepsis ,what organism is most likely NOT ivolved?
[options: GAS, E.coliI, C.difficle, peptostrepticoccus, staph A.]
158. What test to confirm syphilis infection
[? FTA or something]
159. What test to confirm Chlamydia T infection
[? NAAT]
160. Young + multiple Sore vulval lesions+ dysuria. What virus?
[herpes]
161. Call from GP, pregnant woman got contact with chickenpox
[?test for IgG]
162. Call from GP, pregnant woman developed chickenpox
[?acyclovir]
163. 35 yrs on stable relationship, came C/O a painless growth around the vulva. Diagnosis?
[options: herpes, condylomata, tertiary syphilis, infectious syphilis]
164. Woman on Cocs, complain of tiny growths around the vulva. What virus?
[?HPV]
165. At 12 wks GA, lady came to know that her husband has HIV. She,s worried about her fetus.
Her booking test were negative. Action?
[options: test for sero conversion, reassure, start ART,…..]
166. A lady has flu like symptoms, then developed rash on the face then spread. What virus

16 | P a g e
[? rubella]
167. A lady has flu like symptoms, then developed rash on the face + arthralgia. What virus?
[?parvo]
168. Pregnant woman presented with 1ry herpes at 36 wks. Plan?
[?c/s at 39 wks]
169. Long scenario, at the end asked for treatment of syphilis
[benzathin p]
170. Mother has HIV. What’s the duration of neonatal zidovudin prophylaxis?
[4 wks]
• Genetics:
171. Couples, both are CF carrier has IVF + PGD. 12 embryos were formed. how many will be
affected?
[?options 3,4,6,8,….].
172. Haemophilia man + healthy lady. what is the risk of their son affected
[zero]
173. From the following, who has the highest risk for ca ovary?
[? Options: BRCA-1 carrier, BRCA-2 carrier, two 1
st
relatives with ca ovary,…..]
174. Prenatal diagnosis of del 22 (?? Digeorge syndrome)
[options: ? FISH (flurecent insitu hyberdization), PCR,….]
175.
Page 1:

17 | P a g e
MRCOG part 2 September 2015 Recalls
Please, if you have any amendments (more questions, more options, corrections), post at
whatsapp/telegram or email- m7md82@gmail.com for a better final recall.
In case that many options were recalled, the one that most of candidates thought as true is
highlighted yellow( but MAY NOT BE TRUE).

• INCIDENCES/FIGURES:
1. UAE was done for a 30 yrs old woman. % of those who will require re-intervention?
[25%]
2. Abroutpio recurrence %
[?4.4% after one time]
3. OASI incidence.
[?2.9%]
4. Risk of miscarriage after 3 consecutive miscarriages
[?40%]
5. What range is coded as very rare
[<1/10,000]
6. In what percent of PHA a cause can be found?
[options: 40-50%, 50-60%,……].
7. Risk of recurrence of shoulder dystocia above population in a woman with PH of dystocia.
[In folds: 10]
8. % of Ureteric injuries that can be identified during laparoscopy (intra-operatively):
[1:3]
9. Incidence of post dural tab/puncture; or duration of puncture headache?
[?0.5-2.5%, 7-10d]
10. Incidence of moderate to severe OHHS

18 | P a g e
[3-8%]
11. Risk of ovarian cancer in BRCA-1 carrier
[?40%]
12. Pt with PH of severe pre-eclampsia <28 wks, what is the chance of recurrence?
[1:2 (or 55%)]
13. Incidence of twins after successful IVF cycle?
[25% (1:4)]
14. what percentage of couples will succeed to conceive within the 1
st
yr?
[>80%]
15. Laproscopy ,serious complications
[2:1000]
16. A woman with atypical endometrial hyperplasia is planned for TAH. What is the risk of finding
a concomitant cancer?
[?40- 50%,…]
17. What is the incidence of OAB among the elderly ladies??
[?15%.]
• UROGYNE:
18. What is the most common presentation of vault prolapse ?
[options: bulge in vagina, sexual dysfunction, stress incontinence, constipation, voiding
dysfunction]
19. Cystoscopy [I don’t know how did they get the photos]: Your management?
[?Refere to urologist (? tumor)]
20. Cystoscopy, diagnosis? here Woman has 3 yrs history of urgency frequency, 2 or 3 times
culture positive, now no growth. Options were chronic cystitis, IC, schistosomiasis,……

19 | P a g e
[petechial haemarrhages =IC ?]

21. A woman presented for cystocele correction. She also complained of stress incontinence. O/E:
cystocele grade( ) , no demonstratable sress incontinence. Next step?
[options: urodynamics, video urodynamics, urodynamics with measuring of residual
volume,ant.repair…..]
22. 52 yrs women with recurrent UTI. UG shows microscopic haematuria and no growth. Next
step?
[?Cystoscopy]
23. The most suitable drug :?Elderly lady with OAB symptoms, ?mainly frequency, ?urodynamics
confirmed DI:
[? darifenacin]
• MMR/MBBRACE:
24. Commonest overall cause of MMR in 2008-2008 report?
[cardiac]
25. Long scenario, at the end asked for the commonest cause of neurological death in last MMR
report.
[? SAH]
26. How eclampsia kills according to new repor (? the most common cause of death in PET)?
[options : ICH, ARDS, uremia,…]
27.
• GYNE:
28. 9 years old girl brought C/O fever, nausea vomiting + pain in left side of pelvis.
[options: laproscopy, analgesia+ observation, treat PID , inform safeguarding services, refere to
surgeon,…]
29. A woman with IUD in-situ came with 5 wks amenorrhea & +ve PT. U/S: fundally locted sac no
fetal echo seen. 1
st

20 | P a g e
hcg around 600, repeated 48hrs around 1300. Most likely diagnosis?
[options: viable intrauterine, ectopic, PUL,…]
30. A woman with mild vaginal bleeding and mild lt lower abdominal pain. U/S shows a sac –
intrauterine- measuring around 20×20mm and a left ovarian cyst around 2cm. HCG around 900
iu. Most likely diagnosis?
[same options as above]
31. Anti-D dose after kleihauer test -done after blunt trauma- showing ≈ 4ml bleeding at 20wks GA
[?500 u]
32. EMQ: refere letter from a GP with results of HIV & 1
st
TM screening: (thanks to d.mutaz)
Birth date .../...1975
Uss : CRL= 54 mm head _ 11-12 w abd =12-13 wks
Hcg =... normal
PAPPA .24 MOM
RESULT : 1/ more than 350 for down
1/ 12 for edward
1/ 12 for pataue
There is a hint of possibility of edward and patau Syndrome..
Q1. this pt has also a high titre of HIV, what is your management? [?refer to FM unit]
Q2. You notice that the birth date of pt . Is different from the date written in the referral letter (
1972) how you manage this case? [? repeat all the investigations]
33. ;;;;;
34. A lady was referred with results of 1
st
TM screen. All withen normal except ?↑(?↓) PAPP-a.
Management?
[options:???? Serial growth scans, uterine Doppler at 20wks,…]

21 | P a g e
35. A lady with 1
st
TM screening result for down’s: ?1/15. She has a very high HIV viral load.
Management?
[options: AC, CVS, placental biopsy, refere to FM unit,..]
36. 74 y post ? coital [or PMB? ] bleeding. ET 3mm + some fluid in cavity.
[Options: D&C, hysteroscopy, hysteroscopic guided biopsy, pipelle, do nothing]
37. ?5th post op(TAH) day hematuria
[?options]
38. 10th post op day hematuria
[?options]
39. What abortion clause: young girl,…..(social reasons)
[C]
40. What abortion clause: anencephalic fetus
[E]
41. Suture material: for mass closure of periton & rectus sheeth after midline incision in obese lady
[ ?PDS 2]
42. Suture material: for repair of perineal skin.
[?2/0 vicryl]
43. Suture material: for re-anastomosisof faloppian tubes(reversal of sterilisation)
[?vicryl 4-0, 4-0 prolene with reverse cutting needle, ,..???]
44. Suture material: to close lap.port site
[j shaped needle]
45. Instrument used for: clamping the uterine artery/pedicle
[pictures rather than names: ?semicurved zepilin]
46. Instrument used for: ?deviding/dissecting the ureter during Wertheim surgery.
[?lahey’s ( loong forceps + small curve at the end)].
47. Instrument used for: handling/clamping the bowel during radical surgery

22 | P a g e
[? babcok]
48. Pst-op complications: Pt. after abdominal surgery presented few days later feeling unwell,
febrile, left hypochonrial pain radiated to the left shoulder. O/E stony dull percussion other
wise no other finding
[? Subphrenic abscess, pneumonia, bowel obstruction]
49. Post-op complications: Pt. After 4 hrs of retrieval of oocytes develope severe tachycardia and
hypotention
[?vessel injury]
50. Post-op complications: Pt. After 3 days of oocyte retrieval develooe abd pain e guarding and
rebound. O/E distended abdomen, ?sluggish bowel sounds
[? Bowel inj, paralytic ileus,…]
51. Post-op complications: 24 hrs post hysterectomy .. feeling unwell .tachycardia + mild fever ..
cathetr shows 50ml of dark red blood
[? Review fluid balance, return to theater,….]
52. Post-op complications: 5 days post hysterectomy she came with fever and haematuria
[?UTI]
53. Post-op complications: diabetic patient on PCA developed SOB. O/E pinpoint pupils. Bp 80/60.
Action?
[options: naloxon (opiate overdose), adrinalin, atropine, glucose]
54. Post-op complications: 7 days after hysterectomy she came with tender abdomen, left loin
pain and haematuria.
[?ureteric injury]
55. What is the main complication to notify: Laparoscopy for endometriosis
[? Options: ureter/bowel inj, bleeding, BT,……]
56. What is the main complication to notify: Hysterectomy for multiple fibroids
[? Options : as above] ?BT

23 | P a g e
57. Management options of [or prophylaxis from] vault prolapse with different 3 scenarios: An
elderly with comorbidities [? colpocleises], 2
nd
one with short vagina [ ?Sacrocolpopexy], & the
3
rd
in whom vault was reaching the introitus during VH [? SSF]
58. ;;;;
59. ;;;;;
60. What is the mech of action of tranixamic acid?
[?tissue plasminogen activator- inhibitor (ie anti-fibrinolytic)]
61. What is the main mech of action of Cocs?
[inhibition of ovulation]
62. What pair of hormones have very similar composition?
[? TSH/HCG]
63. What sign do you expect to find: obese girl BMI is 40 , mother DM type 1 .. has irregular cycle
and hirsutism ..
[?Acanthosis N]
• ONCOLOGY:
64. 55 years old undergone ?LLETZ cGIN with incomplete margin. Action?
[? hysterectomy]
65. After teated CIN, repeat HPV after 6 months was negative. Time of next smear?
[? 3 yrs]
66. Smear was borderline. HPV was +ve. After 6m, smear again borderline but HPV –ve
[? Routine recall]
67. Colposcopy showed ?leucoplakia, mosaicism, punctuations. Next step?
[options: biopsy , cone, LLETZ,…..]
68. 45 years old woman underwent MRI for (whatever reason). Incidentaly, a 6 cm simple ovarian

24 | P a g e
cyst was found. Management?
[???options]
69. while investigating an old post menopausal 67 years lady for bilateral complex ovarian cysts,
CA-125 was found to be 75. Management?
[????options]
70. A patient with stage 2b Ca Cx who refused treatment. What is the most relevant complication
to notify?
[? Renal failure, metastasis, bleeding,…..]
71. A woman with a huge u.fibroid that caused heavy periods, anemia, preesure
effect(?hydronephrosis). What is the most relevant complication to notify if she refused
surgery?
[same options as above]
• INFIRTILITY/ENDOCRINE/FAMILY PLANNING
72. Infertile couple,Women with normal investigations, husband with azospermia. Likely cause?
[klienfilter?]
73. Two embryo transfer in 1st cycle at what age/condition?
[?≥40]

74. EMQ asking about management of different presentations of PCO: 1


st
is a young lady not in a
relationship, main complain is irregular periods. She has normal BMI but her BP was 140/90.
[options: high dose progesteron, Cocs, metformin, Cocs+metformin, clomit citrate,….]
75. As above but lady is p3, with↑ BMI.
76. Cause of 2
nd
ry A
0

25 | P a g e
: 8 months after delivery with ↓FSH/LH & ↑↑ PRL(2000)
[options: pregnancy, sheehan’s, prolactinoma,…]
77. What is the commonest cause of POF
[?autoimmune, genetic,…]
78. Lady on Cocs developed diarrhea for 1 day in 3
rd
wk. Advise?
[options: reassure & no action, continue+1wk extra cc, ommit pill free interval,…]
79. Lady on DMPA, came 3 wks post-due. Action?
[?give DMPA+ extra CC for 1 wk]
80. An epilyptic lady. What contraceptive measure is (?not) suitable?
[options?????]
81. Sexualy active women 19 yrs old asking for Cocs. When to start ?
[ options: start immediately, start & continue barriers for 7 days, start in the next cycle day 1-5 e
out barriers,…..]
82. From the list, what is the hormonal profile for Cat-3 WHO anovulatory disorders?
[hyper..hypo:↑FSH/LH, ↓E ]
83. A lady on Cu IUD for 3 yrs. Her routine Cx smear showed actinomyces. What to do?
[options: leave in-situ, remove,…..]
84. A woman on continuous combined HRT, came after 2m C/O spotting. Advice?
[options?????]
85. A girl on Coc pill ,came after 2m with irregular bleeds. Advice?
[options: increase estrogen from25 to 35, reassure, change pil,….]
86. The most suitable drug: a woman needs a drug for PMS. She has a PH of Ca breast & currently
on tamoxifine. B6 failed to improve her symptoms.
[? Options:?????SSRI?]
87. From the following, what is considered 1
st

26 | P a g e
line therapy for severe PMS:
[options: Cocs, SSRI, Amytriptaline, CBT,..]

• STATISTICS:
88. Someone is studying about PMS, wants to know the prevalence, made questionnaires &
distributed among young girls. What type of study is this?
[cross-sectional]
89. 4 cell table given.. what is the possitive predictive value?
[a/a+b]
90. Because of full beds & increased admissions of PTL, a hospital is thinking of a test for PTL .
For
PTL to be more accurately diagnosed, what is the best predictor of the test?
[?specificity, that is, a test with the least false +ve]
91. Details of a? RCT were given (one group given m.dopa, other group given another drug.) RR &
P
value were given & they asked for interpretation. There was another Q. (? NNT or something).
[Options were :Convencing evidence of bennifits.. Convencing evidence of harm .. non ethical
trial... floughded trial e an trustable results ...]

• OBS:
92. 32 yrs , last pregnancy ended in c/s at 28wks with GA due to severe PE, birth wt was 650g.
NOW she is pregnant at ?24 Wks. ? what intervention to reduce the risk of PE/IUGR?
[Option varies from aspirin, folic, scan, vitamin c, nothing] *May be the question was that she
specifically asked for aspirin, whether it’ll be of value at this GA or not.
93. ?As above but GA 12 wks
94. A lady was referred from midwife for anxiety.
[options: CBT, refere to psychologist,….]
95. Type I Dm with nephropathy now 12 weeks. Interventions?

27 | P a g e
[options???]
96. DM + IUGR at 34wks. Management?
[options???? Dopplers,….]
97. Diabetic lady 36 weeks became unwell. Investigations showed ketosis & glucose is raised .
Management?
[? admit for fluids and insulin sliding scale]
98. From the following list, what biochemical figures are going with DKA?
[? ↓oxygen +↑ co2 +↓ bicarb]
99. Cystic hygroma on scan. what is the likely cause/association
[?turner]
100. Short femur length on scan. Likely cause. (or may be next step of action)
[?Down syndrome, many others,….]
101. Mother has anti-kell titer of 1:64, father is heterozygous for kell. Next step?
[options: repeat titers 2wkly, test maternal cFFDNA, refere to FM specialist, …..]
102. Rh-ve lady with 20 week pregnancy. U/S: Hydropic fetus.Ab titer of 1/64. Next step?
[?refere her to fetal medicine specialist.]
103. Thallasaemia ?major with Hb 98%, came for preconception session
[options: offer echo, offer MRI, offer chelation, give oral iron, give iron infusion, give packed RBC,
offer TOP,..….]
104. Thallasaemia ? intermedia + anemia. Management?
[options: as above]
105. Thallasemia, GA 8wks, C/O SOB at rest, ejection fraction↓
[options: as above] TOP?
106. Mode and time of delivery MCMA twins
[c/s at 32wks]
107. MCMA twins+ demise of twin 1 at 30wks: Next step in surveillance?
[?options: US, MRI, Mg sulphate,…….]
108. Diaphragmatic hernia on scan. Next step?

28 | P a g e
[karyo]
109. Woman 40yrs, p3, her grandmother has DVT. What thromboprophylaxis?
[options????? Was it ante or postnatal???]
110. After Instrumental delivery, came after 3 days with fever+urinary incontinence. Likely cause?
[? Options: UTI, fistula,..]
111. Cause of anemia: detected on 3
rd
TM. Woman found to have ↑BP.
[? hemolysis]
112. Cause of anemia: Woman on dialysis with history of HMB

[?↓ erythropoietin, chronic disease, blood loss,….]


113. APH: P3, last one c/s, amount?, GA 33 wks, came c/o mild bleeding & lower abd pain. Now
bleeding setteled. CTG normal. ?uterus irritable
[Options: Observe, steroids, ? Tocolytics+ observe, c/s, group & save, transfuse & observe,………]
114. APH: Another scenario, GA was 22 wks, breech.
[same options as above]
115. APH: Another scenario, GA was 29 wks, ? irritable uterus?
[same options as above]
116. Classification of the instrumental deliveries?
[options????]
117. What “maternal” drug can cause “neonatal” hypoglycemia.
[?labetalol]
118. Cause of death: multiparus women delivered fast and post partum developed severe
shortness
of breathing ..chest pain e clotless vaginal bleeding arrested and died?
[?AFE]
119. Cause of death: Tall pregnant pt . develope retrostenal chest pain radiating to scapula +SOB.
while complaining she suddenly collapsed & died.

29 | P a g e
[? Aortic dissection]
120. Pregnant with longstanding type-1 DM, present e retrostenal chest pain radiated to the jaw
[?MI]
121. What is the most common(?most risky) factor responsible for VTE recurrence ?
[options, prothrombin gene, anti thrombin 3 dif, FVL,…]
122. What treatment for a pregnant lady with severe falciparum malaria?
[i/v artesunate]
123. A pregnant lady presented with a breast lump. What is the 1
st
line investigation?
[options: FNA, mammography, breast U/S, CT,….]
124. An obese women with BMI 50, had a PH of difficult intubation & postpartum admission due to
infection. Now for CS. After induction of anathesia; she develop bronchial spasm
&hypotension & ↓ Po2 . What happened?
[options : air embolism, fat embolism, AFE, gastric aspiration, anaphylaxis, tension p.thorax,…]
125. From the following, what is the highest RF for P.previa?
[options: previus scar, advanced age, smoking,…]
126. From the following drugs, what could be a risk factor for abruption?
[? cocaine]
127. A lady came for the 1
st
episode of RFM. No risk factors in history. FH present on Doppler. Plan?
[?CTG; if normal-reassure]
128. An asthmatic lady for which her asthma became less controlled with short acting β agonist &
inhaled steroids. Next step?
[?inhaled long acting β agonist]
129. What test (or? Placental enzyme) that can predict pre-eclampsia?
[? Options: ↑PAPP-A, ↑ VEGF, ↓HCG, ↓sFIT,..]

30 | P a g e
130. From the following history, what is the main risk factor for stroke?
[?migraine]
131. What are the long term effects of pre-eclampsia?
[options: HTN, MI, stroke,…]
132. PPH: A 70 kg, 30m after delivery developed PPH. Now BP started to drop sh’s tahycardic &
slightly disorientd. From this information, the estimated loss would be?
[options: <500, 500-1000, 1000-1500, 1500-2000, 2000-2500,…..]

• LABOUR:
133. Baby went to shock after ventose delivery, HB 3g/dl, O/E diffuse ?regular shaped head
swelling. Cause?
[options: subgelial haematoma, cephalohematoma]
134. CTG: shows sudden deceleration down to 90 bpm,extended to the rest of ctg. Action?
[??? Cat-1 c/s (need to remember Cx status)]
135. CTG: induction of labour ..mild pain early may be 2cm ..ctg shows many early decelerations..
asking about the cause of the abnormaility.
[?Head compression. Myocardial ischemia was given???]
136. CTG: PPROM 34(or 37) wks. CTG sowed tachycardia then the ctg improved e good variability
and accelerations.
[? IOL, I/V fluids, oral antibiotics]
137. CTG: Pt. With epidural sited 5 min ago shows fetal brady cardia.
[? Reposition the women and observe for few more minutes].
138. CTG: 3hrs 2
nd
stage, head -2 ,mild caput, CTG shows ?late decelerations
[?c/s, forceps at room/theater,FBS,…..]
139. CTG: as above but head at spines:

31 | P a g e
[options: as above]
140. CTG: 2
nd
stage, CTG showed bradycardia, head +2, DOA
[?forceps in the room; up to 3pulls (NOT 4)]
141. Breach in labour, 6cm,membrane intact ..you feel cord presentation ..you decide c/s .. theatre
occupied. Action?
[? Terbutalin]
142. Breech in labour, decelerations at 4cm
[? c/s ]
143. PG + breech in labour. After hours; contractions are not adequate. Sacrum at -2. Action?
[options: oxytocin augmentation, cat1 c/s, cat2 c/s, cat3 c/s]
144. Breech + PPROM at 22 wks. Management?
[?expectant, expectant + steroids,…]
145. Shoulder dystocia,first maneuver?
[Mcrobert]
146. Claustrophobic with previous scar, ? asking for home delivery
[?options: counseling, CBT, refere to psych,..]
147. post partum: hallucinations, want to harm baby
[?admit to mother baby psychiatric unit, admit under mental health act,…]
148. The description of ? OP? DOP position was give. What diameter?
[?occipitofrontal for DOP, suboccipitofrontal for OP]
149. Ideal way to ascultate FHR in 1
st
stage?
[ every 15 minutes for 60 seconds.]
150. What measure can reduce the risk of perineal tear?
[options: Hands on technique, routine episiotomy, supine position, pelvic exercise,….. ]

32 | P a g e
151. What measure from the following can reduce the risk of instrumental delivery?
[?support in labour.]
152. ? Late booker, parous, GA 37 wks, “?footling” breech. Management?
[???offer ECV, offer electivec/s,…]
153. Labouring VBAC, sudden PV bleeding + tachycardia
[dehiscence]

• INFECTIONS:
154. What suitable antibiotic: Mastitis
[?flucloxacillin]
155. What suitable antibiotic and dose: GBS carrier, now in labour
[?benzyle penicillin 3g(5 M.U) then 1.5g(2.5 M.U) 4-hourly until delivery]
156. What suitable antibiotic: ?GAS sepsis
[options:]
157. Pt present with post partum genital sepsis ,what organism is most likely NOT ivolved?
[options: GAS, E.coliI, C.difficle, peptostrepticoccus, staph A.]
158. What test to confirm syphilis infection
[? FTA or something]
159. What test to confirm Chlamydia T infection
[? NAAT]
160. Young + multiple Sore vulval lesions+ dysuria. What virus?
[herpes]
161. Call from GP, pregnant woman got contact with chickenpox
[?test for IgG]
162. Call from GP, pregnant woman developed chickenpox
[?acyclovir]
163. 35 yrs on stable relationship, came C/O a painless growth around the vulva. Diagnosis?

33 | P a g e
[options: herpes, condylomata, tertiary syphilis, infectious syphilis]
164. Woman on Cocs, complain of tiny growths around the vulva. What virus?
[?HPV]
165. At 12 wks GA, lady came to know that her husband has HIV. She,s worried about her fetus.
Her booking test were negative. Action?
[options: test for sero conversion, reassure, start ART,…..]
166. A lady has flu like symptoms, then developed rash on the face then spread. What virus
[? rubella]
167. A lady has flu like symptoms, then developed rash on the face + arthralgia. What virus?
[?parvo]
168. Pregnant woman presented with 1ry herpes at 36 wks. Plan?
[?c/s at 39 wks]
169. Long scenario, at the end asked for treatment of syphilis
[benzathin p]
170. Mother has HIV. What’s the duration of neonatal zidovudin prophylaxis?
[4 wks]

• Genetics:
171. Couples, both are CF carrier has IVF + PGD. 12 embryos were formed. how many will be
affected?
[?options 3,4,6,8,….].
172. Haemophilia man + healthy lady. what is the risk of their son affected
[zero]
173. From the following, who has the highest risk for ca ovary?
[? Options: BRCA-1 carrier, BRCA-2 carrier, two 1
st
relatives with ca ovary,…..]
174. Prenatal diagnosis of del 22 (?? Digeorge syndrome)

34 | P a g e
[options: ? FISH (flurecent insitu hyberdization), PCR,….]
175.

MRCOG part 2 March 2016 Recalls 1st edition, amenable for


modifications A mere collection of a marvelous group’s
work INCIDENCES/FIGURES: [All were SBAs]

1. Recurrence rate of AFLP. -25%


2. Risk of congenital heart block if mother has SLE with both anti-ro/la.- 2%
3. GA at which monitoring starts for TTTF. A. 14 B. 16 - correct C. 18 D. 20 E. 24 wks
4. Life time risk of pelvic organ prolapse.-40%
5. Risk of adhesion formation with midline incisions. A. 10 B. 20 C. 30 D. 40 E. 50%- correct
6. Incidence of dural tap A. 0.5-1-correct B. 0.5-2.5 C. 1-3 D. 2-5%
7. Cut-off threshold level of lactate to diagnose [?manage] severe sepsis. A. 2 B. 4-correct C. 6 D.
8 E. 10
Page 1 of 61
Past recalls March 2016 8. A primi has opted for home delivery. According to NICE; there is ….%
risk for hospital transfer. A. 15- correct B. 25 C. 35 D. 45 E. 55%
9. Risk of SUI after VD [or after CS].-4%
10. Incidence of steroid resistant LS.-4 to 10%
11. % of vertical transmission of parvovirus at? 15 wks GA.
Ans- Risk of transmission of Parvo virus is Under 15 weeks-15% 15-20 weeks- 25% Term -70%
12. Risk of a simple cyst in premenopausal lady to be malignant
Ans- 1:1000 Gtg-The overall incidence of a symptomatic ovarian cyst in a premenopausal female
being malignant is approximately 1:1000 increasing to 3:1000 at the age of 50.
13. Duration of thromboprophylaxis after TAH A. 24h B. 5-7 days C. 2wks D. 4 wks E. ?
Ans- B. 5-7 days Nice guidelines- Continue pharmacological VTE prophylaxis until the patient no
longer has significantly reduced mobility (generally 5–7 days). Extend pharmacological VTE
prophylaxis to 28 days postoperatively for patients who have had major cancer surgery in the
abdomen or pelvis. ………………………………………………………………………………

35 | P a g e
Page 2 of 61
Past recalls March 2016 ………………………………………………………..
UROGYNE:
14. 57 yrs, diagnosed as a case of MS. Now complain of voiding difficulties. ? Advise Most of us
opted CISC, couldn’t remember the options well.
Ans- ?intermittent self catheterisation. Cisc could be done later if intermittent doesn't work out I
feel. Pt info leaflet-persistent conditions (eg, neurological conditions), self-catheterisation
techniques may be learned .
15. A scenario was give for a lady with symptoms of OAB, asked for 1st step. A. PFMT B. Bladder
drill C. Drugs D. Urodynamics E. ?
Ans- B. Bladder drill Nice -Offer bladder training lasting for a minimum of 6 weeks as first-line
treatment to women with urgency or mixed UI.
16. A 50 yrs old lady presented with symptoms of voiding difficulties. 1st invest? A. UG B. Urine
culture C. US for PVR D. Urodynamics.
Ans- C I didn't understand what option A meant Nice-Undertake a urine dipstick test in all women
presenting with UI. Measure post-void residual volume by bladder scan or catheterisation in women
with symptoms suggestive of voiding dysfunction or recurrent UTI. Use a bladder scan in
preference to catheterisation on the grounds of acceptability and lower incidence of adverse events
17. 80 yrs old with OAB. Bladder training was not effective. The most suitable drug?
A. Immediate release oxybutinin B. Transdermal oxybutinin C. Paroxitin D. Duloxtine
E. Tropsium
Ans- E
Nice-Do not offer oxybutynin (immediate release) to frail older women. Oxybutynin crosses bbb and
causes cognitive defects. tolterodine is usually offered for old women but it's not there in the
option. Duloxetine offered as 2nd line in stress incontinence if patient doesn't want surgery.
Paroxetine is a ssri. It causes urinary Incontinence as an adverse effect. -Trospium is an
antispasmodic and antimuscarinic agent used in treatment of incontinence.
18. ? yrs old with OAB. All antimuscarincs were not tolerated by her. Next suitable drug? A. TCA B.
SSRI C. Mirabegron D. Duloxitine E. ?
Ans-c
19. ? yrs old with genuine SI. PFMT failed to improve her symptoms. She denied surgery. What drug
can be given to her? A. Oxybutinin B. Paroxitin C. Duloxitin D. ? E. ?
Ans- C

36 | P a g e
………………………………………………………………………………
………………………………………………….
EMQ: “The most appropriate mng plan” Options: A. Bladder training B. PFMT C. Bladder training
and PFMT D. drugs [many ere given] E. Other options
20. A scenario where the lady complains of frequency, urgency, nocturia + stress incontinence.
Ans- A she has mixed incontinence. Nice -Offer bladder training lasting for a minimum of 6 weeks
as first-line treatment to women with urgency or mixed UI.
EMQ: Most likely cause for the following urodynamic results:
Options: A. Genuine SI
B. Urodynamic SI
C. OAB
D. DO
E. Mixed UI
F. Cystocele
G. Chronic retention with overflow
H. Diabetic neuropathy
I. Fistula
J. ??Other options 1st desire Bladder volume detruser pressure on filling flow rate/s PVR 21. 90 ml
200 30 ? normal 50
Ans- C . DO Here detrussor pressure on filling is more.( normal-15) rest all are normal
22. 100 300 30 + leak on stress 20 50
Ans: E. Mixed Here both detrussor pressure raised along with stress incontinence
23. 150 400 10+ continuous leak ? 2
Ans- I Here all indices normal ... Only continuous leak is the catch word
…………………………………………………………………

37 | P a g e
24. [? May be an EMQ] An obese bus driver, 40s yrs old, BMI 40, allergic to peanut, suddenly
collapsed & died despite resuccitation. The most likely underlying cause of death is
A. Anaphylaxis.
B. Pulmonary embolism
C. ??Other options
……………………………………………………………………………… …………
EMQ. Categorization of maternal death Options:
A. Not a maternal death
B. Direct
C. Indirect
D. Co-incidental
E. Accidental
F. Late direct
G. Late indirect
H. Late co-incidental

25. Pre-eclampsia, developed ICH, died 5 wks after delivery.


B. Direct

26. Died 2 days post evacuation of IM. Post mortem identified Eisinmenger complex.
C. Indirect

27. A pregnant lady murdered by her x-husband at 24 wks GA.


D. Co-incidental

28. H1N1 infection, died of sepsis 5 wks after delivery.


C. Indirect

38 | P a g e
GYNE-EARLY PREGNANCYØ

29. What is the route of Misoprostol with the earliest onset, fastest & longest duration of action?
A. Oral
B. Sublingula- correct
C. Buccal
D. Rectal
E.vaginal

30. US at 10 wks showed herniation of gut with umbilical cord on top of sac. Diagnosis?
A. Omphalocele
B. Gastrochesis
C. Physiological ( occurs bet 9-12 weeks)
D. ?

31. 41 yrs. US showed GA= 13+4 wks. What is the most appropriate screening method for down’s?
A. Combined test.
B. Triple test
C. Quadruple test
D. NT
E. ?
32. Early scan for a woman who isn’t sure of her dates. Measures were as follows:
CRL: 90mm HC:111mm BPD:98mm FL: ?? AC: ?? Which diameter should be used to determine her
GA?
A. CRL
B. HC
C. BPD
D. FL
E. AC

39 | P a g e
EMQ. The most appropriate next step of action for a lady who presented to EPAU
complaining of 7 wks amenorrhea + mild lower abd cramps & spotting. PT +ve. Options:
A. Diagnose ectopic & arrange laparoscopy
B. Offer medical management of miscarriage
C. Offer methotrexate
D. Measure HCG level in serum now & repeat after 48 h
E. Offer US after 1 wk
F. Offer US after 2 wks
G. Other options
33. US showed single intrauterine GS measuring 25x23x28 mm. No fetal pole.
Ans- E
Nice
If the mean gestational sac diameter is 25.0 mm or more using a transvaginal ultrasound scan and
there is no visible fetal pole:
1.seek a second opinion on the viability of the pregnancy and/or
2.perform a second scan a minimum of 7 days after the first before making a diagnosis.
34. US showed a 10 mm central echo in the uterine cavity. There is a hemorrhagic Rt.ovarian cyst
[?measures] + small amount of fluid in POD.
Ans- ?D . I don't know for sure Central echo in uterine cavity means ectopic preg . Without the size
difficult to say the answer.
35. US showed intrauterine GS [? + yolk sac present]. There is small amount of fluid in POD plus
4x3 echogenic mass near the Rt.ovary. GYNE:
Ans- A This is heterotrophic preg.
36. What is the most common site for uterine perforation during evacuation?
A. Ant. Wall
B. Post.wall
C. Cx canal
D. Fundus
E. ?
Ans- A Tog

40 | P a g e
Sites of perf Ant wall- 40% Cx canal-36% Rt lat wall-21% Left lat wall-17% Post wall n fundus-13%
37. Inferior epigastric artery is a branch of: A. Ext.ileac B. Int. ileac C. Femoral D. Int. thoracic E. ?
Ans- A
38. What is the commonest complication of surgical evacuation? A. Peroration B. Infection C.
Bleeding requiring BT D. Cx injury/laceration E. Effects on future fertility.
Ans- B Consent advice no 10
A. Peroration- 5:1000
B. Infection-3:100
C. Bleeding requiring BT- 1-2:1000
D. Cx injury/laceration-rare
E. Effects on future fertility.- no evidence
39. A lady came from abroad, admitted genital cutting as a child. O/E: infibulation. How to report
this in her notes? A. Circumcision B. FGM type 1 C. FGM type 2 D. FGM type 3 E. FGM type 4
Ans- 3 GtgType
1: Partial or total removal of the clitoris and/or the prepuce (clitoridectomy).
Type 2: Partial or total removal of the clitoris and the labia minora, with or without excision of the
labia majora (excision).
Type 3:Narrowing of the vaginal orifice with creation of a covering seal by cutting and
appositioning the labia minora and/or the labia majora, with or without excision of the clitoris
(infibulation).
Type 4: All other harmful procedures to the female genitalia for non-medical purposes, for example:
pricking, piercing, incising, scraping and cauterization.
40. A lady has sustained a perineal tear in which 50% of the EAS was involved. What degree?
A. 2nd
B. 3a
C. 3b
D. 3c
E. 4th
Ans- D GtgGrade 3a tear: Less than 50% of external anal sphincter (EAS) thickness torn. Grade 3b
tear: More than 50% of EAS thickness torn. Grade 3c tear: Both EAS and internal anal sphincter
(IAS) torn.

41 | P a g e
41. For a lady with adenomyosis, what is the most likely other uterine pathology to find on US?
A. Endometrial polyp
B. Fibroid
C. Endometrial hyperplasia
D. Endometriosis
E. ?
Ans- b Myomas present in 30-55% and endometriosis in 6-22% Ref- uterine adenomyosis (text
book) by springer publications.
41. For a lady with adenomyosis, what is the most likely other uterine pathology to find on US?
A. Endometrial polyp
B. Fibroid
C. Endometrial hyperplasia
D. Endometriosis
E. ?
TOG 2009 - Adenomyosis Disease % Leiomyomas 20.5–70 Pelvic endometriosis 6.3–24 Salpingitis
isthmica nodosa 1.4–19.8 Endometrial polyps 2.3–14.7
Endometrial hyperplasia 7.3–13.6 Endometrial hyperplasia with atypia 3.5 Adenocarcinoma 2.2–5.3
………………………………………………………………………………
EMQ: Unexpected finding. Most appropriate action Options:
A. Abandon procedure & plan future management
B. Proceed as planned/consented for
C. Ask a collegue D. Meet with next of kin
E. Remove X & Y
F. Remove Y from X
G. Take biopsy from Y & close
H. Remove Y.
I. Other options

42 | P a g e
42. The surgeon has opened the young girl as acute appendicitis. The signed consent included
laparoscopy + removal of the appendix ± laparotomy. The appendix was found normal but there
was an uncomplicated ?cm cyst [y] on the Rt.ovary [x]. The surgeon decided to leave the appendix
& called you for the cyst.
A. Abandon procedure & plan future management

43. A woman is undergoing laparoscopic TAH for HMB, signed for laparoscopy + removal of the
uterus ± laparotomy.
A cyst [y] was found on the Rt.ovary [X].
B. Proceed as planned/consented for
44. There was a 3rd scenario where a “structure” rather than cyst was found LMP was 6 weeks back
and UPT was positive, Lt ovarian mass
( I think it should be ectopic in lt ovary so remove Y from X
……………………………………………………………….
EMQ: Vascular injury. What vessel Options: Actually internal ileac & ALL its branches
A. Internal ileac
B. Superior gluteal
C. middle rectal
D. obturator
E. internal pudendal
F. vesical
G. ??
Other branches of the internal ileac were also given.

45. After difficult high forceps, there was profuse bleeding. On exploration there was arterial
bleeding coming right upper pelvis ABOVE piriformis. By now the patient has bled 4 L & the
radiologist was called urgently for embolisation.
B. Superior gluteal ANSWER

43 | P a g e
46. Heavy bleeding after MIDLINE episiotomy. E. internal pudendal ( inferior rectal – branch of it) In
midline episotony the injury will be inferior rectal artery a branch of internal pudendal artery

47. Heavy bleeding during SSF upon fixing sutures to the the Rt ischial spine. E. internal pudendal
(inferior gluteal artery … branch of it) SSF.... inferior gluteal artery
………………………………………………………………………………
…………………………………………………………..
EMQ: The likely diagnosis: Options
A. IBS
B. PID
C. Chronic pelvic pain syndrome
D. Interstitial cystitis
E. Many other options

48. A young lady complaining of persistent lower abd pain. When about to do pelvic exam she
became agitated & admiited child abuse.
Answer …C. Chronic pelvic pain syndrome

49. A young sexually active lady complaining of lower abd pain associated with fever, profuse
discharge
Answer…..B. PID

50. ? There was a 3rd stem here.

EMQ: Post op complications. The most likely cause.


51. Options
A. included Internal bleeding
B. Bowel injury, Bladder/ureter inj
C. Vault hematoma
D. Infected vault hematoma/?abscess

44 | P a g e
E. Opiate toxicity
F. Chest infection
G. PE
H. Many other options

52. TAH. Day 5, unwell, slow to mobilize, swinging fever, mild abd distention but +be bowel sounds.
Wond minimally bruised. ? 2 episodes of diarrhea. Vitals given were ? within normal apart from ↑T.
[This stem may belong to the next EMQ, asking for most appropriate next step rather than
diagnosis]
Answer……D. Infected vault hematoma/?abscess

53. TAH. Day 2, unwell, drowsy. Exam mostly normal. Vitals given within normal but RR 6/m. Hb is 8
gm/dl [pre op ? 10]. E. Opiate toxicity

EMQ: Post op complications. The most appropriate next step. Options

A. Start therapeutic LMWH


B. Initiate septic screen
C. Start IV antibiotics
D. Invite the surgeon
E. Ask for medical opinion
F. Review drugs chart
G. Request CXR/ABG
H. Consult microbiologist

54. TAH. Day? 3. Fever, cough, SOB. Chest exam: ↓ air entery, crackles & signs of consolidation.
Answer……..G. Request CXR/ABG

45 | P a g e
55. ? There was a 2nd stem here. ………………………………………………………………………………
…………………………………………………………
EMQ: The most appropriate next step

Options
A. Flucloxacillin + metro
B. Another combination of antibiotic [? Including Co-amoxiclav]
C. secondary suturing
D. repair of the torn anal sphincter
E. Offer endoanal US + manometry
F. Many other options.
56. 4 days after water birth, the lady came back with her episiotomy wound gaping. On exam
sutures exposed, gapped wound with creamy discharge but no sign of infection.à
Answer……. C. secondary suturing

57. A woman has sustained a 3rd degree tear in labour. On her follow up visit there was mild
incontinence to flatus & soft stool. PR showed weak anal tone.
Answer…..E. Offer endoanal US + manometry
58. An elderly lady complaining of persistent lower abd pain. US showed normal uterus &
endometrial thickness, normal Rt ovary. Lt ovary was NOT seen [?due to ga]. Next step in
investigations?
A. CT
B. MRI
C. PET
D. US survey
E. ?
Ans- ? B Gtg says if Usg inconclusive go for mri

46 | P a g e
59. HPV testing came negative after a routine smear showing mild dyskaryosis. Next step?
A. Colposcopy referral
B. Smear after 6m
C. Smear after 12 m
D. Routine recall with GP after 3 yrs
E. ?
Ans-D
Ref- colposcopy guidelines

60. PMB. Biopsy showed Crouded gland in “back to back ” fasion, minimal strom, NO cellular
atypia. Likely diagnosis?
A. Endometrial carcinoma
B. Simple EH
C. Complex EH
D. ?others
Ans- C
Ref tog

61. 60 yrs. PMB. Biopsy showed Hyperplasia without atypia. Best plan of mng?
A. TAH
B. TAH + BSO
C. Mirena
D. Oral progesterons
E. ?
Ans- C
Ref- gtg

47 | P a g e
Mirena is 1st line mgt. regression rates are mirena- 84 to 100% and oral preg( mpa) - 50 to 64%.
Continue 6 monthly surveillance until 2 negative biopsies .
62. An elderly lady presented with a sore itchy vulval lesion 3cm ulcer with a clear margin in Lt
labium majus. The most likely diagnosis?
A. VIN
B. Squamous cell carcinoma
C. Syphilis
D. Keratoacanthoma
E. ?
Ans- B
Symptoms of vulval cancer include vulval itching, irritation or pain.
Vulval cancer is most common among women over 65 years old.
But I'm concerned about the clear margin mentioned . Usually vulval ca ulcers have irregular
margins if I'm not mistaken
Tog - Sqc is UNIFOCAL and on labia majora
………………………………………………………………………………
……………………………………………………………….
EMQ: Post menopause + ovarian cyst. Most appropriate management Options:
A. Discharge from follow up
B. Laparoscopic cystectomy
C. Laparoscopic oophorectomy
D. Laparoscopic BSO
E. US follow up after 4 m
F. US follow up after 1 yr
G. Staging laparotomy
H. ?Others.
63. Simple Rt. ovarian cyst. Ca-125=?10.
Ans- E

48 | P a g e
Gtg- Simple, unilateral, unilocular ovarian cysts, less than 5 cm in diameter, have a low risk of
malignancy. It is recommended that, in the presence of a normal serum CA125 levels, they be
managed conservatively. 50% resolve in 3 months . Risk of malig less than 1%.
follow-up ultrasound scan for cysts of 2–5cm, a reasonable interval being four months

64. 4cm cyst in Rt ovary, ? bilocular + small solid area. Ca-125=10


Ans- D
Rmi is 90. Lap oopherectomy in cancer unit. It is recommended that laparoscopic management of
ovarian cysts in postmenopausal women should involve oophorectomy (usually bilateral) rather
than cystectomy.
65. Simple 4.5 cm cyst in Rt ovary. In her records by her GP 1 yr ago, same findings but she didn’t
come back for follow up.
Ans- A
………………………………………………………………………………
………………………………………………………………….
INFIRTILITY/ENDOCRINE/FAMILY PLANNING:
66. A lady with subfertility. Given her picture of HSG [not the same picture, indeed] What is the
most likely urinary tract anomaly to associate with this condition?
A. Renal agenesis
B. Pelvic kidney
C. Horseshoe kidney
D. Ectopic ureter
E. Double ureter
Ans- ?
67. Infertile couple. Man severe oligo [1.5 million] with small testes, ↑ FSH/LH. Woman has regular
period. Progesteron level 30. PH of Chlamydia infection. What is the most appropriate next step in
mng?
A. Testing man for Karyo + y del
B. Testing female for FSH/LH
C. HSG woman
D. LAP/dye woman

49 | P a g e
Ans- A
Could be hypergonadotropic hypogonadism in male. Female is ovulating normally.
68. Infertile couples. Man azo with ↑ FSH & LH. All woman tests including tubes are normal. Their
best option?
A. IVF with partner sperm
B. IVF with donor sperm
C. IUI with donor semen
D. ?
Ans- ?c
69. Infertile couples. Man previously fertile but now azo with ↓ FSH. Possible cause?
A. Klienfilter
B. Kallman
C. Cryptorchidism
D. Anabolic steroids
E. ?
Ans- D
Exogenous steroids causing suppression of fsh.

70. Young couples, Trying for 2 ½ yrs. All investigations are normal. Best option?
A. Advise to wait another 6m
B. Repeat tests
C. Offer IVF
D. Offer ov.induction with clomit citrate
E. ?
Ans- C
Nice -Offer IVF treatment to women with unexplained infertility who have not conceived after 2
years

50 | P a g e
71. SBA: Infertile couple. Man’s tests normal. Woman’s normal but HSG hydrosalpinx. Your mng?
A. Salpingectomy + IVF
B. Occlusion by clips + IVF
C. ???
Ans: A. Salpingectomy + IVF Nice: Fertility

72. Young lady presented irregular periods + progressive hirsutism. US polycystic ovaries.
Testosteron level 7 with normal free androgen index. The most appropriate next investigation?
A. DHEA
B. DHEAS
C. 17(OH)P
D. ??
Ans: C. 17(OH)P Late onset CAH Ref:StratOG
73. A post menopausal woman presented with PMB + progressive hirsutism. Her husband noticed
deepening of her voice. Testosteron level 11. The mostly likely cause?
A. PCO
B. Androgen secreting adrenal tumor
C. Cushing syndrome
D. Ovarian hyperthecosis.
Ans: D. Ovarian hyperthecosis. Ref:StratOG

74. What AED concentration is affected by COCs?


A. Carbam
B. Phenytoin
C. Lamotrigen
D. Phenobar
Ans: C. Lamotrigen Ref: FSRH, drug interactions As a result of new evidence, CHC is not usually
recommended in women on lamotrigine monotherapy due to the risk of reduced seizure control
whilst on CHC, and the potential for toxicity in the CHC-free week.

51 | P a g e
75. A lady was admitted with severe OHSS. There is persistent oliguria despite adequate fluid
replacement. Next step?
A. More fluids
B. Paracentesis
C. Diuretics
D. ??
Ans: B. Paracentesis Ref:OHSS, GTG

76. A lady with PMS, not seeking fertility. From below, what is BEST 1st line option for her?
A. CBT
B. New generation COCs
C. Mirena
D. E patch
E. SSRI high dose
Ans: B. New generation COCs, since it asking for best 1st line

77. 20s age, HMB. First line option to her according to NICE?
A. COP
B. NSAIDs
C. Tranixamic acid
D. Mirena
E. Progesteron injection
Ans:Mirena

52 | P a g e
78. Same scenario as above but age was 30s, NICE was not referred to. [if it makes any difference]
A. COP
B. NSAIDs
C. Tranixamic acid
D. Mirena
E. Progesteron injection
Ans:Mirena…………………………………………………………………………………….

EMQ: A lady presents with oligomenorrhea. The most appropriate invest.


Ans C. DHEA/S
Options A. AMH B. 17(OH)P C. DHEA/S D. Synacthen test E. Dexamethason suppression test F. ??
Many others.
79. Age 20s, Also complains of progressive fatiguability & headache. US showed polycystic
ovaries.
Ans: ? Prolactin
80. Age 38. FSH & LH found high [20s for both]. Worried about her fertility.
Ans:AMH
……………………………………………………………………………… …………………
EMQ: Young girl, primary amenorrhea. Cause Options:
A. Turner
B. Swayer
C. Longitudinal septum
D. AIS
E. MRKH
F. Many others.
81. Well developed breasts & pubic hair. Short blind vagina. No uterus on US.
Ans: E. MRKH The vagina is absent or hypoplastic. The uterus is usually absent although there
may be a small non-communicating rudimentary remnant or anlagen, which may or may not contain
endometrium

53 | P a g e
82. Well developed breasts but scant pubic hair. Short blind vagina.
Ans: D. AIS Pubic and axillary hair are absent. The labia minora tend to be juvenile and the vagina
is short and blind ending. ………………………………………………………………………………
………………………………………………………………
EMQ: The most suitable advise for emergency CC: Options:
A. LNG 1500 mg stat
B. LNG 750 mg now & after 12 hrs
C. LNG 750 mg 4 doses 12 h apart
D. IUCD E. Mirena F. Mifepriston [dose?]
G. Misopristol [dose?]
H. UPA 30 mg stat
I. UPA 5 mg stat
J. Wait for her next period then discuss.
83. The lady is ? P2, using POP [?350mg NET pill]. LMP 1 wk ago. Missed last 2 pills. UPSI 2 days
ago. Never wants to get pregnant again.
Ans: D. IUCD
84. The lady was on IUCD for 3m. Last period 3 wks ago & was heavy with clots, threads unpalpable
since. Was on vacation for the last 3 wks & there was multiple UPSI, last one 3 or 4 days ago. US
loop missing.
Ans:H. UPA 30 mg stat Ref:Emergency contraceptipn: UPA can be used till upto 5 days.

85. The lady was on ? COP, can’t remember details but was straight forward.
………………………………………………………………………………
…………………………………………………..

54 | P a g e
STATISTICS/ETHICS/CG:
86. A group of doctors are auditing Decision-delivery interval [DDI] according to the hospital
protocols. Data were collected, results were announced on meetings and a member was dedicated
to compare against NICE & RCOG guidelines. Next audit is planned after 6 months. What is missing
in the audit cycle?
A. Criterion
B. Setting the standard
C. Spreading the results
D. ?
E. Closing the audit cycle
Ans:Implement changes, I think though its not mentioned

87. A meta analysis was carried out to review the effect of metformin in those with PCO, A
subanalysis was carried out on its effects in relation to BMI. The results were illustrated on forest
plot below [This is just a representative figure but at least the 3 diamonds were to the left]
Interpretation?
A. More effect on obese [BMI>30]
B. More effect on BMI<30
C. Equal effect
D. Neither effective in both groups
E. ?
Ans?? We need the plot, likely it would be B. More effect on BMI<30, since 3 diamonds were on left
as mentioned
88. A colleague has carried out a retrospective study on the outcome of severe preterm delivery &
argued that all should be delivered by CS: From above info, what is against his conclusion?
A. The study was not carried for the intention to treat
B. The results are not statistically significant
C. The results are incorrectly interpreted
D. The results/groups are not comparable
E. There was not a defined control
Ans: E. There was not a defined control

55 | P a g e
89. MRCOG part 2 exam is composed of a written & OSCE parts. What type of assessment is it?
Written OSCE
A. Formative Formative
B. Formative Summative
C. Summative Summative
D. Summative Formative
E. ?
Ans: C. Summative Summative Ref:StratOG
90. SBA: A 14 yrs old girl came for TOP. What essential part is needed in her management
A. Gillick compitence
B. Fraser guidelines
C. ???
Ans: A. Gillick competence Ref:FSRH, Contraception in young people Ref:Obtaining informed
consent from under 16s
The Gillick ruling in 1985 established a precedent for treating minors in the UK without their
parents' consent. The ruling stated that minors of any age who are able to understand what is
proposed and have "sufficient discretion to be able to make a wise choice in their best interests"
are competent to consent for medical treatment. The guidelines set out by Lord Fraser in his
judgement of the Gillick case are referred to as the Fraser Guidelines
91. An ST4 is doing an emergency CS in a lady with a previous scar. After he opened the abdomen
he noticed plenty of tortuous veins covering lower uterine segment. Most appropriate action? [? 22
wks scan: placenta anterior but not low].
A. Proceed in CS
B. Call consultant & wait
C. Call consultant; deliver baby through upper segment incision then wait
D. Ask for US to localize placenta then open the uterus below its lower edge,
E. Call radiologist to place ? femoral catheter then proceed.
…………
92. A patient is diagnosed with Ca Cx. Upon reviewing her records it was noticed that her last
smear result was severe dyskariosis. She said that she had moved house & was never contacted
after her last smear. E. Initiate root cause analysis

56 | P a g e
93. A hospital is questioning about a one year supply [fibronectin essay kits or something] which
was finished after only 6 months
A. Audit

94. ? There was a 3rd stem here. ………………………………………………………………………………


………………………………………………………….
. EMQ: Consent requirement: 95. Options:
A. Girl or woman written consent
B. Girl or woman verbal consent
C. Ask another colleague for opinion
D. Parents signed consent
E. Father’s written consent
F. Cannot proceed as consent requirement are not fulfilled
G. Ask next of kin
H. Ask Court/legal advise
I. ?? Others

96. Emergency CS was needed for fetal distress. Mother refusing, father insisting.
Ans: F. Cannot proceed as consent requirement are not fulfilled

97. A 14 yrs old lady is planned for ERPOC.


Ans: A. Girl or woman written consent

98. In the colposcopy clinic, the lady is waiting for pelvic [or vaginal] exam.
Ans: B. Girl or woman verbal consent(with Chepron)

………………………………………………………………………………
…………………………………………………………………

57 | P a g e
EMQ: The most suitable type of study Options:

ALL types of studies in the world were given 99. A group of doctors want to review the literature to
get information about the effectiveness of certain procedure [or drug] (Retrospective) Observation
study

100. A doctor has collected a 15 yrs data on the effect of carboplatin on 5 yrs survival Review of
literature ………………………………………………………………………………
………………………………………………………………………………
LABOUR:Ø
101. A scenario by the end of which they asked for the presenting diameter of face presentation.
A. Suboccipitofrontal
B. Suboccipitobregmatic
C. Submentobregmatic
D. Mentovertical
E. Biparietal

102. 2nd stage. Turtle sign. 1st step? [call for help not in the list].
A. Mcrobert
B. Suprapubic pressure
C. All four
D. Delivery of post.arm
E. ?

103. IUFD at 26 wks. Appropriate dose of Mifeprison follwed 24-36h by Misoprostol?


A. Mife 100 mg, Miso 100mg QDS total 5 doses
B. Mife 100 mg, Miso 100 mµ QDS total 4 doses
C. Mife 200µg, Miso 100 mg QDS total 5 doses
D. Mife 200mg, Miso 100µg QDS total 5 doses
E. Mife 200 mg, Miso 100 mg QDS total 5 doses

58 | P a g e
104. As a complication of oxytocin, what electrolyte imbalance?
A. Hypernatremia
B. Hyperkalemia
C. Hyponatremia
D. Hypokalemia
E. Hypocalcemia
105. What is the site of action of opiate analgesics [or where receptors are located]? *not sure
A. Brain:Basal ganglia. Cord: Grey mater
B. Brain: “?Para”hipoocompus. Cord: Grey mater
C. Brain: Periaqueductal grey mater. Cord: grey mater
D. Brain: Corpus callosum. Spine: ?
E. ?
Ans- C
106. [May be EMQ]. Class II NYHA, 2nd stage, head +2 DOA, urge to push. FHR normal
A. Cat.1 CS
B. Allow VD
C. Larg episiotomy & allow VD
D. Outlet forceps
E. Ventouse VD.
Ans- B
107.[May be EMQ]. Recently arrived from Latvia. Poor English. No ANC record available. Presented
with trickling V.bleeding [?≈ 100 ml]. O/E stable, [?FL= term, 4/5th palpable head]. Midline
abdominal scar. Abdomen is soft, fetal parts palpable & FHR normal. The most likely cause?
A. ?Show
B. Abruptio
C. Rupture/dehiscence
D. Placenta previa
E. Vasa previa Ans- D

59 | P a g e
………………………………………………………………………………
……………………………………………………………..
EMQ: Most appropriate action [Actually 2 emqs] Options:
A. Emergency CS
B. ARM
C. Start oxytocin
D. ARM + oxytocin
E. Offer intermittent auscultation
F. Offer CEFM
G. Offer amnioinfusion
H. Forceps in the room
I. Trial of forceps in theatre
J. Ventouse
K. PV after 2 hours [NO 4 hrs in the options]
L. FBS
108. Parous lady. 1 ½ h of active 2nd stage. Head +2 DOP, Actively pushing. Abnormal CTG.
Ans- I Parous lady having prolonged 2nd stage raises suspicion . I would go for trial of
instrumental delivery in ot as she has +2 station with malposition. The only instrument in ot given
is forceps.
109. Parous. Breech at 36 wks. ECV failed & opted for elective CS. Came in labour at 37 wks, PV
fully dialated, sacro anterior breach at +2, CTG normal.
Ans- H Allow for vd and use forceps For after coming head.
110. ? Parous low risk, term, came in labour : 4cm, membranes intact, FHR normal. Contractions
seem infrequent [2/10]. PV after 4h 7cm, ruptured membranes & FHR normal.
Ans- K??
111. As above but after 4h Cx was 5 cm & membranes still intact. FHR normal
Ans- B Here we can suspect delay in labor as after 4 hours only 1 cm progress.( atleast 2cm should
be there). Nice says- if delay in the established first stage of labour is suspected, amniotomy
should be considered for all women with intact membranes.
112. 2nd stage for ? hrs. Abnormal CTG, PV OT at spines, ++ caput/moulding.
Ans- A

60 | P a g e
113. Unevenful ANC. Came in labour. PV cephalic, 4cm, intact membranes, FHR normal.
Ans- E
114. Unevenful ANC. Came in labour. PV cephalic, 4cm, ruptured membranes, meconeum stained
liquor. FHR normal.
Ans- E This is not significant meconium . Hence can continue intermittent auscultation . Def: This is
defined as dark green or black amniotic fluid that is thick or tenacious, or any meconium stained
amniotic fluid containing lumps of meconium.
EMQ. PPROM. Most appropriate action Options:
A. Steroids
B. Erythromycin
C. I/V atosiban
D. Emergency CS
E. FBS
F. Forceps
G. Ventouse
H. Many others
115.GA 34 [?or 35] wks. 2nd stage. Head +1 ?DOA. Abnormal CTG [?prolonged decel]. The lady is a
known case of ITP & her Plt count is 70x 103 /ml.
Ans- D She needs her delivery to be expedited as ctg is abnormal. judicious use of FBS and
forceps – these should be performed by a senior obstetricia after balancing the small fetal risk
against the maternal risk of emergency caesarean section in advanced labour/second stage.

116.GA 28wks. Was admitted 2 wks earlier as threatened PTL & was given a course of steroids. Now
presented with PPROM. Speculum Cx closed with draining liquor.
Ans- B erythromycin for 10 days
117.GA 29 wks. Presented with tightinings. Cx 3 cm, intact membranes.
Ans- A
Ref : nice guidelines . offer tocolysis and steroids . But nice says offer nifedipine as first line
hence I didn't choose atosiban. Rescue cerclage only bet 16-28 weeks Hi everybody, here are the
next set

61 | P a g e
.EMQ. Post partum complications. Most appropriate action Options:
A. Return to theatre & re-open & do B-lynch
B. Reassure midwife & check after 30-60m
C. Insert balloon tamponade
D. Explore genital tract under good illuminating light
E. Explore genital tract in theatre [? Under anesthesia]
F. Return to theatre for MROP under aneasthesia
G. Give Oxytocin
H. Give ergometrin.
118.Difficult forceps. Placenta delivered completely. Given oxytocin. Uterus well contracted but still
active bleeding.
Ans- D
119.VBAC. Baby delivered. Placenta not delivered despite oxytocin. Catheter in. Now started to
bleed significantly.
Ans- F
120.Emergency CS after failed forceps. Bladder injury rapaired. No other intraop complications. 6
hours later you were called by the concerned midwife as she noticed continuous bleeding per
vagina. Uterus well contracted. Vitals normal. Urine clear & normal amount.
Ans- E
Failed forceps… ?cervical tear
…………………………………………………………………………………………………
Obs:
121. A woman with PH of Ca breast, recurrence free & just finished 5 yrs tamoxifen treatment.
Asking when she can conceive.
A. Immediately
B. After 6m
C. After 1yr
D. After 2 yrs
E. ? Advise against.
Ans-D

62 | P a g e
122.Pregnant lady on anti-tuberculus drugs. What is the most relevant monthly test?
A. CBC
B. LFT
C. UG
D. RFT
E. ?
Ans- B Tog- Liver function tests should be performed at least monthly
123.Preconception advice. Schizophrenic on Olanzipine. Drug effect on pregnancy? [More risk
for…]
A. Gestational diabetes
B. Anomalies
C. SGA
D. PTL
E. Pre-eclapsia
Ans- A
124.Pre conception advise. Post renal transplant. What drug to stop?
A. Azathioprin
B. Cyclosporin
C. Ramipiril
D. Prednisilon
E. LDA
Ans- C
125.Pregnant mother with HIV. On HAART. PPROM at 35 wks. VL 1 wk ago was undetectable.
Action?
A. Emergency CS
B. Immediate IOL
C. IOL after 24 hrs
D. Repeat VL
E. Steroids & conserv

63 | P a g e
Ans- B spontaneous pre-labour ROMs delivery should be expedited with induction if VL< 50
copies/mL If between Vl- 50 to 999- consider imm lscs If VL > 1000- recommend lscs

126.Known HIV. HAART started at 24 wks GA. When to stop after delivery?
A. Immediately
B. After 1 wk
C. Aftter 4 wks
D. ??other options.
Ans- ?a
127.Pregnant lady, developed MI. If possible, delay labour by….
A. 1-2 wks
B. 2-3 wks
C. 3-4 wks
D. 4-5 wks.
E. ?
Ans- B Tog- as the risk of maternal mortality is increased during this time.
128.Pregnant lady, developed MI. What is the most sensitive marker?
A. CK-MB
B. Troponin[?I or T]
C. Myoglobin
D. ?some peptide
E. ?
Ans- B TOG- troponin is never increased above the upper limit in normal pregnancy, not affected
by Anesthesia , prolonged labor, lscs and hence the investigation of choice.
129.What is the most relevant ECG finding in MI during pregnancy?
A. ST elevation
B. ST depression
C. T wave inversion
D. ?? Others

64 | P a g e
Ans- A Tog- the most sensitive and specific marker is ST elevation in ecg which normally appears
within minutes of onset of symptoms
130.Pregnant lady. Developed skin eruptionsEczematous rash. Most likely to be?
A. OC
B. Atopic eruptions
C. PEP
D. Pemphigoid gestationis
E. ?
Ans-B It's other name is eczema in pregnancy
131.Pregnant lady. Late 2nd TM. Nodules & papules in chest, trunk & extensor surfaces. Most likely
to be?
A. OC
B. Atopic eruptions
C. PEP
D. Pemphigoid gestationis
E. ?
Ans- B Atopic erruptions- incidence 1:300 , T2 or T3 onset, multiparous, papules in extensor
surfaces and chest, improves at delivery , no post partum exacerbation.

132.Pregnant lady, developed skin eruptions affecting abdominal striae. What feature is A/W good
prognosis?
A. Involvement of flexor surfaces
B. Involvement of extensor surfaces
C. Periumblicals sparing
D. ??
Ans- C This is polymorphic erruption of preg. Umbilical sparing is characteristic.

65 | P a g e
133.Pregnant woman. Developed Ca breast in 2nd TM [ductal carcinoma, ?stage]. Best drug?
A. Taxanes
B. Methotrexate
C. Anthracyclin
D. Cyclophosphamide
E. ?
Ans- c Gtg- anthracylines safe from second trimester.

134.Pregnant. Hyperthyroid on Carbimazole. Developed sore throat. What test is needed urgently?
A. CBC
B. Throat swab
C. Blood culture
D. ?Others
Ans- A To rule out agranulocytosis.
135.Asymptomatic bacteruria picked up at 24 wks GA. Plan?
A. Treat now
B. Treat now + IAP
C. IAP only
D. ??
Ans- A

136.Primi, 28 wks GA.US male, OHA, IUGR, REDF. What is the cause of OHA?
A. Posterior urethral valve
B. Renal hypoperfusion
C. Polycythemia
D. ??
Ans- A Puv obstn causes bilateral renal impairment . Oligohydramnios is also predictor of poor
outcome of renal fn. ‘key hole ‘ sign is classical.

66 | P a g e
137.A lady with major depressive disorder-with recent admission-is now controlled by fluxetin ?
40mg. Became pregnant. Worried about effect on baby. Advise?
A. Stop immediately
B. Stop & replace with TCA
C. Stop & resume after delivery
D. Explain low risk & that she can continue
E. ?
Ans – D
Nice- pregnant woman is taking a TCA, SSRI or (S)NRI for severe depression, take into account
previous response to treatment, stage of pregnancy, risk of relapse risk associated with
medication and her preference, and discuss with her the following options: -continuing with the
current medication -changing medication if there is a drug that is effective for her with a lower risk
of adverse effects -Cbt in combination……
138.Pregnant lady. 2 previos CS, both were emergency and considered traumatic for her. She is
asking for vaginal delivery. Plan?
A. ?Advise against
B. Refere to another consultant four further counseling
C. Counsel about risks
D. Allow VBAC but no IOL or augmentation
E. ?
Ans- C?
Was CBT an option?
139.A pregnant lady developed asthma [Vitals given]. What is the best predictor of respiratory
complications?
A. RR of 24
B. HR 100
C. PEFR of ?
D. ??
Ans- C?

67 | P a g e
140.Asthmatic.Post natal HTN. Best drug?
A. Labetalol
B. Nifedipine
C. Ramipiril
D. Methyle dopa
E. ?
Ans- D?(acc to spc) B(according to tog)
Spc With labetelol- breastfeeding is not recommended , also it's contraindicated in asthma
Methyldopa- anticipated benefits be weighed against possible risks.
nifedipine should not be used during breastfeeding.
Ramipril- not recommended in breast feeding
But tog (post partum htn) says
Safe in bf-labet, nifedipine, enalapril, captopril, aten, metaprolol

141.Β-Thal major. Post natal, on desferrioxamine. Mother worried about its effect on baby while
breast feeding:
A. Safe; reassure
B. Stop BF while on it
C. ???Others
Ans- A
As oral desferrioxamine not orally absorbed.

142.Post CS. Wt 110 kg. What prophylactic dose of Dalteparin for 7 days?
A. 2500
B. 5000
C. 7500
D. 10000 u
E. ?
Ans- C Ref gtg

68 | P a g e
143.What duration of post natal thromboprophylaxix: Known homozygus FVL. Her mother had
pregnancy related VTE.
A. 10 days
B. 2wks
C. 4 wks
D. 6 wks
E. ?
Ans- D
Gtg- Women with a family history of VTE and an identified thrombophilia should be considered for 6
weeks’ postnatal thromboprophylaxis.
………………………………………………………………………………
………………………………………………………….
EMQ: In each case; what is the risk of maternal mortality & the risk of fetus having congenital heart
disease Options: Maternal mortality: low: ≤1% significant:15-25% high: ≥50% Fetal risk of CHD
low:<5% significant: 15-25% high: 50%
A. Maternal mortality: Low Fetal risk: Low
B. Maternal mortality: Low Fetal risk: Significant
C. Maternal mortality: Low Fetal risk: High
D. Maternal mortality: Significant Fetal risk: Low
E. Maternal mortality: Significant Fetal risk: Significant
F. Maternal mortality: Significant Fetal risk: High
G. Maternal mortality: High Fetal risk: Low
H. Maternal mortality: High. Fetal risk: Significant
I. Maternal mortality: High. Fetal risk: High
144.The mother has radial hypoplasia + ASD. Diagnosed by the genitist as a case of Holt-Oram
syndrome.
Ans-? C
I know it's AD. So 50% chances of baby having it ( although it's not so simple with gene penetration
issues being around 75% for cardiac defects)

69 | P a g e
145.The mother is a case of a syndrome with single ventricle, repaired by fontan’ procedure. [? Can
climb stairs]
Ans-? G
I think this is high risk for mother . I'm not sure about fetus chd in mother has a chance of
recurrence in offspring 5-10%
146.The mother is a case of HOCM. Recent echo Mild heart enlargement + normal EF.
Ans-?C
Familial hocm is AD.
…………………………………………………………….
EMQ [?or may be 2 SBAs]: Thalassemia major: Management Options:
A. Arrange echo
B. Arrange MRI
C. LDA
D. LMWH
E. LDA + LMWH
F. Start desferrioxamine
G. ?
147. Splenectomised. GA ?22 wks. Plt count 610x 103 /ml.
Ans- E
Gtg- Women with thalassaemia who have undergone splenectomy and have a platelet count above
600 x 109/l should be offered low-molecular-weight heparin thromboprophylaxis as well as low-
dose aspirin (75 mg/day).
148.GA 22 wks. Her preconception liver iron was very [or significantly] high.
Ans- F
Gtg- If liver iron exceeds 15 mg/g (dw) prior to conception, the risk of myocardial iron loading
increases so iron chelation with low-dose desferrioxamine should be commenced between 20 and
28 weeks under guidance from the haemoglobinopathy team.
……………………………………………………………………
…………………………………………………………….

70 | P a g e
EMQ: Maternal collapse. Most appropriate action. Defibrillator ready on the scene. Options:
A. Basic life support with CPR 15:2
B. Basic life support with CPR 10:2
C. Stop rescussitation
D. One shock followed immediately by CPR for 2m
E. 3 consequative shocks
F. ?? Other options
149.The mother collapsed suddenly. ECG [?or by palpation] NO pulse.
Ans- ? Iv adrenaline 1mg

150.The mother collapsed. ECG VT. DC shock successful cardioversion. 4 minutes later she was
arrested again. ECG VF.
Ans- D
Ref gtg
EMQ: Symphasis pubis pain [diatheses or something] during pregnancy. Plan of delivery
Options:
A. Admit for IOL
B. IOL after 1 wk
C. IOL at 41[?or 40] wks
D. Offer PFMT & analgesia
E. Deliver by CS

151.The mother has sustained a vertebral crush injury in 2nd TM. GA now 38 wks & she is
complaining of intolerable [? site] pain despite being on regular PFMT & maximum analgesia. She
used to used crutches when getting out out of home but now using it inside home while using
wheel chair outside.
Ans : No IOL not indicated, I think ( it was there in option – I remember it exactly)
I wrote in exam - E. Deliver by CS
Ref: TOG - spinal cord injury

71 | P a g e
152. GA 37 wks. On max dose analgesia. Denied further PFMT. She is requesting IOL. (This scenario
was related to Pelvic girdle pain)
Ans : No IOL not indicated, I think ( it was there in option – I remember it exactly)
Ref: PIL - pelvic girdle pain

153.After delivery [? days], the mother is tearful & disinterested in baby.


Answer : D. Offer support Scenario was S/O baby’s blue, in exam

154.After delivery [?days], The mother is acting weird & experiencing auditory hallucinations but no
intentions [ideas] of self or baby harm. Same history with her mother
Ans : C. Advise meeting with psychotherapist - this Q s/o postpartum psychosis with family history.
-(Same history with her mother , was given in exam). I am not sure. Any correction welcome.

Ref : NICE

1.5.10 Refer to a secondary mental health service (preferably a specialist perinatal


mental health service) for assessment and treatment, all women who:
have or are suspected to have severe mental illness have any history of severe mental illness
(during pregnancy or the postnatal period or at any other time). Ensure that the woman's GP knows
about the referral. [new 2014]

1.5.11 If a woman has any past or present severe mental illness or there is a family
history of severe perinatal mental illness in a first-degree relative, be alert for possible symptoms of
postpartum psychosis in the first 2 weeks after childbirth. [new 2014]

72 | P a g e
155.ANC visit at 28 wks [?missing most previous ANC appointments]. The mother is in regular
follow-up with addiction services & stablised on methadone. Despite this she admitted continuing
heavy use of [? crack]heroin, amphitamin & cocaine. US normal. No idea. Even don’t remember
whether this exact Q was or what was the exact Q.

My Ans: C. Urgent referral to addiction service & inform community midwife

156. ANC visit at [?wks]. The lady is well known to the addiction service as chronic drug abuser &
stable on methadone. US was done but she escaped before meeting with the consultant.

Not sure for my answer B. Arrange meeting with addiction service & community midwife

EMQ. The most appropriate next step. The woman is Rh-D sensitized. Options:

A. Check titre after 2 wks


B. Check titre after 4 wks
C. Check fetal genotype using PCR on maternal urine cffDNA
D. Check fetal genotype using PCR on maternal serum cffDNA
E. Check fetal genotype using hybridazation technique on maternal serum cffDNA
F. carry out fetal dopplers using MCAPSV
G. Carry out fetal dopplers using uterine artery RI
H. Carry out fetal umbilical artery dopplers.

157.At booking, Anti-D titre is 1024. Partner is hetero for D antigen


Ans?: D. Check fetal genotype using PCR on maternal serum cffDNA Partner is heterozygous so
better to know fetus rhesus first.

158.At 22 wks GA, Titre is 12. Partner is homozygous.


Ans: B. Check titre after 4 wks

73 | P a g e
GTG :
Where clinically significant maternal red cell antibodies are detected, the paternal phenotype can be
ascertained by serology. However with the rhesus D (RhD) antigen specifically, in an
antigenpositive father, while a likely phenotype can be deduced, genotyping is required to
determine whether he is homozygous or heterozygous for the RHD gene. If the father is
homozygous for the red cell antigen then all pregnancies are potentially at risk.

6.8 Once detected how often should antibody levels be monitored during pregnancy?

Anti-D and anti-c levels should be measured every 4 weeks up to 28 weeks of gestation and then
every 2 weeks until delivery.
159.At ?13 wks. Titre is 0.2 & partner is homozygous.

Reassure, no need to monitor. Don’t remember whether it was there in option


? Monitor every 2 weeks

EMQ [or may be 2 separate EMQs]. Preconception counseling. What is the risk to the couple’s
offspring being affected by the condition.

Genetically normal means not affected nor a carrier.

Options:

ALL FIGURES IN THE WORLD + one option of “Offer genetic counselling”

Difficult to answer 160-165 as there are no options mentioned here.

Options in exams were really wiered.

160.Mother genetically normal. Father affected by CAH.

74 | P a g e
All children carrier ( but I think it was not there in options)

161.Mother genetically normal. Father affected by Duchene muscular atrophy.

X linked recessive No son affected, all daughters obligatory carrier but I don't remember, it was
there in options.

162.Mother & father are untested. The mother’s younger sister is diagnosed with CF.

Ans : ????
Our pt’s chance to be carrier 2:3 Partner considered normal unless mentioned Chance of baby
carrier / affected…. ?
163.Mother has β-Thal major. Father genetically normal. Asking about risk to their offspring of
having thallasemia disease.
Baby would be carrier but not affected as father is normal But in exam none of the options were fit
to answer according to fact and logic.
164.Mother has β-Thal major. Father has Hemoglobin E disease. Asking about risk to their offspring
of having thallasemia disease.

Ans : ???? - Refer to geneticist – it was there in options


HbE means… condition related to SCD Difficult to answer without proper options.
GTG : SCD
165.Mother has asymptomatic β-Thallasemia. Father has asymptomatic α-Thallasemia.

Refer to geneticist – it was there in options

75 | P a g e
166.A pregnant woman developed flu/corysal symptoms. One wk later she presented ?high fever &
cough. O/E: ↓ air entery, crackles, signs of consolidations. [?sputum color?] What is the most likely
causative organism?
A. Strept.pneumonia
B. H1N1
C. ?Pseudomonas
D. ??other organisms.
Ans- A There is secondary bacterial infection after flu. (Ref: 2013: Bacterial and viral infections
associated with influenza. Influenza and Other Respiratory Viruses) Streptococcus pneumoniae
continues to be the dominant pathogen involved in this synergistic process followed mainly by
Staphylococcus aureus and Haemophilus influenzae.
167.What is the commonest cause of sepsis in pregnancy?
A. GBS
B. Strept.Pyogenes
C. E-coli
D. Pseudomonas
E. ?
Ans- B Ref :gtg
168. Elderly lady, C/O vulval pruritis. O/E fissuring + resorbtion of labia. The 1st line drug?
A. Topical Lidocaine
B. Clobetason cream
C. ?Podophyllin cream
D. ??
Ans- B This is lichen sclerosis.
169. Age?. Co/ pruritc lesion. Biopsy VIN reaching margin. Next step?
A. Re-excision
B. Colposcopic guided biopsy
C. Vulvectomy
D. ??Other options

76 | P a g e
Ans- B Ref: bassh All patients with VIN should be referred for up-to date colposcopy to
exclude CIN and VIN. Multiple biopsies may be required as there is a risk of missing invasive
disease
………………………………………………………………………………
………………………………………………………………..
EMQ: Most appropriate next step Options:
A. Reassure
B. Excision
C. Take biopsy
D. Offer steroids
E. Vulvectomy
F. Prescribe antifungal
G. Refere to dermatologist
170.Young lady who noticed small [8mm] pigmented papule on Rt. labium majus. No symptoms.
Ans- C
Ref: vulval ca guidelines Any change in the vulval epithelium in a postmenopausal woman warrants
a biopsy. These changes include: a swelling, polyp or lump, an ulcer, colour change (whitening or
pigment deposition), elevation or irregularity of the surface contour) premenopausal women all
other vulval signs and symptoms should be managed as for those in postmenopausal woman
unless there is a confirmed infection. Lesions should be biopsied rather than excised.
171. Young lady presented with a scaly lesion on her vulva. O/E there was a similar lesion on the
scalp.
Ans- D
…………………
EMQ. What likely finding to be expected on exam Options:
A. Strawberry Cx
B. Acanthosis nigricans,
C. Lichenifaction
D. Inflammed Cx with discharge
E. [?Tender] nodule on posterior fornix F. ???Many others 172. 50 yrs old who complains of itchy
vulva + fissuring & superficial dysparunia which didn’t improve with lubricants.
Ans- c This is lichen sclerosis

77 | P a g e
173.A young woman with long history of pelvic pain, painful periods & deep dysparunia [? PH of
PID or something].
Ans- E Endometriosis
………………………………………………………………………………
EMQ [?or SBA]. What is the most likely diagnosis. Options:
A. TV
B. Chlamydia Trachomatis
C. VIN D. LS
E. Behcet disease
F. Syphilis
G. H.ducryi
H. Chronic infection with low infectivity
I. Chronic infection with high infectivity
174.Young lady. PH of CT infection. Now C/O conjunctivitis, lower abd pain & discharge.
Ans- B Causes pid, pneumonia and conjunctivitis
175. HBV serology came as follows: HBsAg +ve, DNA +ve & E-Ag +ve.
Ans- I

Hepatitis B. When e antigen present indicates high infectivity


………………………………………………………………………………
………………………………………………………………..
EMQ. Most appropriate next step of action Options:
A. Serum tests now & repeat after [? duration]
B. Reassure
C. Refer for hospital assessment
D. Offer IVIG
E. Offer Acyclovir
F. Offer serial scans
G. Expectant mng

78 | P a g e
H. Immediate induction
I. IOL after 24h J. ??
176. GA ?16 wks. Got rash + arthralgia. Worried about rubella. Repeat test showed her being
immune to rubella but evidence of parvovirus infection.
Ans- F To rule out non immune hydrops- weekly usg up to 12 weeks after maternal exposure
177.GA 26 wks. Got rash + arthralgia. Tests showed rubella infection.
Ans- B After 20 weeks no increase in risk of adverse fetal outcomes. <11 weeks- 90% 11-16 weeks-
20% 16-20 weeks- min risk of deafness >20 weeks - no risk
178.GP asking for adviseGA 24 wks. The woman had contact with chickenpox & developed rash 2
days ago.
Ans- ?G As 48 hours have passed since rash developed
Gtg- oral aciclovir for pregnant women with chickenpox if they present within 24 hours of the
onset of the rash and if they are more than 20 weeks of gestation.
179.PROM on top of recurrent HSV at 39 wks GA.
Ans- I In recurrent hsv manage as normal women.
………………………………………………………………………………
……………………………………………………………
EMQ. ?Superficial dysparunia. Most Suitable plan of mng Options:
A. Offer referral to sexual psychotherapist
B. Offer dialators
C. Offer local lidocaine 1%
D. Offer PFMT
E. Give advise about local hygene
F. Reassure
180.2 months after NVD, the woman recently resumed sex. She complains of superficial dysparunia.
O/E there is NO area of localized tenderness but there is some pelvic floor spasm.
Ans- ?B
181.Young woman complains of ?superficial dusparunia + tingling sensation & pain on vulva. No
abnormality on exam
Ans- ? C
Warm regards , Tanushree

79 | P a g e
GYNECOLOGY SBA
1. 82 years old lady who suffers from urgency, frequency and stress urinary incontinence. She
has medical history of osteomalacia and chronic constipation. She has tried physiotherapy and
bladder retraining but was ineffective. On examination; she has moderate vaginal atrophy.
Which medication you will offer her:

-Immediate release Oxybutinin


-Transdermal Oxybutinin
-Vaginal Estrogen
-Trospium Hydrochloride
-Duloxetine

2. In cases of lichen sclerosis, What percentage of patients will not respond to treatment with
topical steroids:
-15-22%
-25-30%
-35-40%

3. During evacuation of retained products of conception, what is the most common site of uterine
perforation
-Anterior uterine wall
-Posterior uterine wall
-Right lateral uterine wall
-Left Lateral uterine wall
-Uterine fundus

4. A 45 years old lady who had under gone a hysterectomy for abnormal uterine bleeding and
dysmenorrhea. On histopathological examination was found to have Adenomyosis. What would
be the most common co-existing uterine finding:

-Endometrial cancer
-Endometrial polyp
-Uterine fibroid
-Endometrial hyperplasia

80 | P a g e
5. In postmenopausal women what is the percentage of endometrial cancer in those who present
with abnormal uterine bleeding
-5%
-10%
-15%
-20%
-25%
6. A lady presents with missed miscarriage at 7 weeks gestation. She opted for surgical
management. What is the most common complication she is at risk of:

-Cervical trauma
-Infection
-Blood transfusion
-Uterine perforation

7. A postmenopausal woman presented with slow onset of increasing facial acne and hirsutism
that started initially 6 months back. On examination: she has male type baldness. What is the
most likely diagnosis:

-Androgen secreting tumour


-Ovarian Hyperthicosis
-Adrenal tumour
-Polycystic ovaries

8. A couple presents to the infertility clinic for advice. The husband on investigation was found to
have low FSH, LH and testosterone levels. What is the most likely cause:
-Kalmann syndrome
-Anabolic steroids
-Kleinfilter Syndrome
-Cystic fibrosis

9. A lady presents with vulvar irritation. On examination: wou found an annular lesion with scaling
on the vulva with a similar one on the scalp. What is your management:
-Refer to dermatology
-Clobetasol
-Immunomodulators

10. A lady presents with vulval irritation. On examination you found a small lesion <1cm in diameter
on the labia majora. What is your management
-Excisional biopsy
-Clobetasol
81 | P a g e
-Flurouracil
-Vulvectomy

11. A lady presents with vulval irritation. On examination you found a 1cm growth on the clitoral
hood. What is your management
-Excisional biopsy
-vulvectomy
-Edge biopsy
-biopsy from the center

12. A couple presented to the infertility clinic for advice. The lady has a normal hormonal profile
with ovulatory cycles. HSG showed normal patent tubes bilaterally. The husband had semen
analysis of 0.1 million sperm/ml, abnormal forms, small testicles with high FSH and LH. Which
treatment option you will offer the

-IUI with donor sperm


-IVF with donor sperm
-IVF with husband sperm
-Surgical sperm retrieval
-ICSI

13. A lady had hysteroscopic tubal sterilization she is currently on combined oral contraceptives.
You advise her that the procedure will be effective:
-After her first menstrual cycle
-After finishing the current packs of COCPs
-After having HSG confirmation after 3 months

14. An Epileptic drug whose efficacy is reduced if used concomitantly with COCPs
-Phenytoin
-Valproic acid
-Lamotrigine

15. A 17 years old girl presented with primary amenorrhea. She has well developed breast and
sparse pubic and axillary hair. On examination was found to have a blind ending vagina.
-MRKH
-CAH
-Androgen secreting tumour
-Turner Syndrome
-Androgen insensitivity

82 | P a g e
16. A 40 years old women, came to see you with her husband who mentioned that she lately
developed acne and hoarseness of voice. She has grade 2-4 hirsutism. Testosetrone is 11, 17-
Ketosteroids are normal. FSH, LH are normal. What is the most likely cause:
-Androgen secreting tumour
-Adrenal tumour
-Cushings syndrome
-PCOD

17. Patient had ovulation induction for IVF on day 9 she developed severe OHSS and was admitted
to the hospital. She remained oliguric despite adequate fluid maintenance. What is your
management:
-Expectant
-Diuretics
-Paracentesis

18. A 35 years old lady with irregular periods has seen the GP who did some blood tests for her:
FSH is 35 iu/l and LH is 45 iu/l. What investigation you will do for her first:
-AMH
-Repeat FSH and LH on day 2-3 of her periods
-HSG

19. Patient presents with history of difficulty emptying the bladder. She has history of multiple
sclerosis. On investigations was found to have high residual volume. What is your
management:
-Tamsulosin
-Botulinium injection
-Suprapubic catheterization
-CISC

20. Patient presents with persistent urge incontinence after failed conservative management of
bladder retraining and decreasing fluid and caffeine intake. What is your management:
-Desmopressin
-Imipramine
-Trospium Hydrochloride
-Venlafaxine

21. A 34 years old Aerobics instructor, complaining of stress incontinence. She has tried
physiotherapy and pelvic floor exercise but no benefit. You advise her for surgical
management, but she refused. She asks you if there is any medication available for her case.
What is your management:
83 | P a g e
-Duloxetine
-Imipramine
-Desmopressin
-Oxybutinin
-you tell her that there are no medications for her condition

22. You review a patient in the gynaecology ward. She had TVT with postop 48hrs catheterization.
After removal of the catheter she had urinary retention. How long you would re-catheterize her
for:
-48 hrs with clamping
-24 hrs free drain
-Catheterization with antibiotic cover

23. Middle age Patient presents to the Gyn clinic with vulval irritation and progressively worsening
dysparaunea even with using lubricants. It started initially 6 months back. On examination
Labia are flushed. What is the most likely diagnosis:
-Lichen sclerosis
-Lichen planus
-VIN

24. Patient presents to the gyn clinic with multiple small painless erythematous red fleshy lesions
on the vulva. What is the most likely diagnosis:
-VIN
-Vulval Cancer
-HPV infection
-HSV infection

25. Patient presents with irregular bleeding. An endometrial biopsy was collected. The pathology
report showed: Irregular glands, packed together back to back but not invading the stroma.
What is the most likely diagnosis:
-Complex endometrial hyperplasia
-Simple endometrial hyperplasia
-Endometrial Adenocarcinoma

26. A patient presents in early pregnancy at 6 weeks of gestation. Pelvic scan showed a small fetal
pole with no fetal heart. GS is 28 x 25 x 22 mm in size. What is your management:
-Repeat scan in 7-10 days
-Counsel Re-TOP
-Medical management

84 | P a g e
27. Patient presents with pap smear that shows low grade changes. You did HPV triage testing and
it was negative. What is your management:
-Colposcopy
-Routine recall in 3-5 years
-LLETZ

28. Patient is 58 years old. She is on HRT. She is scheduled for TAH and BSO in 7 days. What
would be your plan for postoperative thromboprophylaxis:
-LMWH x 4 days
-LMWH x 7 days
-LMWH only during her inpatient stay
-TEDS + mobilization

29. This is a study that was done to compare the effectiveness of Metformin and clomid in
subfertile patients with PCOD. A sub analysis was done according to the patient weight. The
results show:

-Metformin is equally as effective as clomifene in the obese group (BMI > 30)
-Metformin andclomifeneare equally affective in the non-obese group (BMI <30)
-Metformin is less effective than clomifene in the non-obese group (BMI >30)
-Metformin is more effective than clomifene in the non-obese group (BMI < 30)
-The statistical difference is insignificant

30. The risk of infraumbilical adhesions after a midline laparotomy is:


-20%

85 | P a g e
-30%
-40%
-50%
-60%

31. What is the initial management in a 25 years old patient with HMB:
-COCPs
-Tranexamic acid
-Mirena

32. What is the management for a 47 years old patient with DUB with normal investigations:
-Mirena
-COCPS
-NET
-Progesterone injections

33. Patient who is postmenopausal was started on continuous combined HRT 3 months back. She
presented with aacomplaint of abnormal uterine bleeding. What is your management:
-USS
-Change type of HRT
-stop HRT
-Increase the dose of HRT

34. A 38 year old woman who smokes 30 cigarettes a day comes for six week
postnatal checkup after delivery of her third child. She is keen not to become
pregnant again but does not wish to be sterilized. She does not wish to gain weight. What is the
single most appropriate prescription treatment.
-Medroxyprogesterone acetate(Depot Provera)
-Mirena
-IUCD
-POPS
-Postcoital oral contraception

86 | P a g e
35. The pathology shown in this HSG is most commonly associated with which anomaly:

-Pelvic kidney
-Double ureter
-renal agenesis
-ectopic ureter

36. A patient was diagnosed with PMS after having symptom diary. What is the initial management:
-Psychotherapy
-COCPS
-High dose SSRI

37. A 29 years old lady attends the Gyn clinic 6 months post difficult vaginal delivery. She is
complaining of vulval pain and dysparaunea. On examination the perineum looks normal with
vaginismus. What is the appropriate initial management:
-Lidocaine ointment to the perineum
-Refer to physiotherapy
-Refer for Psychosexual counseling
-Amitriptyline

38. Misoprostol for medical management of miscarriage has the fastest onset of action and longest
duration of action if administred through which route:
-buccal
-sublingual
-vaginal
-rectal
-oral

39. Risk of vaginal prolapse in women is:


-20%
-30%
-50%

87 | P a g e
-60%

40. Patient comes to see you in the clinic who is 14 years old and wants to TOP. You will proceed
according to what:
-Fraser competence
-Gillick competence
-Mental capacity
-Parental consent

41. A couple married for 3 years with subfertility. On investigations all normal for both of them.
What is the management that you will offer:
-IVF
-ICSI
-IUI
-Clomid

42. A 45 years old lady with history of HMB for which she is receiving medical management. In
which of the following conditions ultrasound scan will be done as an initial investigation:
-Prior to insertion of IUS
-Heavy bleeding causing anemia
-Failed medical treatment

43. You are doing Gyn clinic and a patient came to see you who had a cervical mass of 2 cm in size.
She complains that 1 year ago she had a pap smear that showed sever dyskariosis and no one
informed her. She mentioned that she has shifted her house during that time. What is the action
that you will take:
-Clinical incident report
-Governance patient safety report
-Inform CMO
-Inform GMC

44. 41 years old patient known to have PCOD, her BMI is 35. She has family history of heart attack.
According to her risk factors, what is the most important risk factor that puts her at risk of
metabolic syndrome:
-Her age
-Her BMI
-Her family history
-PCOS

45. A patient with estrogen positive breast cancer she has completed 5 years of Tamoxinfen. For
how long you would advise her to wait before she gets pregnant:
88 | P a g e
- 6months
- 1 year
- 2 year
-immediately

GYNECOLOGY EMQS
What is most possible diagnosis in each of the following cases:

Options:
-Stress urinary incontinence
-Mixed urinary incontinence
-Overactive bladder
-Overactive bladder + stress incontinence
-Diabetic bladder
-Urge incontinence
-Vesicovaginal fistula

46. 58 years old woman suffering from urinary incontinence, urgency and dribbling of urine. Her
urine culture is negative. Her UDS are as follows:
Bladder capacity: 200 ml
Residual volume: 25 ml
Detruser pressure during the procedure: 30
flow rate : 30 mls/sec
first desire to void at 90 ml
Incontinence on provocative tests

47. 58 years old woman suffering from urinary incontinence, urgency and frequency. Her urine
culture is negative. Her UDS are as follows:
Bladder capacity: 400 ml
Residual volume: 50 ml
Detruser pressure during the procedure stable throughout
flow rate : 10 mls/sec
Had continuous leakage from the vagina throughout the procedure

48. 58 years old woman suffering from urinary incontinence and urgency. Her urine culture is
negative. Her UDS are as follows:
Bladder capacity: 300 ml
Residual volume: 50 ml
Detruser pressure during the procedure: 30
flow rate : 50 mls/sec

89 | P a g e
first desire to void at 90 ml
no incontinence was demonstrated

What is most possible diagnosis in each of the following cases:

Options:
-MRKH
-CAH
-Androgen secreting tumour
-Turner Syndrome
-Sweyer syndrome
-Androgen insenstivity

49. A 19 years old girl with primary amenorrhea, normal secondary sexual characters. She is
sexually active. On examination found to have a blind ending vagina.

50. A 16 years old girl, presenting with primary amenorrhea. She has grade 2-4 hirsutism, acne and
small non developed breast. She refused vaginal examination.

A postmenopausal woman presents with a single episode of vaginal spotting. On USS was found to
have ET of 1.3 mm, with incidental diagnosis of left ovarian cyst of 4.5 cm in size.
What is the most appropriate management in each of these cases:

Options:
-TAH + BSO
-TAH +BSO + Omentectomy + Peritoneal cytology + lymphadenectomy
-Left Oophorectomy
-Bilateral oophoprectoomy
-Ovarian cystectomy
-Cyst aspiration
-Discharge with no Followup
-USS in 4 months
51. The cyst is noted on the scan to have regular smooth surface with clear fluid content. CA125 is
10.

52. The cyst is multilocular with solid component <1cm . CA125 is 30.

90 | P a g e
53. The cyst is simple, and it was noted that she had a scan 1 year back where that cyst with same
measurements was noted. CA125 is 30.

What is the most relevant cause of this patient’s symptoms?

Options:
-Complicated Chlamydia infection
-Gonorrhea
-Trichomonasvaginalis
-Herpes simplex

54. Patient in a monogamous relationship for 4 years. 2 years back she had Chlamydial infection
that was treated but she did not follow up in GUM clinic. Now she presents with vaginal
discharge, abdominal pain and mild dysuria. She also generalized joint pain, arthritis and
conjunctivitis.

A patient presents to the early pregnancy unit with left iliac fossa pain. Pregnancy test is positive.
What is the most appropriate management in each of these cases:

Options:
-Admit for observation
-Discharge with no followup
-Booking scan at 12 weeks
-Do BHCG now and after 48 hrs
-Laparoscopy
-Serum progesterone now and in 48 hrs

55. On USS a viable foetus is seen with small subchorionic bleed. And a 3 cm haemorrhagic left
ovarian cyst was noted too.

56. No IUGS is seen on USS.

57. IUGS is seen on USS, with a 3 cm left ovarian mass and moderate free fluid in POD.

What is the most appropriate management in each of these cases:

91 | P a g e
Options:
-Await menstruation/ check BHCG
-IUCD
-Mirena
-Levonelle
-UPA
58. A woman had an IUCD inserted 3 months back. She had her periods 3 weeks ago, it was heavy
with clots. Then she went on a vacation for 2 weeks where she had regular sexual intercourse.
Last SI was 4 days back. She came to see you as she could not feel the threads. And she is
requesting emergency contraception. On USS there was no IUCD.

59. A lady had unprotected SI 3 days back. She has an abnormal shaped uterus. Requesting
emergency contraception.

60. A lady had USI 3 days back. She is 35 years old, wants contraception that she can retain for a
long time.

What is the most likely cause in each of these cases:

Options:
-Bowel obstruction
-Intrabdominalhaematoma
-Vaginal haematoma
-Infected vault haematoma
-UTI
-Chest infection
-Narcotic misuse
-PE
-Mismanagement of IV fluids
61. You went to review the patient 6 hrs post-operative who is postmenopausal , 60 years old.You
found her with low respiratory rate, otherwise all was normal.

62. A patient post TAH on day 2 who is a cigarette smoker is having pyrexia, feeling unwell. Her
observations are normal and abdomen is soft. She is obese with BMI of 39.

63. Day 1 post TAH, she is having oliguria. Urine output < 30 ml/hr. Abdomen soft, wound with
signs of bruises but no bleeding or signs of infection. All observations are normal.

92 | P a g e
What is the most appropriate management in each of these cases:

Options:
-Continue same management

64. 24 hrspost-operative the patient had a spike of temperature 37.8 and was started on antibiotics.
All her observations are normal and she is stable since then.

OBSTETRICS SBA
65. A pregnant lady at 14 weeks gestation , she got Parvovirus infection. What is the risk of mother
to fetus transmission intrauterine at this gestation:
-5%
-10%
-15%
-25%
-35%

66. The risk of urinary incontinence after Elective cesarean section is decreased in comparison to
normal vaginal delivery to what percentage:
-4%
-10%
-8%
-14%

67. A pregnant woman at 24 weeks gestation known to have pulmonary TB on Isoniazid, Rifampicin
and Ethambutol. Which investigation you will do monthly:
-Liver function test
-Urea electrolytes
-Drug levels
-CRP
-Full blood count

93 | P a g e
68. A patient who is known to have hyperthyroidism on Carbimazole. She presents at 26 weeks
gestation with a complaint of sore throat. Which investigation you will do urgently:
-FBC
-CRP
-Throat swab
-Thyroid function tests

69. You received a call from the GP asking for your opinion as he has a patient who presented at 24
weeks of gestation with a 2 day history of chicken pox rash who came asking for advise. What
would you advise him to do:
-Refer her to hospital
-Offer acyclovir
-Reassure her
-Topical emollients
-Intravenous IG

70. Woman in the second trimester of pregnancy, presenting with erythematous rash over her face,
neck, chest and extensor surface of arms. What is the most likely diagnosis:
-PEP
-Atopic eruptions of pregnancy
-Prurigo
-… (no infectious causes were in the choices)

71. A lady with MCDA twins presents antenatally. At what gestational age you should start
screening for TTTS
-14 weeks
-16 weeks
-18 weeks
-20 weeks
-24 weeks

72. Patient comes to see you in the clinic. She is 3 months post Forceps delivery with 2 nd degree
vaginal tear. She is complaining of flatus incontinence and fecal incontinence too especially
when her stools are soft. What is the investigation that you will do:
-Manometry and endoanal scan
-Pelvic floor exercise

73. Regarding the anatomy of the Inferior Epigastric artery. From which branch it originates?
-External iliac artery
-Internal Iliac artery

94 | P a g e
-Femoral artery

74. Patient presents with IUFD at 26 weeks gestation. What is your management:
-Mifepristone 200 milligrams followed after 36-48 hrs with Misoprostol 100 micrograms x 6
doses.
-Mifepristone 100 milligrams followed after 36-48 hrs with Misoprostol 25-50 micrograms x 6
doses.
-Mifepristone 200 micrograms followed after 36-48 hrs with Misoprostol 100 micrograms x 6
doses.
-Mifepristone 200 micrograms followed after 36-48 hrs with Misoprostol 25-50 micrograms x 6
doses.
-Mifepristone 100 micrograms followed after 12-24 hrs with Misoprostol 25-50 micrograms x 6
doses.

75. Patient at 13+4 weeks gestation. She is 41 years old and wants screening for Down syndrome.
What is the most appropriate test at this gestation:
-Combined test ( NT + Pappa A + BHCG)
-Quadrible test + Triple test
-CVS
-Amniocentesis
-Gestational age not suitable for screen now

76. Patient weighing 120 Kgs, has just delivered by emergency cesarean section. What is the dose
of Deltaparin that you will prescribe for her:
-5000 IU
-7500 IU
-10,000 IU

77. Patient is seen by you in the antenatal clinic in the second trimester. She has history of
admission to the psychiatric unit before pregnancy with major psychiatric disorder. She is on
Fluxetine 40 mg OD for depression. She is worried about its effect on the baby and is asking
you about what she should do. Your advise will be:
-Stop medication
-Reassure her
-Decrease the dose to 20 mg Daily
-Change to TCAs
-Stop the medication and start CBT

78. The most sensitive feature of acute myocardial infarction on ECG is:
-ST segment elevation
-ST segment Depression
95 | P a g e
-Prolonged Q-T interval

79. Patient with h/o PPROM , with temperature of 35.7, BP 80/50 . What is your initial investigation:
-CBC and CRP
-Blood culture+ Lactic acid
-HVS

80. Threshold level of lactic acid for diagnosis of sepsis is:


-1 mmol/l
-2 mmol/l
-3 mmol/l
-4 mmol/l
-5 mmol/l

81. Patient at 30 years old, who is p1 had normal vaginal delivery 6 hrs back. She has family history
of thrombophilia. When she was investigated was found to be factor V leiden homozygous.
What is your management:
-LMWH x 7days
-LMWH x 6 months
-TEDS + mobilization

82. In your hospital it was decided to use FFN for all patients who present with risk of preterm
labour. After 6 months only you realized that the stock that was ordered has all finished. You
want to know whether you have underestimated the number of patients that present to your
hospital or what. What is your management:
-Clinical effectiveness
-Arrange meeting with unit managers
-Run an audit
-Make a research

83. The RCOG exam has two componenets written and OSCE, Which choice best describes each:

Written OSCE
Formative Summative
Formative Formative
Summative Formative
Summative Summative

84. This is a study that was done in a hospital where the researcher recommends that according to
his findings all preterm babies should be delivered by CS as it improves the survival rate.

96 | P a g e
CS SVD P value
Number 150 450
GA 26+5 26+5
Survival high low 0.003

Why is that wrong:


-The numbers on each arm are not even
-The P-value is not significant
-It was not done with intention to treat
-wrong statistical analysis

85. This is a Patient 2 years post renal transplant. She is stable. Came to see you for
preconceptional counseling. She is taking the following medications, which one you would
advise her to stop prior to pregnancy:
-Enalapril
-Labetalol
-chloroquine

86. A 36 years old, obese patient with BMI of 38 at 35 weeks gestation. She has frequent attacks of
headache with diplopia and eye pain. CT was done which was negative. Which treatment you
will offer:
-Acetazolamide
-Propranolol

87. A patient with high BP post-delivery. She is asthmatic. Which medication you will prescribe to
her:
-Labetalol
-Nifedipine

88. Patient with HIV. Viral load is undetectable. She was on HAART antenatally. When should you
stop HAART postnatally:
-immediately
-4 weeks
-6 weeks
-6 months
-12 months

89. Patient who has sepsis postnatally. What is the most common causative organism:
-E-coli
-GBS
-Pseudomonas

97 | P a g e
-Streptococcus Agalactae

90. A pregnant patient who recently arrived to the UK, mentioned to you that she had genital cutting
as a child. On examination: there is no clitoris, no labia minora, labia majora are sutured
together in the midline was small opening admitting 1 finger at the fourchette. How do you
classify that:
-Grade 1 FGM
-Grade 2 FGM
-Grade 3 FGM
-Grade 4 FGM
-Female circumcision

91. A registrar is performing an elective CS,on opening the abdomen he found large vessels
covering the lower uterine segment. At 19 wks. USS showed Placenta anterior low lying but not
previa. what will be your management:
-ask interventional radiologist to come and pass femoral line then proceed
-stop and wait for consultant to come
-give incision above the upper edge ofvessels and deliver the baby until the consultant arrives
-ask for USS to check placental location than proceed

92. Patient who had slow progress of labour. She just had the head of the baby delivered 2 mins
ago with +ve turtle sign. After call for help and the emergency bell what will be your
management:
-McRoberts
-Suprapubic pressure
-All Fours
-Internal maneuvers

93. Risk of congenital heart block in SLE where Anti Ro and Anti La are positive:
-2%
-5%
-10%

94. Recurrence risk of acute fatty liver of pregnancy is:


-5%
-10%
-25%
-35%
-45%

98 | P a g e
95. Risk of post dural puncture headache is:
-0.1-0.4 %
-0.5-2.5%
-3-4.5%

96. Opiates receptors in the CNS are located in:


-Basal ganglia + Spinal cord white matter
-Hippocumpalgyrus + Spinal cord grey matter
-Preaquiductal area of the brain

97. You performed an Audit. you set the criteria, collected the data, made an acyion plan, trained
the team and did a re-audit after 6 months. What is the remaining step:
-Setting the standard
-Closing the loop
-Implementation of the results
-Sharing the results

98. Patient developed rashes over her body at 26 weeks gestation. Which of the following criteria of
the rash would be reassuring:
-Sparing the umbilicus
-If abdominal striae are present
-If rashes are on the trunk and the extensor surfaces

99. Patient had normal delivery with multiple tears: parauretheral , labial and a tear involving 50% of
the EAS but intact mucosa and IAS. Which grade you will classify her tear:
-2nd degree
-3a 3 rd degree
-3b 3 rd degree
-3c 3 rd degree
-4 th degree

100. Patient is having a psychiatric problem on Olanzapine. Which side effect is more common
with this medication:
-Hypocalcemia
-Hypoglycemia
-Congenital anomalies
-Cardiac anomalies

101. The most common side effect of excessive use of oxytocin is:
-Hypernatremia
-Hypoglycemia
99 | P a g e
-Hypokalemia
-Hypomagnesaemia

102. During the 10 weeks scan the fetus was noted to have omphalocele with midline defect.
What is the possible cause:
-Physiological gut herniation
-Edward Syndrome
-Patau syndrome

103. A pregnant patient in the first trimester. CRL is 90 on the scan. Which parameter would be
most accurate for dating:
-HC
-BPD
-AC
-CRL

104. In case Face presentation what is the presenting diameter:


-Submentopregmatic
-suboccipitopregmatic
-Mentoverical
-mentobregmatic

105. Pregnant lady with baby weight <10% and oligohydramnios, male fetus ,what is the most
likey cause:
- Uteroplacental insufficiency
- Posterior uretheral valve
- Decreased renal perfusion

OBSTETRICS EMQS
What is your initial management in each of the following Scenarios?
Options:
-Ergometrine
-Oxytocin IV 5 IU
-Oxytocin infusion
-Hysterectomy
-Bakri balloon
-Brace suture
-Uterine artery embolization
100 | P a g e
-Manual removal of placenta in theatre under anaethesia
-Examination in the room with good light

106. Patient had Cesarean section. During the procedure she sustained bladder injury that the
urologist came in and repaired intraoperatively. After the procedure while the patient was still in
the recovery with Syntocinon infusion running, you were called by the midwife that was worried
as the patient is heavily bleeding. The dressing is dry with no ooze from the wound and the
uterus is contracted. Her preoperative Hb was 11.8 g/dl. Now he Hb is 7.4 g/dl.

107. Patient with previous Cesarean section had a successful vaginal delivery. However placenta
was not separated despite active management of third stage. The patient is heavily bleeding.

108. Para 4 patient, post normal vaginal delivery. She had completed active management of third
stage with small second degree tear that was already sutured by the midwife. You were called
as the patient while still in the labour room started bleeding.

What is your management in each of the following cases:

Options:
-Amniocentesis for detection of infection
-Serial ultrasound scans
-Counsel for termination of pregnancy
-Reassure

109. Patient presents at 26 weeks gestation with rash. On blood investigations you found that
she has got Rubella infection.

110. A school teacher presented with a rash after contact with a boy at school who had a similar
rash that was thought initially to be Rubella. However on serology was found to be negative for
rubella but infected with Parvo virus.

What is your management in each of the following cases:

Options:
-Admit for induction of labour
-IOL at 37 weeks
-IOL at 40 weeks

101 | P a g e
-IOL within next week
-Cesaarean section
111. Patient presented at 38 weeks, she complains that she can’t walk due to severe pain at the
pubic symphysis that she has been taking codeine and paracetamol for. She is demanding
delivery.

112. Patient presented at 39 weeks of gestation, she had history of spinal injury but was fine
before pregnancy. By 31 weeks gestation she started using crutches at home too for
mobilization later she had to buy a wheel chair to use at home for movement. Now she even
uses the wheelchair to go out as can’t walk anymore. She is demanding delivery.

Choose the most appropriate option:

Options:
-Late indirect maternal death
-Indirect maternal death
-direct maternal death
-Coincidental death
-Early indirect maternal death
-Sudden death
-Does not fulfill maternal mortality criteria

113. A lady was murdered by her husband at 22 weeks gestation.

114. A lady who was admitted at 14 weeks gestation and had surgical management for missed
miscarriage. 2 days later she died and was discovered to have cardiac disease with
Eisenmenger syndrome.

115. A lady who had IOL for preeclampsia. On day 4 postnatal her BP was 222/115. She
developed intracranial haemorrhage and died at 5 weeks postnatal.

Choose the most relevant vessel in these cases:

Options:
-obturator artery
-pudendal artery
-deep circumflex iliac artery
-Superficial circumflex iliac artery

102 | P a g e
-Middle rectal artery
-Superior gluteal artery
-Inferior gluteal artery
-External iliac artery

116. During doing sacrospinous fixation for a patient with vault prolapse she started bleeding
heavily during tying a knot near the pyriformis muscle. Which vessel is most likely injured?

117. Patient had a forceps delivery after which she developed a heamatoma on the right vaginal
wall. You took her to the OR for evacuation and found that blood was coming from a deep
vessel. Which vessel is most likely injured?

118. Patient who sustained a 4th degree tear during delivery had severe bleeding from the anal
area. What is the main vessel from which the injured vessel has arisen?

What is your management in each of the following cases:


Options:
-Inform community midwife
-Arrange community midwife visit
-Another ANC appt in 1 week
- Another ANC appt in 4 weeks
-Inform police
-Inform safeguarding services

119. Patient who is known to take Cocaine and other elicit drugs. She was started on Methadone
OD, but she is not complaint with the medications. She is known to the social services. She is
also having reports of domestic abuse. She had a scheduled antenatal clinic follow up
appointment with you today but she did not show up.

120. Patient who is known to take Cocaine. She was started on Methadone. She is taking her
medication regularly. She had a scheduled antenatal clinic follow up appointment with you
today but she only did her ultrasound scan and left without seeing you. The community midwife
has already been informed.

What is your management in each of the following cases:

103 | P a g e
Options:
-Inform the police
-Can’t proceed as consent requirements not complete
-Defer the procedure
-Take husband consent
-Get another opinion
-Respect patient’s rights
-Proceed with the procedure
-Take parental consent

121. 15 years old boy came to see you in the clinic with his girlfriend from school that is 15 years
old too. She is pregnant and requesting TOP, and she does not want to inform her family.

122. A patient in labour seems to have a sound mind. She is having a pathological CTG, you
advised her for emergency cesarean section, but after understanding all the risks she refused
to have the procedure done. However, the husband is asking you to proceed with CS.

What is your management in each of the following cases:


Options:
-Take biopsy from the mass
-Abandon the procedure and arrange appointment for discussion
-Can’t proceed as consent requirements not complete
-Defer the procedure
-Take husband consent
-Take consent from next of kin
-Get another opinion and proceed
-Proceed with the procedure
-Remove structure (x) from structure (y)
-Remove structure (y)

123. Patient at 38 years old is consented for Laparoscopic hysterectomy with ovarian
conservation +/- laparotomy +/- BSO for Abnormal uterine bleeding. During the surgery you
found that the left ovary (y) has a cyst (x) and is adherent to the lateral pelvic wall.

124. A Patient, who is 28 years old with a positive HCG and empty uterus on scan with a 3 cm
right adnexal mass, was consented for laparoscopy + salpingectomy. During the procedure you
found both tubes normal and the right ovary (y) has a 3 cm mass attached to it (x).

104 | P a g e
125. You were called by you surgeon colleague who was doing a laparoscopic appendectomy on
a patient who presented with pelvic pain for suspected appendicitis. During the procedure he
found that the appendix was normal so he deferred the procedure. But he also found a 3 cm
hemorrhagic ovarian cyst (x) attached to the right ovary (y) so he called you in for opinion.

What is your initial management in each of the following cases:


Options:
-Cardiac MRI
-Desferrioxamine chelation therapy
-LMWH
-LMWH + LDA
-LDA
-Deferasirox chelation
126. Pregnant patient in second trimester with thalassemia major, did not come for
preconceptualcunselling. On liver ferriscan was found to have high iron overload.

127. Pregnant patient with thalassemia major who had history of splenectomy. Her platelet count
is 600x 10#3.

What is the most likely diagnosis in each of the following cases:


Options:
-Anaphylactic shock
-Pulmonary embolism
-VTE
128. A bus driver 42 years old, at 35 weeks gestation. She is para 4. At the end of her shift she
collapsed. Resuscitation was attempted but she died after 30 mins. She also had severe peanut
allergy.

What is your management in each of the following cases:


Options:
-Allow home delivery
-Allow VBAC but no IOL or augmentation
-IOL
-CS
129. Para 1 patient. Low risk. In her first pregnancy she did not have any problems antenatally.
During the dlivery she had epidural and rotational forceps delivery. It was a very traumatic

105 | P a g e
experience for her. She wants her delivery this time without intervention at all.

130. Patient with pervious 2 x CS. The first was for Breech and the second as for low lying
placenta at 35 weeks gestation. In the index pregnancy, placenta is not low lying. And so far
pregnancy has been uneventful. She wants to have vaginal delivery.

What is the most appropriate management in each of the following cases:


Options:
-Refer to psychiatry unit urgent
-Admit to psychiatric ward
-Admit to maternity ward
-Start antipsychotoc medications
-Arrange an urgent psychological assessment
-Reassurance, family support
-Isolation from the baby
131. Patient brought by her husband 4 days post-delivery she is tearful, but no harm ideations.

132. Patient is brought by her husband post-delivery; she is anxious and mentioned that she is
hearing voices talking to her. She has no suicidal ideations or harm thoughts.

What is the most appropriate management in each of the following cases:


Options:
-Repeat titre 4 weeks later
-Refer to FMU for MCA Doppler
-Measure cell free F DNA from maternal plasma using PCR
-Measure cell free F DNA from maternal plasma using DNA hybridization
-Umbilical artery Doppler
-Ductusvenosusdoppler
133. Patient presents at 26 weeks gestation with anti D level of 1013 IU/l .Her husband is
Heterozygous for RH-D.

134. Patient presents at 16 weeks gestation with Anti D level of 0.4 IU/L. Husband is homozygous
for RH-D.

What is your management in each of the following cases:

Options:
-Immediate augmentation of labour
106 | P a g e
-Acyclovir
-CS
-Await SVD but no intervention as FSE or FBS
-Augmentation after 24 hrs if not in labour till then
-Augmentation after 48 hrs if not in labour till then
135. Patient presents with recurrent episode of herpes simplex at term with prelabour rupture of
membranes.

136. Patient with HIV who was on HAART natenatally. Her viral load is <50 copies/ml. She came
at term with prelabour rupture of membranes.

What is your management in each of the following cases:

Options:
-Intapartum antibiotics
-Treat now + intrapartum Abs
-Treat now + screen at 37 weeks
-Screen at 37 weeks
-Routine care
-Erythromycin
137. Patient with history of GBS in her previous pregnancy. Baby was not affected. Came asking
about the management in this pregnancy.

138. Patient with GBS in urine antenatally.

139. Patient with PPROM unknown GBS status

Your are counseling a couple in the preconceptional clinic Regarding the risk of thalassemia to the
offspring:

Options:
-50% affected
-25% affected
-50% carrier
-25% affected
-Refer to genetic counseling
-CVS

107 | P a g e
-Amniocentesis

140. A lady who is alpha thalassemia carrier, married a man who is beta thalassemia carrier.

141. A lady who is beta thalassemia major, married a man who is HbE

142. A lady who is beta thalassemia major, married a man who is alpha thalassemia carrier.

Your are counseling a couple in the preconceptional clinic Regarding the risk of to the offspring:

Options:
-50% affected
-25% affected
-50% carrier
-25% affected
-None affected
-all females are carriers
-all females are affected
-all males are carriers
-all males are affected
-(many choices)
143. A woman who has a sister affected with cystic fibrosis is married to a man who is
unaffected.

144. A woman who is known to be screen free is married to a man with Duchenne muscular
dystrophy.

145. Congenital adrenal hyperplasia…

What is your initial management in the following scenarios:

Options:
-CRP 100/min
-Synchronized cardioversion shock
-3 consecutive shocks
-Adrenaline 1 mg
-Adrenaline 5 mg
-CPR 30/2 breaths /min
-CPR 15/2 breaths / min
-Oxygen mask

108 | P a g e
146. A pregnant patient was found pulseless on the floor in the antenatal ward.

147. A pregnant patient that collapsed, attended by the emergency team, intubated and the
monitor showed ventricular tachycardia, a shock was given after which the patient recovered. 2
minutes later she collapsed again. She was still intubated and connected to the monitor which
should ventricular fibrillation this time.

What is your management in the following scenarios:

Options:
-reassess in 30 mins
-CS category 1
-CS category 2
-Rotational forceps
-Rotational ventouse
-Forceps delivery in labour room
-Forceps delivery in OR
-await spontaneous labour
148. A primigravida, fully dilated with head just below the spines. Per abdomen the head is 0/5 th
palpable. CTG is non reassuring. FBS was done with a result of 7.23.

149. Patient is in labour with pathological CTG. Per abdomen head is 2/5 th palpable. She is gully
dilated and head is in ROT position.

150. Patient is in labour, on vaginal examination was found to be 6 cm dilated with cord
prolapse. CTG is normal.

What is the estimated risk to both the mother and the baby in the following scenarios:

Options:
Mother Baby
High (25%) High (25%)
High (25%) Moderate (15%)
High (25%) Low (5-10%)
Moderate (15%) High (25%)
Moderate (15%) Moderate (15%)
Moderate (15%) Low (5-10%)
Low (5-10%) High (25%)
Low(5-10%) Moderate (15%)

109 | P a g e
Low(5-10%) Low (5-10%)

151. A patient who has a single atrium and a single ventricle. She had Fontan operation a child.
She is now pregnant.

152. A patient with ASD, asymptomatic. Was found to have a genetic syndrome.

153. Patient with cardiomyopathy. Had cardiac echo done which showed good stroke volume.

What is your management in the following scenarios:

Options:
-Offer resuturing
-Conservative management
-Augmentin 625 mg x 7 days
-Flucloxacillin + metronidazole 400 mg BID x 14 days
154. Patient came for followup post SVD with 2nd degree tear. She is complaining that her
strutures broke down, she is 4 days postnatal. On examination, the wound is completely
gabbed, with clean base, and creamy vaginal discharge.

110 | P a g e
,

GYNE
statistics: [some of them came in obs] • EMQ :What type of test should be used: Options: Almost
all types of tests were given Questions: 1. We want to find the relationship between Um.A doppler &
pre-eclamsia, after other factors like age, parity, BMI are taken into consideration. 2. To find the
relationship between maternal BMI & fetal birth wt. 3. To look for the different outcomes of
pregnancy in relation to BMI. • Name of the study: 4. To measure the outcome of breech delivery.
Women were assigned into 2 groups: vaginal & c/s ; & followed up. Options:

oncology: • EMQ: Counselling about 5 rys survival rate Options : Different figures Qeustions:
(May be changed next exams) 5. Stage 2b cx 6. 1a ovary (i think it was specifically mucinous). 7. 1b
endometrium • SBA: 8. An old woman with ?1cm mass in clitoris. What should be the enxt step to
reac ha diagnosis? Options included exisional biopsy/incisional from centre surface/incisional
from margin/wide local exision + LND 9. A woman undergone evac of delayed miscarrige.
Histopath. --> Complete mole. What is the risk of developing chorio carcinoma? Options: 1:20,
1:40 , 1:60 , 1:80 , 1:100. [15% was NOT there] 10. what is the % of bilaterality of dermoid.
Options: I think started with 10%, then bigger figures; 15% up to 40%. 11. Management option after
treatment[LLETZ] for CIN 3 with biopsy showing unclear margins? [? test of cure + smear after 6 m]
12. Management option after treatment[LLETZ], age 52, biopsy showed CIN1+cGIN reaching
excitional margins.
Options: cone biopsy, TAH, Radical hysterectomy,& others i can’t remember.

Ethics/clinical goverannce • EMQ: Options were many. We recalled these: Woman wtitten consent
Woman to sign consent & assessing that she understands what she's consenting to Girl or
woman verbal consent Can’t proceed as the consent requirements not complete Encourage girl
to tell parents (Gillick/frasers’s were not in the options) Questions: 13. Emergency c/s, mother
refuse to sign consent. Father insist for c/s 14. A 15 yr girl came with boy friend (15 yrs) for TOP
[GA ?17 wks] & they look aware/understanding. 15. A 14 yrs girl came for ERPOC. • EMQ: Options:
Too many. Unfortunately can’t recall most of them. Questions: 16. I think a patient died & the
coroner request the pt medical records. Option list included full access, restricted access & ?deny
access were in the option list. 17. A woman who underwent TVT 3 yrs ago now have symptoms
recurred & she asks for her medical records. [full or restricted access to patient records??] 18. A
woman diagnosed with severe diskaryosis, colposcopy recommended. She moved house & no
lettre reached her to come back to follow up ( can't remember options). I think later on she came
with cancer. 19. A doctor carrying a study has took the study papers home [papers with private

111 | P a g e
patient info]. The study papers were stolen. (what should the hospital do?) There were options like
“report never event”, “report to information handling commitee” 20. A study carried out to compare
suture material used &seniority of surgeon in suturing 3rd 4th degree perineum tear & compare it
against rcog guideline. • EMQ: Unexpected pathology during surgery. Options: Abandon
procedure. Take biopsy from X & abandon procedure Remove X from from Y Otehr options like
-i think- Remove X, Remove X &Y, Contact enxt of kin........ 21. A young girl who was entered as
acute appendicitis. Appendix was found normal but a dermoid cyst[X] was found on Rt ovary[Y].
Can’t remember size . 22. A 40 yrs woman during TAH FOR HMB was found to have a dermoid
cyst[X] on Rt ovary[Y], both attached to pelvic side wall by adhesions . • EMQ 23. A woman caem at
41 wks GA. You offered her IOL but she denied. [?give her patient inforamtion leaflet]

firtility/andrology 24. Couples, 1ry infirtility for 30 months i think, all tests normal. This Q came
as SBA & EMQ
Options: CC, IVF, advise to try naturally for another 6 m,....... 25. 26. Male previously fertile. Now
azoospermic. Low FSH & testosteron options : klienfilter, anabolic steroids, Kallman’s 27. A
woman with infertility, workup showed blooked tubes with hydrosalpings. [?Salpingectomy
followed by IVF. Also came as SBA & as a part of an EMQ]. 28. 29. From the followin figures, what
hormonal profile is consistent with WHO class 3 ov.disorders? [High FSH, high LH , low E, normal
PRL] 30. A 19-year-old woman was seen in the gynaecology clinic with a history of excessive
growth of facial hair, needing to wax every 2-3 weeks. Her menstrual periods last 3-4 days every 3-4
months. There is no change in her voice. Her BMI is 28 kg/m2. Examination shows Ferriman-
Gallwey grade 2-3 hirsutism over chest and abdomen. A pelvic ultrasound showed no abnormality.
Her day two hormone tests showed LH level 7.4IU/L, FSH level 5.2IU/L, serum testosterone level
2.3nmol/l, SHBG 24 nmol/L. What is the most likely diagnosis? PCO, Idiopathic, Cushing,
Androgen producing tumour

*This is a repeat question, but in the previous exam they put a cycle pattern of 7-8 days every 24-35
days. ( idiopathic). 31. A woman came with hirsutism + virilization. Testesteron 7. No DHEA OR
OTHER INVEST were given, but mentioned to have central obesity [?cushing] 32. A woamn came
with hirsutism, irregular periods, LH was higher than FSH. Testosteron was ? 6.5 . They asked
about next test to help reach a diagnosis. [Options included U/S , TFT, DHEA, 17(OH)P, ? dexameth.
Supression test ] • EMQ: Choose the most likely diagnosis. Options: can’t remember well but
include OHSS. (or manage OHSS according to unit protocol) Ectopic / Heterotropic [each as a
seperate option] Acute appendicitis Tortion/ Ruptured cyst [each as a seperate option] Bowel
perforation Laparotomylaparoscopy (?or lap for detortion)/, Expectant mng,..... 33. Surrogate
woman, came from spain [or somewhere] after replacing 2 embryos , came withe severe LIF pain. I
think Ultrasound showed single gestational sac [?hetero] 34. A woman 2 days after egg
collection(larg number collected,? 25), came with severe LIF pain, vomiting, lightheadedness. U/S
showed ?“spider web”. [? OHSS/ some said Torsion] 35. Iam not sure but i think there was
another acute presentation, with U/S showing ?daughnut sign or something • EMQ or SBA[not

112 | P a g e
sure] : Male factor infirtility: investigations? 36. A scenario was given where semen analysis was
normal [lower normal], the only abnoramlity mentioned was dysuria or turbid urine after coitus.
37. Another scenario, the man has fatiguability & errection problems.(i think the options included
something about “electric wave analysis fo eaculate”. Deeply sorry for the poor recall.) 38. A third
scenario, i think with oligospermia. He has mild gyencomastia.

NERVE /vascular .INJ • EMQ: Identify the most likely injured nerve 39. After forceps, woman came
later with urinary & fecal inc [?pudendal neuropathy] 40. After VD, woman can't flex knee or extend
hip+ absent knee jerk [? Femoral] 41. After surgery in lithotomy, a woman has foot drop & sensory
loss in lateral surface of leg. [Common peroneal wasn't in the options, but Lumbar-peroneal
neuropathy was there.] • SBA 42. During TVT, the surgeon accedently hit the inferior epigastric
artery ( &NOT OBTURATOR). From wich artery does it arise? [ext. iliac ]

urodynamics: • EMQ: Coose the appropriate Mng: Can't remember much but options inclded:
PFMT Bladder retraing PFMT & Bladder retraing Bladder diary Clean intermittent self cath.
Many options including specific drugs & surgeries. I don’t remember if urodynamics were there or
not. 43. A woman presents with urgency, freq, nocturia & STRES Inc. [?diary] 44. Another woman
who i think her symptoms were pure stress UI. [PFMT] 45. A woman with multiple sclerosis +
voiding diff [?CISC] • EMQ: urodynamic interpretation: what is the likely diagnosis Options
included DO, USI, Interstitial cystis, &-i think- chronic cystitis 46. Residual 70. 1st & max desire
both reduced. Normal voiding velocity. I think pressure rise during void was ? Normal. 47. There
was another Q. Can’t remember • SBA 48. woman with recurrent UTI + Pain : cystoscopy: -->
multiple small hemorrhagic areas Options: Interstitial cystitis, transitional cell ca, superficial
cystitis or something. 49. Of women with OAB, what % will also have urge incontinence?

Infections
• EMQ: What is the diagnosis Options: Primary syphilis, 2ndry syphilis, Chancroid, Vulval
candidiasis, Lichen planus/sclerosis, Vulval cancer, Melanoma, others. 50. A young lady, came from
trip to china. C/O vulval lesion. O/E : 2 symmetrical painless ulcers with raised edges on labia
majora. [we argued about chancroid-kissing ulcers. But painless?] 51. An elderly 82 yrs woman at
nursery home c/o blood staining of underwares. NO SYMPTOMS. O/E small pigmented lesion with
rolled edge on Lt labium majus, with small satellite lesions around. [?candidiasis,? cancer] 52. A
young lady came from somewhere, c/o mass, o/e: sessile growth on perineum , painless, no itching.
[wart] Others 53. Young lady with PMS mainly psychlogical (& her partner prosecutig her for
assaulting him). Her GP advised exercise or something. Options included all steps of managing
PMS, & also referal to social worker 54. what is the % of U perforation after evac (?of missed ab?) [?
5:1000] 55. what is the absolute C/I to UAE from the following? This Q came twice. & the only
option came twice was adenomyosis....HINT OR DISTRACTOR? Other options wrere age below 35 ,
IUCD in situ, active pelvic infection, Asymptomatic fibroid. [Pregnany was NOT there]. 56. A

113 | P a g e
woman on stable relationship for past 23 years & on IUCD for last 12 years now she had abd.
Pain.U/S showed a mass beside right ovary. [?Actinomycetoma?, can’t remember the other
options] 57. What is the 1 st line managment of menorrhagia? This Q came twice. The difference
was in age &-in the first Q, they specifically referred to NICE. [We didn’t think of a difference: both
IUS?] 58. A long history but at the end: ovarian tumour + pathology report of Call hexener bodies.
[granulosa cell tumour] • EMQ: What sign do you expect to find. Options: bilteral adnexal masses
Longitudenal vagianl septum. Retroverted uterus Thickinings in POD [?or tuero sacral lig]
Uretrhal caurencle Other optiosn i can’t remember 59. A young girl with recurrent UTI. Now also
has dsyparunia [Most said urethral caruncle. Why not long.septum?] 60. A PCO woman taking
clomifine . [?adnexal masses] 61. One with a diagnosis of endometriosis [? R/V uterus, thickinings]
• SBA
62. A 36 yrs old lady with previous term healthy baby, now had 3 recurrent miscarriages at 6,7,8
wks. What invest. are you going to do? Options : APLa, thrombopillia screen, pelvic U/S for
anatomy, other 2 options i can’t remember but did’t include karyo as far as i remember. 63. A
woman with PH of two 2nd TM miscarriages, presented at 18 wks,no contractions but o/e you find
membranes bulging into vagina [? Admission + cierclage] 64. EMQ: sexaully active, C/O frothy-
greenish V.discahrge [?TV] 65. EMQ: sexaully active, C/O malodorous fishy discahrge. [?Gardnella]
66. What is the recurrence rate of bacterial vaginosis? 67. There was a question, where an obese
diabetic lady on metformin undergoen TAH for HMB. After....hrs post surgery, you were called by
the conerend nurse. The patient’s MOEWS chart was given. We were asked about the underlying
pathology. Options icnluded sepsis, DKA, Hypovolemia, Hypoxia. 68. There was a question about a
woman who did an HSG, can’t remember details but i think came later with features of PID. There
was an option of “Manage PID according to protocol” . 69. An 18 yrs girl came with 1ry A. Normal
2ndry sexaul development. Very shrot vagina{?the ygave the length}. Ovaries presetn on scan.
[?MRKH] 70. A 16 yrs girl came with 1ry A. Heavy excersise. BMI 18. FSH 34. [Options included: Wt
related A, primary POF, wait for another 2 yrs] 71. What is the emergency CC option for an epileptic
lady who denied IUCD? [?levoenlle 3g] 72. The concentration/efficacy of the following drug is
reduced by taking COCs Options included the common AEDs [?lamotrigine] 73. What is the
commoenst type of ureteric inj in laparoscopic surgery? [?transection] • EMQ : Antibiotics
prophylaxis choice: ALL CASES HAS SOME SORT OF CARDIAC DISEASE Options included many
endocarditis prophylaxix, Erythromycin/doxycyclin pre & post surgery. NO cefuroxime or
Augmentin as single doses , but there was cefazolin single dose 74. TAH + Ant[?or may be post]
repair. 75. Diagnostic Hysteroscopy + cystoscopy. 76. There was a question about a lady who came
5 wks post IVF conception with mild bleeding. U/S showed intrauterine empty GS of 21 mm. I thik
they asked about the diagnosis ratehr than next step.

114 | P a g e
OBS
• EMQ: for the following scenarios, what is the risk of maternal morbidity/ mortality (low/significant/
high) & the risk of the baby having cong.heart disease(<10%, 15-25% , 50 %: )
77. mother with minor anomaly, with previous baby with CHD.
78. mother with pul.HTN
79. mother with PH of PPCM. In this pregnancy the echo shows structurally normal heart with mild
systolic dysfunction. •

EMQ: PPH: Options were many but included: Bimanual compress Carpoprost Ergometrin[one
option was I/M, other was slow I/V] Synto [one option was bolus, other as infusion] Explore
lower egnital tract under good light Explore in theatre. + many otehr options
80. Heavy bleeding, HD compromise, placenta complete, catheter in. Rescusitate WAS NOT AN
OPTION.
81. Moderate bleeding after delivering 4.1 kg baby .placenta complete. Uterus well contracted.
82. Heavy bleeding. Placenta complete.catheter+ bimanual +oxytocin already given. Patient has
mild asthma. •
EMQ: what is the single most imp. Paremeter you want to know: Options were BP, CBC, RBG,Coag
profile, GCS, O2 sat.
83. You are about to undertake hysterectomy for massive I.O bleeding. The anesthetic team are
managing to keep stable BP.
84. After delivery, a lady developed a grand mal fit. 85. After c/s , pt became hypotensive. No
v.bleeding but ooze from wond. • EMQ: Most propable underlying cause: Options inclded:
anaphylaxix, Amnitic fluid embolism, Air& Fat embolism, Maligannt hypertehrmia, tension pneumo
thorax. 86. During c/s, pt developed generalized rash, hypotension. 87. During c/s , pt became short
of breath, hypotensive, low O2 sat( & also CO2) • EMQ: What is underlying cause: 88. before
emergency c/s, pt complained of headache + visual disturb. BP ?160/110 i think. Smooth section &
viable fetus but pt didn't awake from anesthesia & died. Autopsy: IVH + conation of ? Cerebellar
vermis. In the options, there was "acute hydrocephalus due to herniation of medulla" or something
like that. 89. An obese bus driver suddenly collapsed & died • EMQ: 2nd stage & heart dis. The
classification given was not NYHA ,but that mentioned in the FSRH GL (CC & cardiac dis). Options:
Aim for VD, give oxytocin, deliver with forceps, deliver with ventose, C.section 90. Class 2,
symptomless, in 2nd stage but pushing well, head +3. 91. 2nd woman was class 3 ,in 2nd stage,
head + 2. I think she has some symptoms. DERMA:
92. Woman with purpuric rash. Cant' remember GA. rash disappeared but woman still itchy. LFT:(
normal bilirubin, normal AST, high ALP , slightly low albumin[32 instead of 35]. What further
testing? Options: serum bile acids, virology screen, caog.profile, repeat LFT, no further test

115 | P a g e
needed. 93. Mother with purpuric rash involving abd straie. What feature is a/w good prognosis?
(?Sparing umblicus,....) 94. Mother with purpuric rash. Biopsy showed immune complex deposition.
What’s the likely diagnossis? Options: they gave the D/D of purpuric rash in pregnancy.
INFECTIONS: 95. A pregnant lady developed chickenbox rash. Phoned on same day. [? Acyclovir].
96. A pregnant lady developed chickenbox rash, phoned 3 days later . [? avoid contact with preg
ladies OR no further actions? OR U/S after 5 wks?]. 97. A pregnant lady came with rash + joint pain.
RETESTING( that how it was written) showd her infected with parvo- not ruballa. GA was 2nd TM .
What action? [? U/S follow up] 98. There was a question about congenital malaria. A woman has
malaria in pregnancy. What should you do? Options after delivery: 1)Blood film for baby at birth
then wkly for 4 wks 2) plasental films; if neg for malaria, no further action. Labour • EMQ: IPC
Options: Re-assess after 15m/30m/1h/2h/4h [each as a seperate option], Forceps delivery, Ventouse
[2 cup types given]delivery, C/S, Reassess in theatre for forceps or c/s,... 99. PG,effective epidural,
good progress, 1h passive 2nd stage & then actively pushing for 90m. head at spines & CTG
noraml. 100. Parous lady, in 2nd stage for 90m, now pushing for 30m. Head ? OL at spines. 101. A
woman after 20 m in 2nd stage tells you taht she can’t push more & demadns c/s. She allows you
to examine her. Head? OP at spines. Headache 102. What is the most common finding in CVT?
[? headache] 103. Preconeption counselling of pt with migrane. The optiosn were combiantions of
↑, ↓,unchanged migrane attacks & ↓, ↑ or uncahnged risk of pre-eclampsia. [PE ↑. Attacks
?unchanged] 104. A question about headache where the lady described it as the worst ever.
[?SAH]. 105. Headache developing post partum + focal neurological signs. MRI showed filling
defect [?CVT, others said Posterior reversible encephalopathy syndrome]

OTHERS: 106. what is the chance of survival without disability for a baby born at 24 wks. 107. what
is the most abnormal karyo to a/w trunkus arteriosus? [Options: trisomy 13 /21/ deletion of q22,
others ].
108. Mother with type 1 DM came with PPROM. What is the more common cause of death for this
baby [ prematurity or lung hypoplasia?]. 109. in what country is obstetric cholestasis most
common? [?Cheli] 110. A woman with PH of recurrent DVT, now on thromboprophylaxis with
Rivaroxaban - when to stop pre op? 24 hrs , 5 days , 7 days , 4 weeks . After we did some search,
it appeared to be an oral heparin, given daily. Needs to be stopped 24 hrs prior to surgery. This was
a web search. Please check again 111. What is the length of the presenting diaemter in Face
presentation [9.5 cm] 112. A midwife ot sure abotu presentation called you. You palpate malar
eminence + ? alveolar margin. [?Face] 113. A pregnant women with breast lump. 1st invest will
be: (options were u/s- mamograme- chest x ray). 114. A woman found to have asymptomatic GBS
bacteruria. Management plan? [?treat now and IPAP in labour]. 115. Waht is the rate of spont.
Reversion to breech after ECV in PG [? 5%] 116. A woman & her partenr both are CF carriers. What
is the cahcne of an affected baby? [1:4] 117. From the following, what is NOT a risk factor for
twins? Options: advanced age, PH of monozigotic twins, IVF, others. 118. What is the chance of
success of Mc’robert with suprapubic pressrue to effect delivery? [? 90%] 119. From the following,
what is a major risk factor for SGA? [can’t remember options] 120. What is the best measure of GA
at 84 mm diameter ?GS. [ options: CRL, HC, BPD]. • EMQ: HSV:can’t remember options 121. A
woman came in labour, tells you that her partenr was recently treated for HSV. 122. A woman came
116 | P a g e
in labour. Was treated 2 wks ago for recurrent HSV. 123. A woman came in labour, was treated ? 2
wks ago for 1ry HSV. After counselling; she’s willing for VD. • EMQ : Management Options:
Augment with oxytocin, IPAP, Erythromycin, I/M steroids, Expectant.Mng,.... 124. GA 22wks, Known
GBS carrier, now PPROM ,? Features of ch.amnitis 125. GA 28 wks. Threatened PTL. Speculum cx
3 cm with intact membranes. Already given full dose I/M steroids 2 wks ago. 126. What of the
following conditions has more association with amle fetus Options included AFLP, obstetric
cholestasis,....... • EMQ : The causative organism. (GAS, GBS, HSV, CT, Staph,... ) 127. A neoante
developd eye infection(stick eyes) after 1 wk from delivery. [?CT] 128. A mother who had a vulval
soreness, gave birth to a neonate who latter developed signs of sepsis & he had vesicles on his
back. [?HSV] 129. What is the anti-D level above which there should be referral for fetal medicine
unit? [was it “above which” or “at which”?]
130. Waht drug you choose for an asthamtic lady with post partum HTN? [? Nifidipine. There was
an ACEi in the options but not enalapril/captopril] 131. What is the definition of perinatal mortality
rate? [? After 24wks GA, up to 7days/1000 or 10.000?] 132. There was an EMQ with 3 scenarios of
APH. 133. There was a question about the mode of delivery in HIV mother on HAART whose last
VL was 150 [?? The 2011 NICE c/s guideline says don’t offer c/s if VL <400. 2014 BHIVA said
“consider” c/s].

MRCOGTRADEMARK Groups
Sept 2014 MRCOG 2 Recalls

1. Pregnant_ previous abortion without the knowledge of her patner_ found to be Rh isoimmunized
about 10u _parner is Rh -ve _ following ur tactic couselling she is still worried. Anwer: fetal Rh
group by cffDNA using PCR. 2. 18wk pregnant _partner hetrozygous_18wks_about1100 u/l?
Anwer:cffDNA using PCR. 3. CTG picture and cenario: pegnant 32_34wks? Admitted 12_16hr ago
after spontaneous ROM cervical os 3cm . CTG: sudden reduction of the fetal HR to less than
90bpm which contiue through the rest of the trace. Answer:Cat 1CS 4.Woman in 2nd stage _ primi_
more than 3hr _ with epidural anesthesia_ pushing for 1 hr_ head station +2 _ fully dilated CTG: 1
conracion every 7_8 min.

TE PROPHYLAXIS
1.65 yr old operated for major surgery,
2. 67 yrs, TAH, epidural Anaesthesia
Options - LMWH after 2-4 hrs surgery -LMWH after 6- 8hrs - LMWH after 12 hrs - LMWH after 24 hrs.
--
PRE PREG COUNSELLING:

117 | P a g e
1. prev 2 miscarriages, one, APL Ab present, no f/ h of Thrombophilia
2. BMI 30, Factor v present, APL Abs absent
Options : - reassure - aspirin - aspirin+ LMWH -LMWH

VULVAL CANCER
1. 67 yrs, operated for wide lesion, margin 8mm, out of 12 LNs, 1 is 3mm deep seated, capsule not
ruptured.
2. Operated for wide lesion. Margin free, out of 12 LNs, 3 deep seated with ruptured capsule.
3. 78 yr old, 4 cm mass near to anus,
Options: - re excision surgery - radiotherapy -chemo radio - dont do anything - wait further -
neoadjuvant radio followed by surgery - neoadjuvant chemo radio followed by surgery
PHYSICAL FINDINGS ( what r physical finding s)
4. Severe painful intercourse, not relived by lubricant post cotal bleeding,
5. Vaginal irritation, soreness, fissuring, not on any medication, no vaginal discharge
Options: - lichenifivation - atrophic vaginitis -Endometriosis -strawberry cervix
6. 28 yr Woman having vulval irritation, on aceto white mosaic patterrn, vascular - VIN -HPV -HSV - -
-
7. cronic abdominal pain, .(..?? Menstrual problems), severe pain in micturition but mainly during
menstruation -Endometriosis -interstitial cystitis -inflammatory bowel syndrome - -
8. Woman having severe heavy menstrual periods, on peritoneal dialysis, what is cause of her
problem - Wilson disease - - -
ABORTION ACT 1967 UK
9. Woman having two children, one having esperger syndrome, need to give constant attention,
unexpectedly conceeived, now 11 wks pregnancy
10. Woman requesting termination on social ground
Options: -ground A -ground B -ground C -ground E -ground A+E - ground D - dont comply abortion
Act - not related with Abortion act
11. Rashes on palm, sole, body, sero negative for STI
12. Single Painless nodule on vulva, papule
Options: -hsv -hpv -infective syphilis - tertiary syphilis - lympho granuloma venerum -parvo virus b
19 - - -

118 | P a g e
TUBAL PREGNANCY: UPT positive woman request least invesive treatment,
13. Asymptomatic, adnexal mass 23 mm, mild peritoneal fluid, ut empty
14. Pain in RIF, ut bulky, mild vag. Bleeding, bp 95/45, tachycardia
15. Ut empty, midline (? Sac, fliud collection, something like, dont remeber), no adnexal mass,
bhcg 980, after 48hrs 1150, after 48 hrs, 1265 (these figures I dont rmember exactly but suggestive
of PUL)
Options: -expectant management - methotrexate 50mg/ m2 im -methotrexate iv - methotrexate
locally in tube - lap unilateral salpingotomy -lap unilateral salpingectomy -laparotomy , unilateral
salpingotomy -laparotomy, unilateral salpingectomy
16. Women planned for lap'scopy appendicectomy, consented for laparotomy, appendicectomy.
Previously was rt oophorectomy for dermoid cyst. Per op appendix normal, lt oray adhere to pelvic
wall, has dermoid cyst. Your advice
- abundand surgery, explain, -do surgery as consented -others as usual option list, I dont remeber
much
17. Something like above question, consented for TAH, lt overy dermoid. ?. ( I don't rmember
exactly)
18. Woman conceived by IVF, after unexplained infertility of 2 yrs, previous 2 miscarriages, pain
RIF, ut, single sac, fetal pole present, cause of pain
- torsion overy, - ectopic pregnancy -endometriosis
- - -(ohhs) was not in list
PAPER 2: OBSTETRICS
MODE OF INHERITANCE
1. Preg Woman with two brothers, both wheelchaired, worried about her child
2. Previous 3 miscarriages, all down s syndrome, this time miscarriage fetal karyotype showing
downs syndrome
Options: -sex linked - non dysjunction -robertsonian translocation -sporadic - AR -AD - - -(as usual
other option)
CAUSE OF ANAEMIA IN PREGNANCY;:
3. woman afrocarrabian, feeling unwell, temp 38 degree, cough, (? Chest pain, crepitation... dont
remember),
4. Pregnant, bp 165/ 105, protein urine++, epigastric pain, vomiting
Options: - hemolytic -HIV infection, -blood loss -haemoglobinopathies

119 | P a g e
- - - -(as usual routin options)
HIV:
5. woman at booking, HIV seronegative, married to new partner, knew just, he is HIV positive since
6 month. Worried about her baby infected by her partner. -reassure -reassure of immunity -consult
paediatrician -wait till her become seropositive (????? dont remember) -cord blood for HIV test - -(
all above options I dont remember exactly)
6. What kind of infection is it?
HBsAg +, HBeAg +, HBcAbs +, HBV DNA +, ( some other related to HBV related Ag, Ab negative, I
got tired to write down all)
-chronic infections with high infectivity -chronic infection with low infectivity -Acute infection - - -
(dont remember other)
IV FLUIDS:
7 Type 1 DM, planned for cs at 9:00, insulin injection (? Infusion...) given, 8.30 Hb1AC 8.9
9. Type 1 DM, (feel unwell, vomiting....????, dont remember exactly), Hb1AC 29 or something
Options: -5% dextrose -0.5% dextrose -0.9% sodium chloride -9% sodium chloride -hartmann' s soln
-????? Sodium chloride with potassium ( dont remember) - blood transfusion
SIGNIFICANT APH IN THIS PREGNANCY, PREVIOUS 3 DELIVARIES, ONE BY CS:
10. 27+3days, bleedind stopped now, breech presentation, ut contractions 1 in 10 min, Cx 2cm
dilated, 2.5cm long
11. 22+3days, frank breech, brownish discharge from vagina, membrane bulging from vagina
13. 36 + 5days, breech, ut contractions 3 in 10 mins, CTG normal, 3cm dilated
Options: - expectant mamangement -admin steroids, cs after 24 hrs, -admin steroid, vaginal
delivery ( something like that) -emegency cs - - -
SGA RELATED ( dont remember, but like this... as they were very long ques)
14. 10th percentile, 39+2wks, umb doppler normal, AFI 159 mm, (? AC normal, something like that)
15.10 th percentile, AFI 67 mm, umb doppler raised PI, reduced end diastolic flow, (???? 37+2 wks)
- Send to midwife for expectant manage -induction of labour -emergency cs -admit, steroid,
induction labour, - - (Dont remember much)
RELATED TO DM
16. Type 1 DM , 3rd preg, prev two vag delivarie s- 37 wks, baby 3.7 kg, 39 wks baby wt 3.9 kg
Hb1AC ....?. normal... AFI normal, ut doppler normal, 50th percentile growth ( like that... very long
ques, dont remember exactly)

120 | P a g e
17. Something like above but very long ques.... request for vag delivery (? All parameters normal)
Options: Induction bet 37 - 40 wks - electve cs -

Pt had treated ca. breast 5 yrs back(estrogen positive),now vaginal dryness. options.- HRT
contraindicated combined HRT Vaginal estrogen tablet or gel(dont remember exactly) sequential
HRT

histology showed <50% myometrial involvement


TAH-BSO radiotherapy chemo TAH

Pt had treated ca. breast 5 yrs back(estrogen positive),now vaginal dryness. options.- HRT
contraindicated combined HRT Vaginal estrogen tablet or gel(dont remember exactly) sequential
HRT

1. Third degree tear suture choice PDS 2. B lynch suture choice monocryl

Scenario had combinations of tapered round bodied and number as well.

3. CAH partner affected women normal risk of baby

4. one more I can't remember but mentioned in earlier emails

5 identify the organ

On scan TTTS develops this organ disappears from donor twin ( ans bladder )

6 structure identified as abnormal in prune belly syndrome ?? Stomach

121 | P a g e
7 a case of lumbar meningomyelocele this structure in brain showing changes concavity at anterior
end ( ? Fora men ovale ? Cerebellum ? Ventricles)

8. Addiction services Referred by community midwife h/o binge drinking at 14 weeks

9 addiction services a woman divulging at booking she is a heavy cannabis


smoker since age of 16 yrs

Options included referral Telephone. Routine referral urgent referral no need etc

10. Genital herpes 2 scenarios both she had herpes one partner new developed herpes other one
she had herpes at 32? Weeks partner has now what is needed at term now

11. Rubella parvovirus combination One scenario women investigated for rubella after exposure
which came out negative but parvovirus Ig G positive on report what is next initial step ( options
like send bloods for various combinations there like IgM , Ig G etc both together

12 another scenario detected hydrops at 32 weeks options including intrauterine fetal blood
sampling my answer can't remember if other options had things like termination

13 ethics

12 ur old girl deciding to agree mirena coil principle beneficence ?

13 a woman refusing Caesarian section although her partner wants it autonomy

15 Recurrent miscarriage scenario next initial step

Seeing a woman in clinic bloods had been sent and report mentions IgM 40 my ans repeat another
one in 12 weeks time

122 | P a g e
16 Another scenario where 2 blood samples given normal excluding APAS not sure what should be
answer can't remember options please feel free to edit here

17 referral for infertility

3 scenarios

First secondary infertility with 2 children last miscarriage few months ago female 31 male 35
options many my answer referral not needed or other confusing options were wait for 6 more
months

Second

Third

18 headache questions Repeat

19 CTG questions straightforward Hyperstimulation terbutaline Pathological CTG at 9 cm in primi


cat 1 LSCS One more early decelerations change position

20 osteoporosis / osteopenia mentioned in earlier emails

21 HRT hot flushes family h/o breast cancer

Two more mentioned in mails can't remember exactly

123 | P a g e
22mcq on GBS incidence of neonatal GBS varies across uk in different areas Approx 1000 some
need antibiotic to prevent one death

23 Mcqs verifying all points n percentages for consent of hysterectomy like bleeding risk of return
to theatre two more

24 Mcqs verifying counselling for VBAC rupture risk in preterm labour induction of labour

25 PPH risk decreased by 40 % guideline Oxytocin increase risk of neonatal jaundice ? Water
intoxication true

26 hemorrhagic disease of new born risk more with oral vitamin K than injectable Risk more in
preterm babies

27 puerperal mastitis Mcqs Continue to Breast feed can't remember rest

28 . Endemetriosis

29 . Medical education 2 scenarios

Mentioned in earlier emails

30 ovarian cancer referral various combinations given whether to refer literally had to RMI
calculation to decide to whether refer or repeat ultrasound in 3-4 months or annually or repeat just
bloods all sorts of ultrasound findings with combination of premenopausal post menopausal and
ca 125 given

One can remember thrombosis q Q1.pt had previous dvt in pregnancy lastone now 10 wks
ante..post natal for 6 wks

124 | P a g e
Risk of postpartum psychosis if the patient has a previous history of postpartum psychosis . A)
50% B) 25% C)>50% D) 60% Incidence of postpartum psychosis in gen population A) 1 /1000
B)4/1000 C)1/4 D)1/100
Universal sign of molar pregnancy A.thyroid dysfunction B. INCREASED UT SIZE C. BLEEDING
D. HYPEREMESIS
30 yr old P2+0 wd 3rd degre uv prolapse best managmnt? 1.manchestr repair 2.sling operation
3.vaginal hystrectomy 4.pessary
Incidence of suicide after childbirth is A) 1 in 1000 B) 2 in 10000 C) 1in 10000 D)2 in 100000 E) 1in
100000
epidural anaestheia most common complication? spinal Headach Inefective technique Fetal
distress Hypotension
PG 32 weeks with bp of 160/100 alb 3+ pedal edema.what immediate treatment should be given
Hydralazine Methyl dopa MgSO4
1-Risk of miscariage in 44 yrs old woman (10-20-30 40-50) 2- pregnant woman 38week breech
presentation with intact membrane , pulsating cord 5cm dilated cervix (1st category cs, 2nd cat cs ,
perform breech extraction , 3rd cat cs , ,,.....)
Primigravida 40wks +5days bishop 2ctg normal next step?
a)Wait b)Induction c)Sweeping d(Cs
Pt with abruption, iud,labor pains having pv bleeding since last 24 hrs platelets count 30000,other
cloting profile is normal, Bishop os 2cm,cx log post, how will u manage A ARM+synto B c/s C
induction with PGE1
oral iron takes how much time to be effective in last trimester? a)56 days b)75 dys c)100 d)120 days
Metabolic alkolosis with hypokalemia is cause by except Frusemide Cushing, s syndrome
Addison's disease Vomiting
Postpartum psychosis appears in ( after 2 days -four to 7 days -14days-.......)
Mifpriston and misoprostol protocole for termination of 26 wks ,iufd?
32 years old with premature ovarian failure most common cause? Familial Idiopathic Turner
mosaic Autoimmune

125 | P a g e
44 yr old,cx smear show low grade dyskariosis,hpv test is negative,next step?
Pt with primary sub fertility all well how long to wait to start treatment??? Like wait 24 month 12
month ??? Pt with subgertility with lh fdh too low 0.2/0.3 and prolactin 657 so next step karyotype
Adult teaching question: 1) given a case to discuss scientific , ..... (I marked problem based learning
) 2) can't remember much but was easy 3) group discussion to identify own learning .... (Marked
peer ... ) Any comments ?

in congenital varicella syndrome mst common compl?a)atopic dermatitis b)cataract c)PDA


d)thrombocytopenia

mode of delivery for moderate hydrocephalus?a)c/s b)cephalocentasis with nvd


Not a cause of dic? ? Ca cervix Obesity DM pelvic surgery
What about cases of placenta previa answers?casea of diabetes?
Risk factor for early pregnancy loss (maternal age under 25 -paternal age over 40 -multi para)
1-Commonest cause of death in hypertensive woman with pregnancy (cardiac -anathesia
eclampsia) 2-common assotiation with myocardial infarction (asthma-hypothroidism-ceiliac
disease)
Ureteric inj in and hysterectomy for endometriosis....options were 1 in 2,1 in,3 1in
4,5,6...? Another FSh-28,LH-21,pal 450....FSh 0.2.lh 0.1,pal 760.....third another...appropriate
investigation...anybody who have appeared got a better idea
73 yr woman with urge and stress incontinence nocturnal uria and urgency option oxybutynin anti
cholinergic tvt tot many option Four month post natal genuine stress incontinence confirm on
urodynamic agitated with it so option physiotherapy bladder retraining and many option One
scenario for mirabegron tablets
Pt with rash maculopapular rashes on and but on and only what type of rash is this that causes no
harm to fetus like option were rash sparing mucosa rash on palm and soles and... Very positive
next step like hemagglutination or for or fluoroscopy or ... Then treatment of syphilis Also consent
question on like 14 yr girl for epics and boyfriend 14 so ?? Consent which type and report to school
head or parent or youth wing Second consent refused doctor on basis of benefits beneficiary
Consent of appendix laparoscopy and laparotomy found kV cyst next step do as planned
Consent of mother refused for c section on pathological ctg but husband insist on c section

C section of a fully seated women best options for delivering the impacted head by pushing ftm
down by nurse assistant during c. Section or reversed breech or t incision in uterus

126 | P a g e
Pt with post hysterectomy threescenario Like 1 st having tachycardia lowbp and our fever ??
Infection 2 and pinpoint pupil took too much painkiller so answer naloxone 3 Rd not mobilization
after hysterectomy low output so ?? Dvt
One of with 26 week prom give betamethason Second prom received steroids so now erythromycin
The 2nd question she was 28 wk PPRom she recievied steroids and now uterine tenderness and
marked oligohydraminos ?? I think CS
Recalls march 2017 1- endometrial hyperplasia with minimal glands , marked stroma and atypical
changes ( complex hyperplasia with atypia-simple
witth atypia -complex without atypia-simple without atypia) 2-pregnant women with boy son with
partener hemophilia A carrier ,percentage of affection of her baby (0-25-50)
Post menopausal woman with cyst first scenario with simple cyst 4.5 cm repat scan 4 month option
Second scenario with complex cyst then next plan ??? Pl previa occuranxe after 4 c section Pph
with pl previa hemostasis secured but lt angle bleeding after synto so next plan ?? Uterine artery
ligation or call interventional radiologist for internal iliac or carbopost Pph with atony synt
carbopost blynch donenow next step One pt with 4 cm fully effaced cx then 8 cm after 4 hr next
step wsit for normal progress Second with 5 cm 2/10 cont then 4 hrs after same so now intervention
like options were synto arm arm with synto

Prev 1 placental abruption? Risk of recurrence?


Pt with PID sensitive to metronidazole with pregnancy ??28 week multiples of antibiotics like
azithromycin/erythromycin/cetriaxone/ and mamy combination
Then a pt post natal pt for anemia answer was 10 hb Pt with routine pap smear and found
actinomycetes asymptomatic so reassurance only

Recall 2017 1-shoulder dystocia rate to normal population rate (two or 3 or 4 or 6 or 10 fold
increase) 2-perimortem cs timing (2 or 4 or 6 minutes)
Risk of progression of endometrial hyperplaia with out atypia to cancer?
Blood vessel injured during 1-sacrospinous fixation 2-4th degree perineal tear 3-removal of
endometriosis
Pt with htn and headache throughout of Found collapsed in house at 28 week 185/114 BP cause
was subarachnoid hemorrhage Option central venosis thrombosis and few more About pco large
email with lady is 25 yr old and all well 3-5/35/60 day cycle k/c pco with estrogen 1400 at day 16 and
progesterone <3 on 23&26 day of cycle and took cycle clinic 50 mg for 5 days and follicle 2 cm
What should be next cycle protocol like I wrote HCG in next cycle with clomifine but some says
direct IVF Forceps for skin edges apposition ?

127 | P a g e
Recalls.. Zika virus testing...serology to be repeated?? 4 wks..8 wks..12 wks Incidence of previa
after 4 scar Iud of one monochorinic twin...refer for fetal brain mri or deliver at 36 wks

19 yr old with 6 week of home pgtest + with mild pain less bleeding so next step reassuring a As
visiting midwife Onewoth of on crl 90 mm with AC BPD ??? No the next step ?? Wait for
conservation management
Flesh like papules on labia majora next could be warts HPV Anti d questions like normal inevitable
abortion admitting 1 finger with bleeding taken for surgical procedures so give 250 ii of anti d Other
with spontaneous abortion so no anti d

Epilepsy which drug development with PvP Like phenytoin carbmazapi or valopro Also pt sle with
multiple drugs which drug to change if want to be pregnant Also one scenario breast cancer
positive estrogen
receptor then treatment done now ?? Want contraceptive pills or what ???
Recall today 2017 Mar Confirmatin of post hysteroscopic sterilizatin is by HSG after 3 months
With absence of other factor , sickle cell dis consider antenatal thromboprophylaxis About 5
question for thromboprophylaxis. With one was like having gross barcode veins and had hx of DVR
once treated so answer were management like start aspirin or thromboprophylaxis with high dose
heparin or anti embolic stocking or compression stocking The most common complication after
TAH (Bladder or bowel or urter The most common site of uterine perforation with surgical
management of abortion is anterior wall The mosr common cause of maternal death related to ht
with pregnancy Inteacranial hage . Eclampsia . ARDS. Schincteric injury associated wuth ....fold
after operative delivery Recurrence with Schulder dystocia ?? and abru One question with
metanalysis study and questions was like study literature if morena to see it's benefit like
something and Is mrcog 2&3 informative or summarize type

Recall 2017 March One of with k/c DM type 1 controlled DM with 32 week all parameter ok except of
Ac is 40percentile and biometrics written ctg ok so next step Option were many like Doppler and
insulin and other but I wrote as revisit at 36 with ctg Second with this email was poorly controlled
DM with 32 week gest so next step I think was Doppler as it was at 10 percentile and afi was 60 mm
In screening for pt his 6 month prrgnant found HIV negative and husband found positive want to
know risk of baby answer no risk

Baby delivered with swelling in head palplabe and not bounded by bone ?? Answer was subgaleal
hematoma option were cephalometa and caput
Preg woman visited some where then have 2% paresitimia and all symptoms like malaria falciparum
Flu like symptoms with rash at bak of eat and limb and chest ?? Which inf like zika or parabolic ???

128 | P a g e
INCIDENCES/FIGUERS:

1.Patient with endometrial hyperplasia without atypia risk of progression to endometrial cancer:
A.<1%
B.2%
C.4%
D.10%
2. Risk of placenta previa after 3 CS:
A.3%
B.10%
C.30%
D.50%
E.60%
3. Para 1 CS due to labour dystocia want to know her chance to have successful VBAC:
A.30%
B.40
C.60%
D.70%
E.80%
4. Patient for elective CS at 38wks need to know how much steroid will reduce respiratory morbidity
at this GA:
A.4-6%
B.40%
C.50%
2

D.60%
E.70%

129 | P a g e
5.Patient with previous abruption need to know recurrence in current pregnancy:
A.3%
B.4-6%
C.10%
D.19%
E.25%
6.Previous shoulder dystocia want to know recurrence compared to general population:
A. 2fold
B.3fold
C.4fold
D.5fold
E.10fold
7.female and partner retained from trip from somewhere suspected zika virus infection when to
check for zika virus infection or seroconversion:
A.2wks
B.4wks
C.8wks
D.12wks
E.16wks
8. The most common time for presentation of post partum psychosis:
A. 1-3 days
B.1-3wks
C.4wks
D.6wks
E.8wks
3

9. Haemophilia male female stutus not mentioned pregnant by baby boy want to know risk to baby:
A.zero

130 | P a g e
B.1in2
C.1in4
D.1in8
E.1in 16
10. Precious puberty the cut off time:
A.7yrs
B.8yrs
C.9yrs
D.10yrs
E.12yrs
11.Hospital want to benchmark still birth rate which is consistent with UK stillbirth rate:
A.1in 200
B.0.5/1000
C.5/10000
D.5/100000
E.
12. Unrecognised ureteric injury during laproscopy:
A.10%
B.15%
C.30%
D.40%
E.50%
13. During laproscopy for severe endometriosis ureteric injury is:
A.1in 5
B.1in10
4

C.1in 20

131 | P a g e
D.1in 25
E.1in 30
14.After ventose delivery ask about the accepted preductal oxygen in 2min:
A. 60 - 65
B. 65- 70
C. 70 - 90
D.80 - 95
E. 90- 95
15. When prescribed complication in OB&GYN as very rare means:
A.1in 1000
B.1in 1500
C.1in10000-1/100000
D.<1/100000
16.40 years lady first trimester pregnancy ask about her risk to have miscarriage :
A. 12%
B.20%
C.30%
D.40%
E.50%
17.Patient need to have forceps delivery in second stage ask about risk of 3/4 perineal tear with
forceps:
A.5%
B.10%
C.15%
D.20%
E.25%
5

132 | P a g e
18. Patient present in labour nulliparous was low risk following with consultant serial scan baby in
70th centile how to follow her in labour:
A.intial cefm for 30 minutes then intermittent auscultation
B. intermittent A using hand held Doppler
C. intermittent auscul using ctg machine
D.CEFM
E. US to see fetal heart
19. Patient delivered baby at the acid base PH7.1 HCO -11 at zero APGAR 3 then 5 and 9 he and his
mother did fine for how long do you keep the ctg paper:
A.5yrs
B. 10yrs
C.20yrs
D.25yrs
E.indefinitely
20.Main cause of litigation due to ctg is:
A. failure to act
B.failure to recognise an abnormal one
C.failure to monitor
D.failure to refer
D.inappropriate oxytocin use
*Choose the most appropriate action
A.CS
B.oxytocin
C.ARM
D.CEFM
E.exam in 2hrs
F.forceps D

133 | P a g e
21.multiparous poor uterine contraction 2/10 admitted cx os 4cm check in 4hrs 8cm MI
22.nulliparous contraction3/10 admitted cx 4cm checked in 4hrs was cx 5cm
23.Unstable neonate post ventouse delivery low APGAR found to have scalp swelling with ill
defined edges whats your diagnosis:
A. cephalohaematoma
B.subglialial H
C.capaut
D.ICH
E.chingon
* Choose appropriate mangment
A.cat1cs
B.cat2 cs
C.cat3 cs
D.cat4 cs
E.ECV
F.ARM
G.instrumental delivery
H.exam in 2hr
I.exam in4hrs
24. 3rd pregnancy 38wks now breech present with reduced fetal movement twice normal US AF
breech extended and normal ctg opted for VD
25. parous term low risk pregnancy present in labour at 4cm intact membranes cord felt pulsating
through the membrane on pelvic exam
26. Patient low risk pregnancy at 39wks present in labour Cx 6cm fully effaced MI at ischial spine :
A. intermittent auscultation
B. CEFM
C.exam in 2hrs
D.exam in 4hrs

134 | P a g e
7

E.ARM
27. Patient in second stage of labour you want to infiltrate the perineum with lignocaine without
vasopressor how much you give
A. 1mg/kg
B.2mg/kg
C.3mg/kg
D.5mg/kg
E.7mg/kg
28.SLE lady pregnant worried about fetal risk what test should be done:
A. APS antibodies
B.lupus anticoagulant
C. Anti Ro &La antibodies
D.dstranded DNA
E.antinuclear abs
29.38 wks patient with primery herps plus HIV what measures you do to prevent vertical
transmission:
A. IOL
B. acyclovir
C. ELCS at 39wks
D.vaginal delivery
E.?
* most appropriate mangment
A.admit to control glucose
B.IOL 37 -40wks
C. increase pre lunch insulin
D.US for umbilical artery Doppler
E.CS at 38 wks

135 | P a g e
F. reassurance and to be seen in 2wks
G.many others options
8

30. known type 1 daibeties at 36wks GA controlled in insulin HBA1C 6.5% came after lunch to
diabetic joint clinic urine++ glucose US baby ok in 40th centile otherwise patient stablE.
31. known diabetic at 33wks variable control HBA1C 7.4% urine++ glucose US baby in 10th centile
otherwise ok.
33. type 2 diabeties para 2 with 1 previous CS HBA1C 7.4% US baby in 70th centile keen for vaginal
delivery
*Causative agent
A. zika virus
B.Epstien bar virus
C.p.falcipram
D.measels virus
E.varicella virus
F.others 2
34 . Patient pregnanat came from Zambia found to have parasitaemia >2%
35. pregnant present with flu like symptom tell to come back if she devolped any …presented 2days
latter with itchy red spot behind ears and scalp then the forehead
35. pregnant at 38 wks admitted with pnaemonia her GP give history of generlised skin rash and
conjunctivitis and otitis media 3days ago
36. what is commonest cause for malaria in UK
A. p.malarae
B.p.ovale
C.p.falcipram
D.p.vivax
E.p.?
*suitable prophylaxis
A.aneinatal LMWH and 6wks post natal

136 | P a g e
B. highdose antenatal LMWH and 6wks postnatal
C.theraputic LMWH and 6wks postnatal
D.antenatal antiembolism stoking and 10 days postnatalLMWH
E.LMWH 10days post natal
9

F. thromboprophylaxis from 28wks


G. no need for thromboprophylaxis
h.many others options
37. smocker with HTN BMI 32 admitted to control her BP
38.onther pregnant with antithrombin deficiency and prvious history of VTE
39 .31 yrs lady undergone CS BMI 30 blood loss 1100 ml
40. patient with growth varicose vein and instrumental delivery
41 .Patient delivered vaginaly and devolped PPH whats the level of HB to define postnatal anaemia
A. 120g
B.110
C.115
D.100
E. 105
42.Breast cancer suspected in pregnant lady referred to breast specialist 1st line investigation:
A.mamogram
B.US
C.CT
D. MRI
E.CXR
43. Patient pregnant with breast cancer need chemotherapy needed
A. cyclophpshamide
B.anthracycline

137 | P a g e
C.steroid
D.etoposide
E.??
*coming step in MX
A.oxytocin
B.Bakri balloon
10

C. 3doses of carbpoprost
D.hystrectomy
E.uterine artery ligation
F.interventional radiology
G.B.lynch
44. patient with previous scar in her 2nd CS placenta found to involve the uterovesical space
deliverd and trying to control bleeding uterotonic and B lynch but still there is bleeding from one
corner at vesicoureteric junction.. anesthetist concerned but said patient stable
45.patient previous 3 scar have atony respond at start to oxytocin,ergometrine and 2doses of
carboprost intramyometrial but again start to bleed anesthetist concerned and mention patient
unstable
46. Patient pregnant came from vacation to some African area screening for syphilis was positive
need confirmation which test:
A. VDRL
B.tropenema agglutination particles
C. tropenema fluorescent test
D. lesion smear
E.RPR
46. patient with MCDA when to start US
A. 12wks
B. 16
C. 20

138 | P a g e
D.24
E.28
47.MCDA with co twin death next step
A. IOL
B. CS
C. MRI brain for other twin
D.? E.? forget
11

48.Ptient with HIV on HAART present at 35wks with PPROM VL <50 next step
A. CS now
B.CS at term
C.IOL now
D. wait for VD
E.?
49.Screening for GDM in current pregnancy for patient with previous GDM should be done when
A.at booking if negative repeat at 24-28wks
B. at 18wks and if negative repeat 24-28wks
C. at 24-28wks
D.at 28wks
E.at16 -24wks
50.Postpartum patient with preeclampsia devolped dyspnia ,tachycardia,basal creptation and O2
saturation 91 and low urine out put diagnosis :
A.pul embolism
B.pul.edema
51. pregnant lady with high BP C/O of headache for many days present collapse GCS3 BP 200/120
your diagnosis
A. SAH
B.CVT

139 | P a g e
52. Patient have massive PPH finding platelets 80 APTT 1.3 fibrinogen 1.2 what blood component pt
need
A. FFP
B.cryopreciptate
C. platelets
D. factor VIII
E.packed RBCs
12

53.Patient postoperative 18 hrs on PCA collapsed with pinpoint pupil RR12 normal O2 saturation wt
medication
A. adrenaline
B. naloxone
C.others
54.During second stage CS which measure has evidence in reduction of neonatal trauma:
A. delivery by breech
B. forceps for disimpaction
C. pressure from below by other
D.Pull method
* what action you do
A. deliver by CS
B. IOL
C.report to occupational health tomorrow
D. Reassure and check for seroconversion
E. start cART
F.many other options
55. patient asylum seeker refused HIV testing during CS on Friday evening your assistant ST2 have
needle stick injury
56. Patient pregnant HIV negative at booking discovered her hasband HIV positive 6month ago she
is worried about risk to baby

140 | P a g e
57. Perimortem cs time
A.3mins
B.4mins
C.6mins
D.10mins
* select the suitable Mx
A.steroid
13

B.plan delivery
C. uterine artery Doppler
D. umbilical artery Doppler
E.US for EFW
G.many others options
58.Midwife referred patient at 28wks with SFH less than 10th centile
59.Patient serial scan indicate static growth
60.Patient high BMI and first pregnancy smoke on occasion her sister has still birth because of
SGA want to know at 18wks GA
* Primigravida come to booking at 11 to 12 wks
A.CVS
B.amniocentesis
C.haemoglubinopathy for the hasband
D. use of nuccal translucency
E.non invasive maternal test
F. many other options
61.Patient known thalsaemia carrier hasband status unknown and cant test him he is in prison
62. Midwife did the family questionare patient and hasband born in UK ,hasband parent born in
Turkey

141 | P a g e
63. Midwife did family questionare patient and hasband born in UK hasband was adoption know
nothing about his mother
64. Pregnant lady combined test show risk for Down 1/12 patient declined invasive perinatal test..
second TM scan showed cystic hygroma and short femur whats your diagnosis
A.Down syn
B.Edward synd
C. Patau
D.Turner
65.Patient with protracted vomiting at 11 wks pregnancy first line antiemetic
14

A. cyclizine im iv oral
B.meteclopromide
C.ondansteron
D.corticosteroid
66.Couple with 1st and 2nd trimester miscarriage came for counseling what can be the most likely
cause of miscarriage:
A.women age <20 yrs
B. man age>40 yrs
C.working with vedio monitor
67. Common finding in ECG of patient with MI
A. depressed st in lead avl
B.elevation of t wave in V2 V3 V6
68. patient blood group negative received FFP group positive what you give
A.anti D 250
B.anti D 500
C. plasmaphresis
D. no need for antiD
69.Pregnant lady known haemophillia her baby status not known. plan of delivery will be and when
to check factor VIII

142 | P a g e
A. CS check factor VIII now
B. induction of labour
tm rdC.allow VD and avoid FBS and instrumental delivery check factor VIII in 3
70. Patient known Von W disease bleed during labour wt medication
A. fVII
B.fVIII
C.platelet
D. desmopressin
E.cryopreciptate
15

71.Asthmatic pregnant lady received short acting beta blocker and 800 steroid but her asthma not
controlled next step:
A. steroid
B. LABA
C.theophillin
D.leukotriene
72.Pregnant lady with renal transplant stable came for prepregnancy counseling which drug to stop
A.ciclosporin
B.predinsolone
C.calcium
D. Ramipril
73. Which condition put pregnant lady in high risk of MI
A.hypothyoidism
B.migraine
C.celiac disease
D.marfan
74. which condition without other risk factor let you consider thromboprophylaxis during
pregnancy

143 | P a g e
A.diabeties
B.sickle cell anaemia
C. IUGR
D.??
*AntiD
A.offer antiD 250
B.anti D 500
C. no anti D needed
D. paternal genotype
16

E.CFFDNA
F.measure maternal antibodies
G.CVS
h.amniocentesis
75. early pregnancy confirmed IUP 2wks ago at 8 wks presented with vaginal bleeding US done
empty uterus.
76. pregnant lady has previous hydropic baby father is DD RH negative
77. pregnant at 12 wks had vaginal bleeding and evacuation of ROPC after 4 days discovered RH D
negative
*antibiotics
A. gentamycin plus clindamycin
B. benzyl penicillin 3g then 1.5g 4hrly
C. benzyl penicillin 2.4 stat
D. cefodar different concenteration oral
E.augmentin
F. all PID regimen
78. Sudanese asylum seeker screening venereal disease confirmed by TPHA and also HIV positive
has mitronidazole allergy cause her vomiting and rash.

144 | P a g e
79. 3days postpartum referred by her midwife due to excessive lochia and clots abd pain and mild
pyrexiA.
80.Pregnant lady with headache no neurological deficit O/E what investigation
A.CT with contrast
B.MRI without contrast
C.MRV
D.2 others not remember
* maternal mortality
A.direct
B.indirect
C.coincidental
17

D.accidental
E.late
F.not maternal death
81. Lady with pre eclampsia developed ICH take 5wks in ICU and died
82. Lady is collapsed 48 hrs post delivery postmortem was Esimenger synd
83. Lady murded by her hasband
* early pregnancy
A. scan in 7 to 10 days
B.HCG in 48 hrs
C.surgicalmx
D.expectant mx
E. evacuation RPOC
F.others options
84. Pregnant lady US CRL 8mm no cardiac activity
85. Pregnant US show MGD 24mm
86.there is 3rd scenario

145 | P a g e
* Diagnosis
A. ectopic
B.appendicitis
C. OHSS
D.hetertopic
E. tortion
F.miscarriage
87. Surrogate for her sister retained 2 babies at 6wks confirmed single IUP present with sudden
onset of lower abd pain and tenderness
88.Lady after egg collection of 20 folliclle present with abd pain and sense of fullness in the lower
tight clothes at width (not exactly but near)
89. Young lady present with sudden onset of LT iliac fossa pain nausea and vomitinG..
18

90.Patient with RH D negative kell negative devolped PPH need blood


A.O negative
B. cross match blood
C.others
91. Indication for IAP for patient had GBS in previous pregnancy and had healthy baby :
A. previous colonization

B. GBS bacturia in current


C.PPROM
D.PROM
92.Which analgesic should be avoided during sepsis:
A. morphine
B.NSAID
C. cocodamol
D.paracetamol

146 | P a g e
93.Booking US CRL 90 BPD 12 AC ? what to use to date her pregnancy
A.BPD
B.AC
C.CRL
D.HC
E.FL
* TOP next step
A. reassure and prescribe analgesic
B.surgical evacuation
C. US
D. do pregnancy test 1wk later
E.others
19

94. early pregnancy loss follow medical TOP call gyn C/O of cramps and some bleeding otherwise
ok.
95. 14 days post medical TOP call the midwife that her PT is positive
96.Pregnant with IUFD at 26 wks wt the best regimen to induce labour :
A. mife 200mg miso 100 mg 6hrly max 4doses
B.mife 200mcg miso 100mcg 6hrly 4doses
C. mife 200mg miso 100mg 6hrly 5doses
D.mife 200 mg miso200mcg 6hrly 5doses
97.Pregnant lady with rash involve the abd striae what is good prognostic finding for baby
A. involve face
B. periumlical spare
C.presence of C3
98.Cystic fibrosis both parent carrier under gone IVF 12 embryo how many will be affected
A. 2
B.3

147 | P a g e
C.4
D.6
99.Evidence based step to avoid perineal trauma during vaginal delivery
A. perineal massage
B. hand on technique
C.warm compresses
100. CEMAC report 2006-2008 common cause of death in preeclampsia is:
A. eclampsia
B.ICH
C.P.edema
* Demonstrated pictures for many instrument
101. instrument used to dissect the ureter in abd hysterectomy
Lahey
20

102.Instrument used to held skin edges together for staples to be in situ


Allis
103.Hyperplasia description low gland to stroma ratio but cells show large nucleus
A. simple without atypia
B.simple with atypia
C.complex with atypia
D.complex without atypia
104 .Patient with HMB diagnosed as having endometrial hyperplasia without atypia whats the risk of
it to devolp cancer
A.<1%
B.3%
C.4%
D.8%
105.Couple with infertility of one yr women 25yrs man 40yrs whatis the most likely cause;

148 | P a g e
A.unexplained
B.male problem
C.tubal factor
D.uterine factor
E.ovulotory
* infertility
A.clomid 50mg od
B.clomid 100mg od
C.clomid 50mg +timed hcg
D.clomid 100mg +timed hcg
E. gnrh analo
F.ovarian drilling
G.IUI
h.IVF
21

106. infertile couple man ok woman PCO with anovulation induced with clomid 50mg estrogen level
was high follicle 20mm but progestron on day 23 of 26 cycle showed un ovulation 3 ithink
107. infertility 2yrs all investigations normal except woman PCO with anovulation evident by
progestron level
108. Infertility 3 yrs all normal woman PCO received 6 cycle of clomifen citrate day 23 out of 26
cycle progestron range between 32 to 67
*Oligomenorrhea
A.ocp
B.progestron
C.repeat hormone premenstrual wk
D.repeat hormone day 1 to 5 of cycle
E. PT
F. cc

149 | P a g e
G.CT
h.MR
i.karyotyping
j.many other options
109 .Young referred from her gp with oligomenorrhea every 3 to 5month ithink her hormonal profile
FSH 28, LH 11 prolactin 500.
110.Young oligomenorrhea feel nausea ,fatigue,breast pain ,control her family by barrier method
,,hormonal profile FSH 0.2 LH 1 prolactin 750.
111.19 yrs secondary amenorrhea hormonal profile FSH 88 LH high prolactin low
*oligomenorrhea
A. ocp
B.vaginal progesterone dialy
C.cyclical progesterone
D.merina
E.induction of ovulation
F.CC
22

G.other options not remember


112.Young concerned about her period .irregular last was 9month ago.BP 150/104 not in sexual
relation ship
113. same scenario with high BP and adult polycystic kidney PCO and not in sexaual relation.
114. 28yrs in relationship not want pregnancy concerned about her irregular perioD..
* Ethics
A. non malficience
B.beneficience
C. veracy
D.paternalism
E. autonomy

150 | P a g e
F.justice
G.others
115. Patient Down syn with HMB affecting her quality of life accompanied by her mother who agree
to offer merina to her daughter ..You discuss the mother and patient about merina pros and cons.
116.Patient with IUGR Ithink abnormal CTG need CS patient refused and said she rely on nature and
every thing will be ok.
117.Pregnant at 36wks ask for induction because her hasband will travel somewhere you refuse to
offer her induction
*study
A.cohort
B.case control
C.retrospective observational study
D.systematic review
E. metaanalysis
F.RCT
G.other option
23

118. DR conducted study over 15 yrs to see effect of carbiplatin on 5yrs survival of patient with
cancer
119.DR looks in literature to see effect of merina in HMB
120. DR conduct study among drs to see effect of smoking and non smoking in lung cancer .
* Learning
A.brainstorming
B.ischema activation
C.ischema refinement
D.1step perception
E.snowballing
F.goldfishbowel
G.icebreaking

151 | P a g e
121. The facilitator let group of learners to study about physiology,pathology of subject(not
remember it)
122. scenario for 5 steps of 1minute perception
123. Group of student sit discuss how to solve problem of project
124. Senario about trainee score 3 in appraisal his consultant comment that they extend to him due
to poor performance and he sit many times for part one .what to do for him now:
A. extend again
B.specific attention and not to extend again
C. questionable information need confirmation
D.stop training
125. Mechanism of action of asprin:
A. increase thromboxane
B. irreversible inhibition of COX
C.platelet aggregation
126 .Active metabolite of androgen is:
A.testesterone
24

B. DHEA
C.DHEAS
D.dihydrosterone
127. AED that reduced by COCs is
A. phenytoin
B.carbamazepine
C.lamotrigne
D. phenobarbitone
*Postmenopausal ovarian cyst
A.repeat US in 4 month
B.BSO

152 | P a g e
C. TAH+BSO
D. CA125
E. MRI
F.CT
G. expectant without follow up
h.unilateral SO
128.60 yrs present with mulocular ovarian cyst CA125 30
129. 50yrs present with simple ovarian cyst 4x4x4.5 and in her note there cyst 1 yr ago not followed
4x4x4.5 and CA125 is 25 (repeated question inall recalls)
130. 40 yrs lady present with multilocular or solid component( not sure ) not simple cyst CA125 30.
131. 9years girl came with her parent to the ER with sudden onset of Lt iliac fossa pain with nausea
and vomiting ithink high TWBC
A. analgesia and observation
B. diagnose cyst accident give analgesia
C. diagnose appendicitis sen for surgery
D. diagnose tortion and prepair for labroscopy
25

132. Patient with history of subfertility and PID present with Rt iliac fossa pain nausea and vomiting
…TWBCS 19.000 CRP 20 US non compressible mass 5cm diameter 10mm what is the diagnosis:
A. acute appendicitis
B. fallopian tube infection
C.pelvic abscess
* 5yrs survival
A. 40 -50
B.60-70
C.70-80
D.80-90
E. (not exact numbers)

153 | P a g e
133. Ovarian cancer in young did unilateral SO histopathology reviled tumor confined to ovary
intact capsule negative wash.
134. Cervical cancer undergone radical trachelectomy tumor completely excised found parameterial
invasion and no other abnormalities
135. Vulvar cancer histopathology come after surgery positive 1 LN with extracapsular extension.
136. 80 yrs lady present with 1cm vulval mass near the clitoreal hood next step
A. excisional biopsy
B. keyes biopsy from margin
C. wide local excision
D. biopsy from centre( ithink)
137. Young lady in sexually active present with pain less fleshy lesion at vulva diagnosis:
A.hpv
B.syphlis
C.herps simplex
D.H.dec
138.Lady using IUD for yrs asymptomatic cervical screening revieled actinomycosis
26

A.treat the condition


B.remove IUD
C. no intervention now
139. 50 yrs Cx screening mild dyskaryosis HPV negative next step:
A. colposcopy
B.RR in 3yrs
C.RR 5yrs
D. hystrctomy
140.50 yrs Cx screening high grade colposcopy unsatisfactory next step:
A.hysterectomy
B.multiple punch biopsies

154 | P a g e
C. HPV
d RR 5yrs
E.RR 3yrs
141. After how long risk of HRT for breast cancer revert like general population for lady taking HRT
for 5yrs after stop treatment:
A.1 yr
B.2ys
C.3yrs
D.4yrs
E.5yrs
*Vascular injury
A. Superior gluteal
B. inferior gluteal
C. ovarian
D. uterine
E.internal pudendal
27

F. internal iliac
G.others
142. Patient undergone laproscopic salpingectomy for ectopic pregnancy surgon tell intraoperative
haemostesis secureD..in the recovery room patient devolped hypovolumic shock retained for
laprotomy .
143. Patient durig VD had 4th degree tear and massive bleeding
144.Patient bleed after sacrospinous fixation
* unexpected pathology
A. abundant and medical ttt
B.abundant and further assessment
C.laproscopic removal of the tube

155 | P a g e
D. remove x from y
E.remove x &y
F. laprscopic biopsy and abundant
G. go as planned
145. Patient consented for laproscopic hysterectomy with past history of dermoid cyst removal.
Intra op surgeon find dermoid cyst X 4cm adherent to the pelvic wall in the overy Y.
146. Opened for appendicitis laproscopically appendix found normal but there is torted ischemic Rt
fallopian tube with watery dischargE..
*Post hysterectomy complication
A. wound infection
B. chest infection
C. UTI
D.infected vault haematoma
E. vault haematoma
F. bowel injury
G. check fluid blance
h.active bleeding
I .others post op complication
28

147. Patient smocker present 48 hrs post hysterectomy with fever temp 39 ,tachycardia and
tachypnia
148. 3days post op not recoverd well ask for analgesia with abd and back pain not febrile mild
tachycardia poor urine out put.
149. 12 hrs post surgery pulse 100 BP 90/45 poor urine out put but said in 3hrs its 80 ml O/E tender
abdomen..
150. The commonest site for uterine perforation during surgical evacuation
A.anterior wall
B.posterior
C. cervical

156 | P a g e
D.fundus
151. What you do to reduce risk of uterine perforation during evacuation
A. straiten the Cx caudally
B. done under US guidance
C. less Cx dilatation
152. Patient with breast cancer positive receptors on tamoxifen with severe PMS ttt
A. SSRI
B.OCP
C. E patch +merina
D. GNRH anal +tibilone
153. MRCOG 2+MRCOG3 is
A.summative summative
B.formmative formative
C. sum +form
D.form + sum
154. Multiple sclerosis patient with history of difficulty emptying bladder with high residual volume:
A. indwelling catheter
B. CISC
29

C.urodynamic
155. Commonest symptom of vault prolapsed:
A. vaginal bulge
B.SUI
C. constipation
D.voiding dysfunction
E.sexual symptom
156.Patient tried 3 antimuscurinic not tolerate them next step

157 | P a g e
A. mirabegron
B.trospium
C. deluxtine
157. SUI in 82 years old ttt
A. oxybutanin
B. merabegron
C.trospium
D.deluxtine
158. actually EMQ patient SLE on methotrexate and predinsolone devolped herps simplex with pain
and palbable bladder optins:
A.give acyclovir
B. refer to GUM
C.admit
159. EMQ patient C/O watery blood stained vaginal discharge and colicky pelvic pain wt finding?
A. polyp protruding through Cx
*Vault prolapsed
A. PMFT
B. PMFT &bladder retraining
C.ASC
30

D. SSF
E. pessay
F.pessary plus local estrogen
G.laproscopic SC
h. V.hystrectomy
160. 80yrs with vault prolapse and sopting normal vaginal exam patient had comorbidities
161.PHVP with short vagina
162. Patient with anterior vaginal wall prolapsed and uterine prolapse ask for definitive ttt

158 | P a g e
16 3. Patient athlet devolped SUI post VD O/E anterior prolapse grade 2 patient start PFMT not
improved next step
A. colposuspension
B. urodynamic
C. bladder retraining
164.Confirmation of post hystroscopic sterlisation:
A.it work immidiatly
B. X.ray with out time limit
C. HSG in 3month
165.The following enhanced recovery in gyn surgery
A. complex carbohydrate drink before major surgery
B.can drink up to 4 hrs to prevent dehydration
166. Risk of pelvic adhesion following midline episiotomy
A.10%
B.20%
C.40%
D.50%
167. Most common serious complication with abd hysterectomy
A. PE
31

B.urinary tract injury


C.blood transfusion
D.bowel injury
E.ovarian failure
168. Subfertility couple normal male partner female mild endometeriosis when to offer IVF
A.6month
B.12month
C.18 month

159 | P a g e
D.24month
E.30month
169.Which of the following reduce post operative wound infection
A. sheaving use clipers
A. wash with antiseptic solution
C.bowel preparation
* Pelvic pain
A.uretheral prolapsed
B.abnormal Cx
C.fistula
D.vaginal septum
C.thikening of uterosacral ligment
E.tender bilateral adenxial mass
170. 17 yrs with history of dysuria recurrent UTI and dyspareunia
171. Patient age ? with dysmenorrhea and blood with defecation( or something like this)
172.37 yrs yrs with history of chronic pelvic pain
173. Patient with suspected deletion of chromosome 9 or 10 want to know investigation of choice to
confirm
A. microarray CGH
32

B. PCR
C.FISH
174. 56yrs old thin vulval skin fused labia taken fluconazole orally and topically no relieve best
option:
A. biopsy
B.high potency steroid
C. emollient
MRCOGTRADEMARK GROUP

160 | P a g e
March 2016 MRCOG Mock
Paper 1 SBAs

1- A doctor needed to undergo an emergency termination of pregnancy in his own to save maternal
life.

What is the appropriate form he needs to sign? A Form MT


B Form MD
C Form HSA1
D Form HSA2
E Form HSA4
2- With regard to Interstitial Cystitis, which statement is true:
A has characteristic cystoscopic features.
B has initial characteristic cystoscopic features.
C treated by botolinum toxin-A
D- Caused by recurrent E-coli infection.
E- Usually treated by steroid instillation

3- 30 -year-old woman attends the scan department. A fetus of 0.5 cm CRL was identified on a
transvaginal ultrasound scan but No fetal heart beat was seen.
What is the most appropriate management option?
A. Repeat scan in 10 days
B .Repeat scan in 7 days
C. Offer manual vacuum aspiration
D. Offer medical management of miscarriage
E. Offer surgical management of miscarriage

4- Compared to warfarin, Dabigatran:


A- Is a direct factor xa inhibitor

161 | P a g e
B- Can be used safely without monitoring
C- Has no haemorrhagic complications
D- Safer for patients with atrial fibrillation
E- Has higher gastrointestinal haemorrhage,

5-A 92 year old woman admitted with procidencia. She suffers severe dementia and develops
pneumonia, which worsens to the extent that she now requires ventilation. Her respiratory function
deteriorates despite ventilation. A decision needs to be made regarding continuation of ventilation.

Who is able to make the final decision? A- An advance directive from the woman before the onset
of dementia which does not specifically mention mechanical ventilation B- Her daughter, to whom
the woman once confided that she did not want to suffer if placed in such a situation C- Her
consultant, using best interest principles D- Her husband E- Her son, who is lasting power of
attorney

6- A 54-year-old postmenopausal woman attends the clinic having been found to have a 6 cm solid
cyst arising from the right ovary. There is no other abnormality on scan. Her Ca 125
is 30. She is asymptomatic and the cyst was picked up on investigation for haematuria. Ultrasound
guided biopsy showed Fibrothecoma of the right ovary.
What is the most appropriate management?
A- TAH + BSO
B- TAH + BSO + omentectomy
C- BSO
D- Right oophorectomy
E- laparotomy staging procedure.

7- A gynaecologist is performing vaginal hysterectomy. There is a good descend but the introitus is
narrow. He is thinking of a way to improve the access and the exposure.
Which of the following will be of help?
A- Rameraz incision
B- Bolam retractor.

162 | P a g e
C- Schuchardt's incision
D- Cheffcot incision
E- Dührson's Incisions.
8- A 3 year old Somalian migrant girl is brought to A&E complaining of severe lower abdominal pain
soon after returning from a holiday back home. On examination a cystic pelvi-abdominal mass is
palpable. The mother of the girl reported same condition happened while back home and resolved
after been seen in the hospital. On examination her genitals look abnormal.
What the proportion of girls with same condition who will have the same complain?
A- 1:3.
B- Almost all
C- Up to just more than half of them.
D- 60 %
E-25 %.

9- Which one of the following true about biphosphonates:


A Vertebral but not hip protection

B Short term courses are advisable

C Do not Protect from jaw fracture on long term use

D Never Cause osteonecrosis


E Not advisable for long term non hormonal replacement therapy.
10- Regarding post hysterectomy vaginal vault prolapse (PHVP), the followings are true except:
A- Pelvic floor muscle training is an effective treatment option for women with stage I–II vaginal
prolapse
B- Vaginal pessaries are an alternative treatment option for women with stage I–IV PHVP
C- McCall culdoplasty at the time of vaginal hysterectomy is effective in preventing subsequent
PHVP.
D- Subtotal hysterectomy has no role in the prevention of PHVP

163 | P a g e
E- Suturing the cardinal and uterosacral ligaments to the vaginal cuff at the time of hysterectomy is
effective in
preventing PHVP following both abdominal and vaginal hysterectomies.
11- A 30 year old woman with BMI of 30 presents heavy frequent vaginal bleeding for the last 3
months. She used to have scanty periods every 8- 12 weeks. Transvaginal scan has shown nothing
apart from 2X2X2 cm subserous fibroid, endometrial thickness of 8 mm and small follicles in both
ovaries. No free fluids. Pregnancy test is negative.
What will be your next step? :
A- Offer progestogens from day 3-21 every cycle.
B- Offer progestogens from day 10-21 every cycle.
C- Endometrial biopsy & or hysteroscopy.
D- Offer laparoscopic myomectomy.
E- Offer gonadotrophine-releasing hormone analogue.
12 A 54 -year-old woman has been with pacemaker for arrhythmia and rheumatoid arthritis
diagnosed with an overactive bladder. The following can be considered:
A- Pelvic floor exercise for 6 weeks
B- OAB drug regime and bladder training C- Percutaneous posterior tibial nerve stimulation
D- Transcutaneous sacral nerve stimulation
E- None of the above
13- Which statement is true about women in UK:
A- have their first smear at age of 25 year.
B- have their first cervical smear by the age of 24.
C- have their first cervical smear by age of 25 year.
D- Have their last smear just before menopause
E- None of the above
14- Which of the following facts describe basaloid rather than differentiated VIN:
A- Not classically associated with CIN.
B- Common in postmenopause,
C- High risk of squamous cell carcinoma.
D- Graded like CIN.

164 | P a g e
E- Unifocal.
15- The operation of total abdominal hysterectomy under GA is being carried out at a peripheral
hospital , approximately 15 miles from the base hospital by an experienced Gynaecologist. There
have not been any complications for the last 6 months but you also note that there is no resident
medical officer in that particular hospital.
A- Report to GMC. B- Let practice to continue and perform an audit. C- Fill serious incidence form.
D- Stop the practice forthwith. E- Both A & C

16- 25 year old woman presents with on and off lower abdominal pain. On examination there is mild
tenderness in the right iliac fossa. Her full blood count showed Hb of 12 g/dl, TWBC of 11.5 X 109 ,
CRP < 1. Transvaginal scan shows a right complex cystic structure 7X 4X 5 cm, most likely ovarian
with minimal free fluids.
What is your next step?
A- Check CA125 level.
B- Check LDH, bHCG, AFP levels
C- Arrange MRI.
D- Urgent diagnostic laparoscopy &/or proceed.
E- Refer to surgeons.

You have been asked to obtain consent from a 32-year-old woman with cyclical pelvic pain for a
diagnostic laparoscopy under general anaesthesia.
What would you advise her regarding the overall risk of a serious complication?
A. 1 in 50
B. 1 in 100
C. 1 in 250
D. 1 in 500
E. 1 in 1000

18- 30 year old woman with chronic renal failure on dialysis, and uses warfarin regularly . Her GP
referred her with heavy menstrual bleeding.

165 | P a g e
What is the most appropriate management?
A- Stop warfarin.
B- Change warfarin to LMWH.
C- Bolus vit-K injection.
D- LNG IUS
E- Tranexamic acid.

19- 15 years woman attend the A & E with her father who is vicar. She complains of 1-day history of
left iliac fossa pain and mild vaginal spotting. A urinary pregnancy test for hCG is positive. As part
of your assessment vaginal examination is required. She insist you do not tell her father she is
pregnant and you consider her as competent in her judgement. Her father becomes angry and says
you must not perform a vaginal examination. How should you proceed.
A- Perform examination with chaperone present and tell her father that it is a routine examination.
B- Perform examination with chaperone present and explain that parental consent is not necessary
in this situation.
C- Defer performing the examination and document the situation fully.
D- Perform examination with chaperone present having assessed the girl's Gillick competence.
E- Perform examination with chaperone present having assessed the girl's Fraser competence.

20- After counselling the couple agreed for the husband to do vasectomy. They attended 10 weeks
later for semen analysis and the count was 50 000 motile sperm/ml. What is the appropriate advice
you will give? A. repeat semen analysis in 2 weeks B. Vasectomy has failed C. Vasectomy was
successful and they can rely on it. D. repeat semen analysis in 12 weeks. E. Mirena in addition

21- A 22 year-old-woman presents to the early pregnancy unit with mild left iliac fossa pain.
Examination is normal. She has a
positive urine pregnancy test. Her serum human chorionic gonadotrophin (hCG) is 800 IU/L. A
transvaginal ultrasound scan reports: ‘Bulky anteverted uterus with a 2 mm cystic area centrally
located within the endometrial cavity. Both ovaries have normal ultrasonic appearances. There are
no adnexal masses or free fluid in the pelvis.’ What is the most appropriate management? A.
Diagnostic laparoscopy +/- proceed B. Methotrexate therapy C. Serum hCG (human chorionic
gonadotrophin) measurement in 48 hours D. Serum progesterone E. Ultrasound scan in seven days
22 the following is true about the new version Levonorgestrel intrauterine system which is licensed
for contraception: A. Contain 52 mg LNG and releases 20 mcg LNG per day for 5 years B. Contains

166 | P a g e
3.5 LNG and releases 14 mcg LNG per day for 3 years C. less side effects than the previous
standard system D. Pear index is less than 0.1. E. Contains 3.5 mg LNG and releases 5 mcg per day
for 3 years

23 What is the most appropriate advice you will give for a patient with heart disease asking about
contraception: A- Cerazette should be avoided because of the increased risk of embolism, B-
Essure should be avoided because of the anaethesia risk. C Mirena rather than copper IUD is
preferred. D- COCPs are contraindicated E- Vasectomy should be considered. 24- Which of the
following gastrointestinal conditions also affect the vulva:
A. Crohns disease B. Hereditary hemorrhagic telangectasia C. Amoebiasis D. Ulcerative colitis
E. Polyposis coli

25- A 22 year old woman presented fever, chills, headache, rigors, myalgia , hypotension and skin
rash , few hours after starting co-amoxiclave as part of her treatment for vulval lesion. She is not
allergic to penicillin.

The most likely cause of her vulval lesion is: A Eczema. B Staph aureus C Treponema Ballidum D
Herpes E Filariasis

26 the following facts are true regarding bariatric surgery except: A- It is the first line management
for women with BMI > 50.
B- It is mainly to achieve weight loss.
C makes the diagnosis of GDM difficult.
D- Vertical band gastroplasty doesnot associate with malabsorption.
F- Pregnancy is better to be delayed for 1-2 years after surgery.

167 | P a g e
27- What is the best technique for detecting Chlamydia?
A.First void urine culture
B.First void urine enzyme immunoassay
C.First void urine nucleic acid amplification test
D.Self-administered HVS (high vaginal swab) nucleic acid amplification test
E.Self-administered HVS (high vaginal swab) culture

28- Ultrasound scanning machines are designed and built to conform to strict international
regulations. In the hands of well-trained, competent operators this equipment has proved an
invaluable resource, especially since, in contrast to imaging techniques based on ionising
radiation, there is no evidence that repeated exposure has cumulative and/or detrimental effects.
Which statement does not comply with ALARA principles?
A. The duration of scan inversely related to thermal index.
B. Machines of high mechanical index rather than high thermal index should be used.
C. Use of pulse and colour Doppler should be used as minimum as possible
D. All ultrasound machines for obstetric purpose should display mechanical and thermal indices.
E. Knowledge of biomechanical effect of ultrasound is prerequisite before starting obstetric
scanning.
29- A 60-year-old woman presents to the gynaecology outpatient clinic with a 6 month history of
urinary urgency, day
time frequency every hour and night time frequency of 3–4 times. She has no urge or stress urinary
incontinence. She has had three proven urinary tract infections over the past 6 months. She is post-
menopausal and is not on HRT. She has had two vaginal deliveries in the past.
Her general and abdominal examination was unremarkable. Vaginal examination showed atropic
external genitalia with a grade 1 cystocele and a grade 1 rectocele. There was no uterine descent.
Her cervix was healthy and the uterus was anteverted and small. No adnexal masses felt.
Urine analysis showed + of RBC and + of leucocytes.
What is the most appropriate next management step?

A.Urodynamics
B.Urine microbiology

168 | P a g e
C.Treatment with oxybutynin 5 mg nocte
D.Cystoscopy
E.Bladder re-training

30 32-year-old woman presents as an emergency with vaginal bleeding. She has never had a smear
test. Clinical examination reveals a mass replacing the cervix, extending onto the vagina and into
the parametria bilaterally. Initial blood tests show acute renal failure with a markedly raised serum
urea and creatinine. Imaging shows a 6 cm cervical mass with parametrial spread and bilateral
hydronephrosis and hydroureter to the level of the mass. Renal cortical thickness is normal.
The next most appropriate initial management:
A.Urgent radiotherapy.
B.To insert bilateral nephrostomies +/- ureteric stents.
C.Radical hysterectomy with pelvic lymphadenectomy.
D.Examination under anaesthesia and biopsy.
E. Opportunistic smear &/or HPV testing.

31- A 35 year old with heavy menstrual bleeding has been diagnosed with uterine fibroid. She was
about to start her family. What is the true advice you will give about uterine artery embolization:
A 50 % of women require second intervention in 5 years.
B Evidence showed that more than 75% of women remain fertile.
C Better outcome than myomectomy with regard to symptom control
D Premature ovarian failure in less than 5 %
E Complications usually occur in the first 3 weeks following the procedure

32- A 30 year old woman diagnosed with stage 2 cervical cancer and is on chemo radiotherapy.
She is brought to A & E with severe vaginal bleeding, BP 80/30, pulse 140/min and she is semi-
conscious. She received 7 units of blood and 3 L of
fluids, 4 units of FFP. He BP improves to 100/60 but she continued to bleed.

169 | P a g e
What will be your next step?
A Urgent palliative radiotherapy session.
B Urgent radical hysterectomy.
C Vaginal packing.
D Urgent intervention radiology arrangement.
E 4 g tranexamic acid bolus dose, then 1g 6 hourly.
33- Compared to first generation ablation, for women undergoing second generation endometrial
ablation what statement is correct?
A- Nausea and vomiting are less likely
B- Amenorrhoea is more likely.
C- Requirement for hysterectomy is less likely.
D- Patients are more satisfied at 2 years.
E- Cervical laceration less likely.

34- 82 -year-old woman presents to the GP with 2 weeks of bleeding. Her last period was at the age
of 54. Past medical history includes chronic hypertension treated with Atenolol, ischemic heart
disease and she on pacemaker. She has had one child at the age of 28 delivered by caesarean
section. Her BMI is 50 kg/m2. She had a 2 week referral for Gynaecology Outpatient where she had
a Pipelle biopsy. The results of the later, ultrasound and CT scan confirm endometrial carcinoma
confined to the endometrium with less than half of the myometrium invaded. She is understandably
devastated but very keen to discuss management with you
Which of the following options is the most suitable to be within her current management plan?
A .Total abdominal hysterectomy
B .Total laparoscopic hysterectomy and bilateral salpingooophorectomy
C. TAH + BSO + Inspection and palpation of the omentum and pelvic and para-aortic lymph nodes
D. TAH + Peritoneal lavage for cytology
E- Radiotherapy

170 | P a g e
35 A woman presents 5 days after being discharged from hospital with nausea, abdominal
distension and dysuria. She had TAH for AUB. She voided 300 mls of urine in 2 occasions before
being allowed home in her first admission. She reported being unable to pass urine since last night.
Bladder scan shows only 10 mls in the bladder. However, after inserting Foleys catheter, 350 mls of
urine is drained. Bloods show Hb 10.8 g/dl, urea of 3.5 mmol/l, creatinine of 108, potassium of 3, Na
136. Abdominal ultrasound shows dilated bowels, minimal free fluid, normal kidneys and liver.
What is your next step to reach a diagnosis?
A. Diagnostic laparoscopy.
B. None, as it is paralytic ileus.
C. Cystogram.
D. CT urogram.
E. Laparotomy.

36 65 year lady is about to have bilateral lymphadenectomy for ca vulva. Which nerve is at most
risk of injury?
A Common peroneal nerve.
B Femoral nerve.
C Obturator nerve.
D Sciatic nerve
F Pudendal nerve.
.
37- Considering anterior abdominal wall, linea alba is formed of:
A Tendons of internal abdominal obligue muscles.
B Tendons of transversus abdominis muscles
C Tendons of external abdominal oblique muscles
DA&C
EB&D
38 A 28-year-old woman is admitted via A&E to the gynaecology ward with an ongoing pyrexia
>38°C, lower abdominal pain, negative pregnancy test and foul smelling vaginal discharge.
Which of the following is the best treatment for her?

171 | P a g e
A. Oral ofloxacin 400 mg twice daily + oral metronidazole 400 mg twice daily for 14 days
B. Oral moxifloxacin 400 mg twice daily for 14 days
C. Intravenous ceftriaxone 2 g daily + intravenous doxycycline 100 mg twice daily followed by oral
doxycycline 100 mg twice daily + oral metronidazole 400 mg twice daily for 14 days
D. Intramuscular ceftriaxone 500 mg immediately followed by azithromycin 1 g/week for 2 weeks
E. Intramuscular ceftriaxone 500 mg followed by oral doxycycline 100 mg BD + oral metronidazole
400 mg BD for 14 days
39- After diagnostic laparoscopy, the patient had tachycardia and BP dropped to 90/45. The
surgeon reinserted the primary trocar and only minimal intrabdominal bleeding found. An
increasing haematoma was recognised at the site of secondary probe.
What blood vessel has been injured?
A Artery that may arise from the obturator artery
B An artery that may arise from external iliac artery.
C An artery that may arise from the femoral artery.
D all the above.
E Non of the above
40 The peritoneal lining drapes over the pelvic viscera and forms the part of the peritoneal cavity.
Which of the following is the most inferior extent of the peritoneal cavity?
A Vesicouterine pouch
B Paravesical fossa
C Rectouterine pouch
D Pararectal fossa
E Rectovesical pouch

41 A woman has just had a scan which has shown snow storm appearance. When counselling the
patient, which figure you may quote?
A risk of needing further treatment is 1:7
B The incidence is 1:970 pregnancies
C The woman should use contraception for 1 year after evacuation.
D If seen in twins, the live birth chance is 1:5.

172 | P a g e
E If it persists after treatment, the chance of cure is more than 80%.
42- Antibiotic prophylaxis is not indicated in the followings except:
A Cerclage.
B Diagnostic Hysteroscopy and dye test.
C Evacuation of retained products of conception.
D Therapeutic hysteroscopy
E Endometrial biopsy.

43 An epileptic woman is on COC. Which of the following drugs may require a dose adjustment:
A Carbamazipine
B Phenytoin
C Rifampicine
D Lamotrigene
E Topiramide
44 The quote which is not correct while consenting for surgical evacuation of RPOC is:
A- Perforation of uterus 1:200
B- Need for blood transfusion 1:100
C- Need for repeat evacuation 1:50
D- Bleeding lasts for 2 weeks 1:1000
E- Pelvic infection 3:100

45 22 A 30-year-old woman presents to the infertility clinic with primary infertility and
dysmenorrhoea and is found on ultrasound to have a 6 cm endometrioma in the left ovary. What is
the most appropriate initial management? A. Gonadotrophin releasing hormone agonist for six
months B. In-vitro fertilisation C. Intrauterine insemination D. Laparoscopic drainage of the
endometrioma E. Laparoscopic excision of the endometrioma

46 A 63-year-old woman developed vault haematoma following a vaginal hysterectomy readmitted


with fever, BP 90/50, and pulse 107/min

173 | P a g e
An arterial blood gas was performed, which showed the following:
pH 7.15 (7.35–7.45)
pO2 10.1 kPa (10–13)
pCO2 4.6 kPa (4.5–6.0)
BE 17 mmol/L (22–28)
What is the most appropriate definition of her condition?
A- Metabolic acidosis B- Metabolic alkalosis C- Mixed respiratory and metabolic acidosis D-
Respiratory acidosis E- Respiratory alkalosis

47 The following is untrue about rivaroxaban:


A- oral anti xa B- Has half live of 7-10 hours C- Can be commenced 6-8 hours post operatively. D-
Can be used in patient with mitral stenosis and Atrial fibrillation. E- Can be used in patients over 70
and congestive heart disease.

48- 26 year old lady comes to your clinic requesting emergency contraception. She had unprotected
intercourse 4 days ago. She is a para 1+2. Her son is 7 months old and she has had 2 abortions in
the past. She is on day 20 of her cycle and has taken Ellaone 10 days ago. She has no significant
medical or surgical history.
The best advice for her would be:
A.Repeat dose of Ellaone
B.Norethisterone tablet
C.IUS insertion
D.Copper IUCD insertion and STI risk assessment
E.2 tablets of Levonelle

49 A 15 year old girl is 10 weeks pregnant and has self referred requesting abortion. What is not
part of a routine consultation?
A. Inform parents as would need general anaesthesia for procedure
B. Explain risks and side effects of procedure
C. Consider child safeguarding issues by asking about details of partner

174 | P a g e
D. Check rhesus status as may need anti D
E. Check for Gillick’s competency as less than 16 years

50 With regard to malignant hyper-pyrexia which statement is not correct?


A Is autosomal dominant.
B Relieved by Dantrolene.
C Can be precipitated by polarizing and non-polarizing muscle relaxants.
D Can be precipitated by Halothane.
E Premedication with tubocurarine may have preventive role.

33

175 | P a g e
USS Features Of Adnexal Masses • PID: -Cogwheel sign:Typically indicate tubal inflammation in PID
with pyosalpinx or hydrosalpinx -Beads on a string sign -Increased vascularity • OHSS: -
Spokewheel appearance:Mul tiple cysts of varying sizes seen in OHSS • PCOS: -String of
pearlsappearance:Peri pheral distribution of follicles in PCOS • Endometrioma: -Ground
glassappearance:Hae morrhagic debris in endometriomas -Unilocular -Low echogenicity -
Chocolate cysts • Dermoid cyst: -Rokitansky nodule -Tip of iceberg sign -Dot-dash pattern -
Unilocular -Mixed echogenicity or hyperechogenicity with acoustic shadowing -Shining echos -
White line appearance(hair) • Struma ovarii:ectopic thyroid tissue in a dermoid cyst -Struma-pearl
sign -Increased vascularity • Mucinous borderline tumours: -Honeycomb sign: Tightly interrelated
septae • Haemorrhagic cyst:eg corpus luteal: -Spider-web sign • Ovarian torsion:More on right: -
Enlarged oedematous ovary -Peripherally located follicles -Free fluid in POD -Whirlpool sign:From
twisted vascular pedicle -Doppler:Absent blood flow • Ectopic: -Doughnut or Bagelsign:hyperechog

176 | P a g e
enic tubal ring.most common sign -Empty cavity 28% -Pseudo-GS in 20%.collection of fluid within
uterine cavity.centrally placed.But note for differentiation:In true GS:yolk sac present,eccentrically
within uterine cavity,double ring or double dicidual sac sign DDSS,intact midline endometrial
echo.However DDSS can be seen in one 3rd of ectopics -Empty uterus & adnexal mass 35% -Empty
uterus with ectopic GS with or without yolk sac or cardiac activity 12% -Fluid in POD 25% especially
if echogenic Causes Of Fluid In POD: • Normal Physiological -Up to 5 ml / Menstruation/Ovulatio n
Pathological: -Ruptured ectopic or tubal abortion -Ovarian cyst rupture -PID -Pelvic abscess -
Malignancies

Emq

What medication need to start now...


Aspirin
Calcium
Vitamin C and E
Methyldopa
Atenolol
No medication required
1-P1 with H/O severe precempsia and IUGR 650 mg at 28 wks.
Now came for booking visit at 8+ wks BP 120/80 with no proteinuria on dipstick.
2-Chronic hypertension on Atenolol now stopped before pregnancy type 1 diabetes
BP 138/78 with no proteinuria at 11+0 wks
Maternal death
Direct
Indirect
Late
Unclassified
Not maternal death
3-one lady died in car accident because could not wear seat belt because of pain in the scar of c
section 1 week before
4-pregnant lady at 5 wks investigated for breast lump but she refused treatment After delivery of
baby she was started extensive chemotherapy but died after 5 months despite extensive treatment
5-one lady died after acute MI in 3rd trimester
Pregnancy of unknown location
Ectopic too early to diagnose
Ectopic with failed trophoblastic
Repeat b HCG after 48 hours
PUV
6- pt 5 wks with mild vaginal bleeding .on USG thickened endometrium .no adnexal mass seen .b
HCG was increasing suboptimal less than65% 945 - 650 345. no free fluid in pod..
7-one with central 8mm sac in fundus another adnexal sac 2.2 ×2.2cm
No free fluid in POD PT was PCO conceived after 3 yrs. b hcg was 3400
Repeat bhcg after 48 hours
Systemic MTX

177 | P a g e
Laproscopic salpingectomy
Laproscopic salpingotomy
USG guided MTX into mass
8-pt with H/O old vaginal bleeding UPT +VE on USG endometrium thickened and Doughnut sign ++
Bhcg >1500 to less than 5000 ..it was 3500
Next management?

Migraine with aura


Migraine without aura
Subdural hematoma
Subarachnoid haemorrhage
Cluster headache
Tension headache
Thunderclap
(All scenarios with photophobia)
9-pregnant lady came h/o pressing
headache /band like for 1 hr before.it was not associated with or aggravated by physical activity or
exertion.neurological Ex was normal
10-a pregnant pt with bilateral throbbing headache. she has h/o diarrhea .she had some facial
paralysis or spasm with partial seeing of black spots .neurological ex was normal
11-a pt came with h/o headache which according to her worst headache of her life she had some
nick stiffness but no other findings
Continue same contraception
Female sterilization
Depotprovera
IUcd/Mirena
Start coc
Next management.?
12-15 yrs old girl unable to take prescribed ocp got pregnant and had top .Needs contraception
13-24 yrs condom break and despite EC got pregnant after TOP need contraception Mother diabetic
type 2.. grandmother had ovarian cancer.she have acne and irregular periods
14-lady had implant ..BMI 41 ..she had initially pv bleed on implant but now settled her implant due
to remove after 1year and came to u for removal after referral from GP for consideration of
sterilization-she want to remove implamt as she thought that osteoporosis is associated with
implant .This is the only reason to remove implant.
Genetics
1 in 2
1 in 4
1in 8
1in 7
1in 14
No risk
All boys affected girls no disaease
178 | P a g e
All girls carrier boys no disease
All carrier
50% carrier50% affected
15-both partners carriers of cystic fibrosis -risk of children affected?.
16-father Duchene muscular atrophy-risk of children affected
17-marfan syndrome-risk forchildren?
18- a couple has previous downsyndrome baby. mother have balanced translocation father normal.
risk for recurrence of diwn syndrome ?
FGM
19-fgm type absent/fused labia majora and minora
20-fgm -tear in old scar...pt wanted restitching but no bleed from tear
21-fgm- PG delivered uneventfully.she had fgm .she delivered a female baby .you assesed her for
risk to the baby and found no risk to the newborn.as a doctor ur responsibility for protection of
newborn?
-next step?
Immediate cs
Instrument delivery
Reasure and review in 30 minutes
Reassure and review in 1 hrs
Reassure and review in 2 hrs
Reassure and review in 4 hrs
Allow labour to continue
Fetal blood sample
instrumental delivery in delivery room
Instrumental delivery in theatre
Start I've fluid and reassess in 30 min
22-PG in active labour 3cm in 4 hrs FBS 1 hr earlier and PH 7.3 now 6 cm ctg showing deep decels
< 50% other features normal
Next step?
23-in active labour progressing well now 9cm CTG persistent deep decels more than 30 minutes
variability slightly reduced end of trace deeper decel and trace finished without knowing recovered
or no
24-ctg prolonged bradycardia to 90 bpm assessment vx DOA +2 heavy meconium-fhr not
recovering
Emergency cs
Iv fluid and reassess
Immediate cs
Can't take any action till further ctg
No action
Reassess in 30 min
Reassess in 1 hrs
25- Preterm labour low risk pregnancy ctg early decels no other abnormal features
26- pt in labour high temperature 38.4 CTG ---- baseline tachycardia 180bpm
Sutures
27-Repair of external and internal anal sphincter
28-Repair of perineal skin
29-Obese lady with midline vertical incision..need to stitch rectus sheath and peritoneum.
179 | P a g e
Iv fluid and reassess
Immediate cs
CNT take any decision till further ctg
No action
Reassess in half hour
Reassess in 1 hrs
Continue CTG for another 20 min
30-PG came at term with high grade fever 38.4 on ctg fetal tachycardia 180 bpm
31-Preterm labour low risk pregnancy ctg showed early decels no other abnormality
Tubo ovarian abcess
Pid
Osteoporosis
Hot flushes
Major osteoporosis fractures
Septicemia
PT refused proposed treatment what worst /possible worst consequences you have to tell her in
case of no treatment...
33-A case of PMOF at 31 years and refused HRT
34-A case of IUD... Had infection symptoms fever feeling unwell with vaginal discharge.. culture
showed bacteroid fragilis. refused loop removal and antibiotics
admit and steroids
Steroids
Fibronectin
Tvs
Reassure and discharge
35-A woman came with h/o previous one preterm delivery at 28wks and baby survived .Now came
at 31 wks with h/o lower abdominal pain .at start of pregnancy have hvs and found to be GBS +ve
.O/E Cx multiparous os.uterus was soft and non tender.no vaginal loss was there.
Administer Iv aciclovir
Reassurance
Give antiviral or antibiotics to treat mother infection
Give antiviral or antibiotics to treat mother and fetal infections
Give antiviral and antibiotics to treat baby and fetal infection
Start oral aciclovir
36-Lady 26 wks pregnant came with painful vesicular lesions
Not had such history before
37-A lady came with painful vulval lesions 36 wks on ex an isolated ulcer on labia majora pt had
history of HSV 9 months ago

Recall September 2016

1-A questionare was given for patients pain response as 0 -10


What type of data is this
a- Nominal
b- Ordinal
c - Categorical
180 | P a g e
2-Which symptom is the most commonly associated with the cerebral vein thrombosis?
a- Headache
b- Altered conscious
c- Papilloedema
d- Seizures
e- Focal neurological signs
3-What is hallmark symptom of overactive bladder?
a- Urge
b- Urge incontinence
c- Nocturia
d- Frequency
4-What is overall cause of maternal mortality in 2006-2008 CEMACE report?
a- Sepsis
b- Cardiac
c- Haemorrhage
d- Thromboembolism
5-Which medical condition is most commonly associated with the male fetus?
a- Acute fatty liver of pregnancy
b- Cholelithiasis
c- Pemphigoid gestationis
d- Obstetric cholestasis
6-Regarding neonatal hypoglycemia which condition usually require continuous monitering of BSL
after birth
a- SGA
b- MotherBMI >35 or 40
c- Post date
d- Instrumental delivery
e- Baby 4.2 k
7-A pregnant lady with h/o anorexia came at 34 weeks with history of pain over Rt hip joint initially
during walking then gradually unable to walk O/E tender what ll be ur most appropriate
investigation
a- X ray pelvis with fetal shielding
b- MR I pelvis
c- CT pelvis
d- Bone density scan (DEXA scan)

8-A young pt has h/o previous IUFD due to hydrops-gave h/o blood transfusion after one
accident.she came for antenatal booking.her blood group was B+ve.which RBC antigens r
significantly related to hydrops other than D antigen
a- Kell
b- Duffy
c- C antigen
d- E antigen
181 | P a g e
9-At what threshold of Anti D u will refer pt to feto maternal specialist
a- 0.2iu
b- 0.3
c- 4
d- 0.8
e- 12
10-A pt came immediately after blunt trauma to Abdomen she is Rh-ve her Kleihauer test
showed 4ml FMH what dose of Anti D required
a- 5001u
b- 200iu
c- 250iu
d- 650iu
e- 1500iu
11-Which test is used to confirm chlamydia infection
a- CRP
b- NAAT
c- ELISA
d- CULTURE
12-1 wk postpartum pt came with headache altered consciousness with nausea and vomiting Her
MRI brain with venography was done it showed centripetal filling defect- diagnosis?
a- CVT
b- SAH
c- Subdural hematoma
13-A study was conducted by ST5 to compare between 2 mesh techniques to see its effect on
some pt variable (pt wt).it was small sample which test to be used as it was mentioned that
normally distributed data
a- Chi squared test
b- T-test
c- Others?
14-What is risk of simple hyperplasia progressing to cancer
a- 2%
b- 5%
c- 10%
d- 8%
e- 255
15- incidence of early onset GBS disese in Uk in which routine screening is not done?
a- 0.5/1000
b- 2/1000
c- 8/1000
d- 12/1000
e- 5/1000

16-Which organism is most commonly involved in the early onset GBS


a- E COLI
b- Streptococcus agaelactea
c- Peptostreptococus
d- Others
182 | P a g e
17-What is the risk of spontaneous reversion rate after successful ECV
a- < 5%
b- 8%
c- 4%
d- 6%
e- 30%
18-Which drug is used for complicated falciparum malaria
a- IV artesunate
b- Iv quinine
c- Chloroquinine
d- Clindamycin
19-PT was treated successfully for malaria And recovered well gestation not mentioned when she
got malaria what is new natal follow up at birth
a- Placenta for pathology and if positive then screen baby
b- Baby blood films then weekly till 28 days
c- Baby sample for parasite at birth only
d- No need screening
20-PT came with history of fever and abdominal pain. LMP 3 weeks ago and was on
contraception.watery vaginal discharge++ what antibiotics to treat her?
a- Ceftriaxone 500mg stat with doxycyclin and Metro for 14 days
b- Ceftriaxone 250mg IM with doxycyclin 100mg bd 14 days mtz 400mg tds 14 days
c- Oflaxacin combination with metronidazole
21-What is most common type of incontinence prevalent in white women
a- Stress
b- Urge
c- Overflow
d- Mixed
22-What is incidence of stress urinary incontinence
a- 10%
b- 20%
c- 30%
d- 5%
e- 15%
23-An obese?pregnant lady came with history of Acute chest pain in ER(description was of Acute
MI) what ll be the most appropriate 1st line investigation
a- ECG
b- ECHO
c- CHEST X RAY
d- V/Q scan
24-A50 yrs old lady planned for TAH BSO due to fibroid and HMB she is keen to know risk of
hysterectomy and most common complication related to procedure how could u explain to her that
which percentage will be related to this
a- >10%
b- 20%
c- 0.2%
d- 5%
e- 1%
183 | P a g e
25-Risk of maternal mortality related to twin pregnancy
a- 1.5 times
b- 2.5
c- 4
d- 8
e- 12
26-Which ECP can be used in pt on AED (antiepileptic drug)
a- Levonorgesteral 1.5mg
b- Levonorgesteral 3mg
27-What is the risk associated with sodium valproate as compared to other drugs or those not
taking medication in epileptic pt
How much fold increased risk?
congenital malformations autism
a- 10 fold 5 fold
Na valproate b- 10 10
c- 10 5
d- 5 5
28-In epileptic pt which combination of medicine can be used who on enzyme inducing anti
epileptic(EIAED)
a- depot---Mirena---Cu IUD
b- Implant---Mirena---CuIUD
c- OCP---depot---CuIUD
d- Transdermal patch ---depot---cuIUD
29-15 year old girl on 16th day of cycle came with UPSI Of 82 hrs not on contraception (No option
of UPA).
a- CuIUD
b- LNG 1.5MG
c- LNG 3MG
d- Mirena
e- Yuzpee regime
30-What is the rate of uterine perforation after surgical evacuation of missed miscarriage
a- 5/1000
b- 10/1000
c- 8/1000
d- 2.2/1000
31-Most common type of ureteric injury
a- Transection
b- Thermal
c- Ligation
d- Cut
32-Risk of child being affected a pregnant lady carrier of CF husband also carrier her sister has CF
a- 1:4
b- 1:8
c- 1:2
d- 1:6
e- 1:5

184 | P a g e
33-A pt after class section C/O headache going to occipital region not relived with analgesic
Success rate of epidural blood patch
a- 70 -80%
b- 60-90%
c- >90%
d- 50%
34-A 39 yrs old lady with 1st trimester screening test following results
CRL 55
PAPP-A 0.4MOM
NT 1.5
B HCG normal
Next management based on results?
a- Offer her invasive testing
b- Serial growth scan
c- ECHO 2nd trimester
35-PT has BRCA1 at 45 years had surgery what is incidence of her having associated ovarian
cancer Which of the following have the highest risk of having ovarian cancer
a- BRCA1
b- LYNCH
c- HCPPP
36-A pt had emlscs when she was having first baby due to failed forceps after that she had 1 VBAC
now came at 36 wks what will be chance of her success of VBAC
a- 90%
b- 75%
c- 60%
d- 50%
e- 40%
37-A pregnant lady having spinal cord injury at T4 Level came to u in active labour
At 2nd stage bradycardia,suddenly developed pounding headache ,nausea,cold clammy ski, pulse
rate 64/min Sweating (S/S of autonomic dysreflexia) CTG normal
a- Spinal cord muscular spasticiy
b- Autonomic dysreflexia
c- PE
d- AFE
e- Spinal cord injury
38-8 wks history of exposure to rubella seronegative for igG
a- TOP
b- Continue with pregnancy
c- Serial growth scans
39-PT with previous EmLSCS -opted for planned LSCS at 39 wks what is her chance of going in
labour before 39 wks
a- 5%
b- 10%
d- 20%
e- 40%
f- 30%

185 | P a g e
40-Risk of scar rupture in planned VBAC
a- 0.5
b- 0.8
c 1%
d- 2%
e- 2.5%
41-What type of fgm - pt with complete removal of clitoris ,labia minora and complete absence of
labia Majora
a- Type 1
b- Type 2
c- Type 3
d- Type 4
e- None of above
42-Para 2 booked for IOL at 41+2 wks might need cs if IOL failed .what is most powerful indicator of
her successful vaginal delivery
a- Previous vaginal delivery
b- Bishop score
c- Age
d- BMI
43-1st line management of severe PMS
a- OCP
b- CBT
c- SSRI
44-what type in face presentation
a- Suboccipitobregmetic
b- Mentovertical
c- Submentobregmatic
45-Diameter in face presentation
a- 9.5
b- 10.5
c- 11.5
d- 12.5
e- 13.5
46-Diameter in deflexed occipito posterior
a- Occipito frontal
b- Suboccipitofrontal
c- Suboccipitobregmetic
47-PT came with history of Subfertility MBI 36 PCO next advice for her
a- OCP
b- WT loss
c- Metformin
d- OGTT
e IVF

186 | P a g e
48-risk of pp psychosis in women with bipolar disorder
a- 1 in 4
b- 1 in 2
c- 1 in 6
d- 1 in 8
e- 1 in 2
49-Which drug used for osteoporosis associated with osteonecrosis of jaw
a- Bisphosphanate
b- HRT
c- Calcitonin
50-Which autosomal condition is associated with trances arteriosus
Chromosomal deletion 22q11
51-Old lady with hirsutism testosterone 7 DHEAS 7 and 17 ketosteroid normal
a- PCO
b- Adrenal tumour
c- Hyperthecosis
d- Sertoli lyding cell cell tumour

52-%of women having adverse outcome from home birth?

53-Pt need Investigation before having UAE,she had h/o HMB10 cm fibroid with distorted cavity
and counselled for UAE.there was no issue of fertility .which investigation u ll do before UAE as
lower limit of endometrium not clear and had abd scan before
a- MRI
b- TVS
c- CTsacn
d- Pelvic angiography
54-Pt came for preconception counselling having CF,what bstetric risk is high related to her
disease
a- Gestational hypertension
b- Gestational diabetes
55-A pregnant lady at 20 wks came with breast lump what ll be ur Immediate management to
diagnose it
a- MRI
b- mammography
c- PET scan
d- Ultrasound
56-Female murdered by husband after 1 month of delivery
a- Fortious death
b- Unclassified
c- Not a maternal death
d- Late maternal death
57-PT came with history of recurrent cystitis with pain during micturation on cystoscopy picture
they found
a- Interstitial cystitis
b- Schistosomiasis

187 | P a g e
58-What is % of developing serious medical complication with delivery in community in low risk
primigravida
a- 3/1000
b- 4/1000
c- 9/1000
d- 11/1000
59-time of delivery in monoamniotic monochorionic
a- 32 wks
b- 34 wks
c- 30wks
d- 36 wks
60-A pt came for hysterectomy having 8cm fibroid in the broad ligament while operating u must
know anatomy , What ll be the course of ureter
a- Ureter passes above uterine artery
b- Ovarian vessels r crossed by ureter
c- Ureter crosses the bifurcation of common iliac vessels
d- Upper one third of ureter crossed by ovarian vessels
61-What hematological Parameters drop in pregnancy
a- Protein S
b- Protein C
c- antithrombin 3
d- Factor x
e- Factor 7
62-What factor favours the 2 embryo transfer
a- 40-42 yrs
b- >37 yrs
c- Low quality embryos in 1st cycle
63-PT came with History of chronic pelvic pain with cyclical exacerbation. What ll be the chance
that she has endometriosis
a- 10%
2- 20%
3- 30%
4- 40%
5- 50%
64-PT had IVF cycle what is risk factors for OHSS
AMH 99%
65-PT had short vagina need procedure for vault prolapse
a- abdominal sacrocolpopexy
b- SSF
66-A pt planned for VBAC p2 1st delivery was forceps vaginal delivery
Last cs for breech what factors will help her to achieve vaginal delivery
a- Upright position
b- Continuous support during labour
c- Epidural analgesia
d- Early pushing in labour

188 | P a g e
67-A pt received blood transfusion of same group
Registrar noticed that pt name on blood bag was different
It was stopped immediately pt was OK with no symptoms as very small amount administered what
will be ur Immediate step
a- Inform ur Immediate educational supervisor
b- Fill the incident former and enter all details
c- Reassure pt and all other that pt has no problem
d- Never event
68-PT came with history of Lower abdominal pain Pregnant 8wks
On ultrasound found 8cm cyst with solid Nodule with hyperechoic lesion findings consistent with
dermoid cyst.there is ovarian tissue surrounding the cyst ur next management
a- Follow-up up scan after 3 months
b- Laproscopy and cystectomy
c- Laproscopy and Oophrectomy
d- Tumour markers
69-PT has suspected uterine perforation after removal of polyp by 5mm polyp forcep next
management
a- Observe and antibiotics
b- Observe and discharge after 24 hrs
c- Laproscopy
d- Laprotomy
e- Observe /antibiotics/discharge after 24 hrs
70-Pt had lapro and dye test and then discharged after 3 days came with vomiting and abdominal
distension
a- Bowel injury
71-Turner mosaic planning for IVF she needs to be investigated related to fertility what test should
be done
a- Ovarian reserve test
b- Cardiac and ECHO
c- Renal assessment
72-Amenorrhoea 6 wks pain and vaginal discharge on ultrasound the intrauterine gestational sac
2.5mm with no fetal pole and yolk sac what is most likely diagnosis
a- Incomplete miscarriage
b- PUV
c- Anembryonic pregnancy
d- Missed miscarriage
73-A 50 years old lady with BMI have atypical hyperplasia on pipelle biopsy
Now planned for laproscopic TAH BSO what investigations u ll like to do pre operative?
a- Chest x ray
b- KUB
c- MRI Pelvis
d- CT pelvis /Abdomen
e- Ultrasound

189 | P a g e
74-PT came with h/previous stillbirth in active labour what risk factor is more consistent regarding
index pregnancy
a- Previous stillbirth
b- Nulliparity
c- BMI >40
d- Age
e- Single/ no partner
75-P4 36 yrs BMI 25 came with H/O polyhydroamnios 281mm was induced delivered vaginally had
PPH .delivered 4.2kg baby Which Risk factor most important for going into PPH
a- Grand multiparity
b- Induction of labour
c- Age
d- Baby wt >4.2 kg
e- Polyhydroamnios
76-PT with h/o previous DVT at 2nd trimester now came to u for preconception counselling and her
risk of recurrence her BMI high ,p3,35 years ,when to start TP during pregnancy
a- As soon as she ll get pregnant
b- From postpartum 6 wks
c- 10 days post part up
d- From 28 wks
77-A young pt with on ocp since 6 months came with irregular vaginal bleeding next step
a- STI screen
b- Reassure
78-52 yrs old on started with CCT her LMP 12 months back c/o pv spotting after 2 months
a- Reassure
b- Tvs scan urgent referral to gynecology
c- Endometrial sampling
79-A pt with reddish skin rash
Biopsy c3 complement deposition -diagnosis
a- Pemphigoid gestationis
b- Polymorphic eruption of pregnancy
c- Atopic eruption of pregnancy
d- Eczema
80-Mechanism of action of entonox
a- Release of endorphins
b- Direct smooth muscle relaxant
c- Decreased neural activity with decreased nocioceptor impulse
d- Altered Na membrane influx
81-Pt with h/o Parvo virus exposure at 10 wks. at 20 wks she had mild hydrops. next step
a- Reffer to FMU
2- amniocentesis
c- Uterine a Doppler
82-PT with AOB. already tried life style changes next step
a- bladder drill
b- PFMT

190 | P a g e
83-PT with h/o frequency urgency and Nocturia Reffered by GP urine ex normal Next step
a- Bladder diary
b- Pad test
c- Tvs
84-PT with h/o vulval itching HR-HPV +ve Diagnosis
a- Verroccous vulgaris
b- Squamous cell ca
85-PT on citalopram stopped 12 wks before now she can not open with 2 yrs old son sleep and
appetite poor you restarted medicine what is the reason most likely after she was Reffered to u
a- Poor sleep and appetite
b- Better tolerance of pregnancy
c- To reduce risk of pp depression
d- To reduce risk of pp psychosis
86-Statistics about a study was conducted in which some cases of disese and some control taken
and see the outcome the sample was from randomly data system
What type of study is this
a- Case control
b- RCT
c- cohort study
d- Cross sectional study
87-pt with h/o irregular periods has pain and bleeding what is optimal minimal timing of viable iu
pregnancy to appear
a- 30 days
b- 35 days
c- 42days
d- 45 days
88-56 yrs old lady came with 1 year h/o itching vulva .o/E she had distorted labia majora and white
area .Treatment?
a- Hydrocortisone
b- Clobetasol
c- Laser vaporization
d- Vulvectomy
89-an induction of trainee to be done they r large in no what ll be most reliable and fast method to
make their education regarding their knowledge
a- Appraisal
b- Induction of interview
c- Written test
90-Complex ovarian cyst in a pt complaining of cyclical worsening of pain /cpp.glass ground
appearance. treatment?
a- Laproscopic ovarian cystectomy
b- Ablation and excision
c- Ulilateral ovarian cystectomy
d- Laprotomy
e- Oophrectomy
91-Absolute contraindications to UAE
a- acute Pid
b- PT refused blood transfusion
191 | P a g e
c- Bicornuate uterus
d- Big fibroid
92-Major risk factor for SGA in a pt having DM,BMI,35
a- DM
b- Hypertension
93-The most important risk factor for predicting preterm labour ad need serial
ultrasound monitering
a- Previous knife cone biopsy
b- Previous 1st trimester miscarriage
c- Previous preterm delivery
94-A GP called u about a pregnant lady got influenza incubation period of H1N1
a- 2 day before and 2 days after
b- 2 days before and 7 days after
c- 7days before 7 days after

WATTS RECALL 2

192 | P a g e
Ca125

Is small amount of free fluid have something to do or just ignore it

: Tog says even with fluid and thin et ignore

Is it cyst or bleeding ????

: Bleeding

: For cyst... Don't ignore

What to do if cyst persists after 1 year?

: Lap or ignore? Lapro... Not sure

: Lap. No it in gtg. Should remove if persist.

[4:16pm, 2/27/2016] +966 54 168 4046: Better I will post pics

193 | P a g e
[4:16pm, 2/27/2016] +91 97916 19530: In premenopausal

[4:16pm, 2/27/2016] +91 97916 19530: If persists do we do mri,ct?

[4:18pm, 2/27/2016] +966 59 022 8446: Gtg for premenopausal

[4:22pm, 2/27/2016] +966 59 022 8446:


For post menopausal... Usually bilateral

[4:23pm, 2/27/2016] +966 59 022 8446: Dr fouzia what's the answer for 2 BD question

194 | P a g e
[4:45pm, 2/27/2016] +966 54 168 4046:
Plz comment as this is controversial. Women with pmb and normal endometrium with fluid in
cavity. ...we will reassure or will do emb

[4:51pm, 2/27/2016] +91 99107 73346: If pt asymptomatic and ,ET < 4mm, ignore fluid

195 | P a g e
[4:52pm, 2/27/2016] +91 99107 73346: If pt asymptomatic, ET>4mm,fluid present, evaluate

[4:52pm, 2/27/2016] +966 54 168 4046: Ok. .thanks. this was big confusion in recall. If fluid with fever
or other symptoms like pmb then endomtritid can be think of as tog[

[5:06pm, 2/27/2016] +966 59 022 8446: First step is echo then MRI

[5:06pm, 2/27/2016] +966 59 022 8446: But management plan is after MRI

[5:07pm, 2/27/2016] +966 54 168 4046: What about drugs?

B is wrong.

We can't start it as safety in first trimester for teratogenicity is not established. May be asking what
is wrong. ..so the answer will be c
196 | P a g e
[5:19pm, 2/27/2016] +966 54 168 4046:
Next now pt is 8 week with cardiac symptoms. .what next

[5:19pm, 2/27/2016] +966 59 022 8446: Before pregnancy ... If reduced ejection fraction preg is
contra indicated

[5:19pm, 2/27/2016] +966 59 022 8446: But at 8 weeks????

197 | P a g e
[5:28pm, 2/27/2016] +966 54 168 4046:
I think should investigate further as top is not a straight forward option. Also starting chelation is
option but after complete assessment. .

[5:29pm, 2/27/2016] +966 59 022 8446: But MRI is not option over here

[5:30pm, 2/27/2016] +966 54 168 4046: It was a emq so you should know answer before answering.
...step by step. ..

If echo has been done then according to symptoms may need ecg or cardiac mri. .?

[5:31pm, 2/27/2016] +966 54 168 4046: These recall are just a mirror of possibilities. ...we can decide
according to scenario. ...

[5:32pm, 2/27/2016] +92 336 2414031: mri?

[5:33pm, 2/27/2016] +966 59 022 8446: Symptoms

Echo

198 | P a g e
MRI

Chelation

[5:33pm, 2/27/2016] +966 54 168 4046: If no option of ecg and no palpitation in either we have to
treat pt in either as told by gtg so start chelation. .?

[5:33pm, 2/27/2016] +966 54 168 4046: Ok next. .

How you will tell a very rare to layman person. .?

1 in ??? Les then one in 10,000

199 | P a g e
[5:45pm, 2/27/2016] +966 50 868 8519:
Both cocaine & amphetamine can lead to vasoactive problems like abruptio iugr & htn

[5:45pm, 2/27/2016] +966 50 868 8519: Heroin less likely

[5:47pm, 2/27/2016] +966 50 868 8519: But follow gtg cocaine

A 19 year old single lady came to er


with painful ulcers in the genitals involving inside of labia minors also with preceding history of
tonsillitis associated with fever 5 days before. According to her she was Virgin.

What is probable diagnosis

Herpes

Syphilis

Lipchutz ulcer[6:24pm, 2/27/2016] +966 59 022 8446: Herpes

[6:24pm, 2/27/2016] +966 59 022 8446: With high grade fever... Oral and genital lesions

But not sure

200 | P a g e
[6:26pm, 2/27/2016] +92 336 2414031: Lipschuz ulcer

[6:27pm, 2/27/2016] +92 336 2414031: Not a STD

201 | P a g e
[6:39pm, 2/27/2016] +966 54 168 4046:
Post dural puncture occur in ??

[6:40pm, 2/27/2016] +92 336 2414031: 0.5-2.5% puncture

[6:44pm, 2/27/2016] +966 54 168 4046: Great .may be asking postpuncture headache which is 70 to
80 percent

[6:44pm, 2/27/2016] +966 54 168 4046: So answer is d. .agreed.

202 | P a g e
[6:50pm, 2/27/2016] +966 50 868 8519:
Total hysterectomy

[6:51pm, 2/27/2016] +966 56 865 2520: why not to repeat the LLETZ .

[6:51pm, 2/27/2016] +966 54 168 4046: But why not radical. .reference. .? She is 58 years. ?

[6:51pm, 2/27/2016] +966 50 868 8519: LLETZ not appropriate

[6:51pm, 2/27/2016] +966 50 868 8519: If cone biopsy Yes

[6:53pm, 2/27/2016] +966 50 868 8519: Why doing radical ?

Actually no enough informations to base on

[6:53pm, 2/27/2016] +966 54 168 4046: Ok ..I got. Radical is for 1b1..

1b 2

2A right. ?

[6:54pm, 2/27/2016] +966 56 865 2520: right

[6:54pm, 2/27/2016] +966 56 865 2520: no need for such extensive surgery in cin

[6:54pm, 2/27/2016] +966 50 868 8519: Yes no need

203 | P a g e
[6:55pm, 2/27/2016] +92 336 2414031:
Atropine

[6:55pm, 2/27/2016] +966 50 868 8519: Naloxone

[6:55pm, 2/27/2016] +92 336 2414031: Nalaxone

[6:55pm, 2/27/2016] +966 50 868 8519: Pin point morphia

[6:55pm, 2/27/2016] +966 56 865 2520: sign of opiod posioning

Yes .Causes of pinpoint pupil

Opioid

Amphetamine

Pontine hgeYes .Causes of pinpoint pupil

Opioid

Amphetamine

Pontine hge

204 | P a g e
205 | P a g e
[7:14pm, 2/27/2016] +966 54 168 4046:
Dr anjali this emq you were talking about anaemia cause in different scenario

[Yes, but my and r different

[ Ans

[7: PIH , physiological anemia unless HELLp

[: B, agree

In pre-eclampsia what type of anaemia. ..let's ask others also.

[: Hemolytic hellp

: Yes agree

: SCD _ extravascular hemolysis,it was there in choices

: Yes csd hemolytic

: Agree help hemolytic

: Pih imean

[0: in recently diagnosed PIH , if on methyldopa it will be drug induced if it was in the options

[ But regarding pt on dialysis may also be anaemia of chronic disease

[: Here lead in important. If in sickle cell diseases asking type of anaemia is hemolytic but if cause
of anaemia hbpathy. ? ?

[: Autoimmune due to aldomet

[: It was type of anemia

206 | P a g e
[: if preeclampsia , it is hellp .

[: heamolytic in drug induced

[7:22pm, 2/27/2016] +91 97916 19530: It causes direct coombs positive

[7:22pm, 2/27/2016] +966 50 868 8519: Yes because aldomet cause SLE like syndrome

[7:23pm, 2/27/2016] +91 99107 73346: Dr Fouzia u r right , but here pt is not on aldomet

[7:23pm, 2/27/2016] +966 56 865 2520: it is autoimmune heamolytic anemia .

[7:23pm, 2/27/2016] +91 99107 73346: In PIH, normally there is hemoconcentration not anemia,
unless HELLp[7:24pm, 2/27/2016] +91 97916 19530: True

[7:24pm, 2/27/2016] +966 54 168 4046: No I am not talking about dialysis. ..they all agreed for
erythopoietin

[7:24pm, 2/27/2016] +966 50 868 8519: Yes hardly anaemia in PIH

[7:24pm, 2/27/2016] +91 99107 73346: So ans according to me as I thought was physiological

[7:25pm, 2/27/2016] +966 50 803 2485: Admited not options for t pet as 1st or 2nd line

[7:26pm, 2/27/2016] +966 50 803 2485: Autoimmune as one of pet theory[7:27pm, 2/27/2016] +966 56
865 2520: if the question what is the cause then it will be autoimmune , if what is the type it will be
heamolytic , am i right ??

[7:28pm, 2/27/2016] +966 50 868 8519: Yes right dr Afraa

[9: If said pt heavy alcoholic what type of anaemia?

[: Nutrional

[7: Mainly megaloplastic

It is right Zahir broad term but more precised folic acid def[7:33pm, 2/27/2016] +966 50 868 8519:
Yes

[: Folic acid def

[: For dialysis... Why not chronic disease anemia?

207 | P a g e
[: Ok, now I remember anemia of chronic disease, ? Microcytic hypochromic

[1: Erythropoietin deficiency is cause[: yes it is microcytic hypochromic

[7: if hellp it is normocytic normochromic

[7:

Iron

Thallassemia

Sideroblastic

Hypochromic microchtic

[7:

Folic

B12

Alcohol

Macrocytic

Hemolytic not due to thalassemia

Chronic disease

Normocytic normochromic:

Yes in chronic disease it's normochromic anaemia.

: To conclude, PIH, physiological

208 | P a g e
SCA, extravascular hemodialysis

Renal disease , NC/NC

: I think sequestration & destruction by spleen is called extravascular

If in circulation it is intravascular

HELLp haemolytic normoch normocytic

Renal anaemia of chromic disease normoch normocytic

also erythropoetin deficiency

Talassaemia haemolytic but microcytic hypochromic

SCD haemolytic normochromic normocytic

Folic acid & B12 deficiency

Megaloblastic anaemia macrocytic

Iron hypoch microcytic

209 | P a g e
210 | P a g e
[8:05pm, 2/27/2016] +91 99107 73346: Syphilis treatment inj
benzathene pn

Mastitis fluclox

Cross section study

Refer to FMU

[8:05pm, 2/27/2016] +91 99107 73346: GBS, benzyl pn

[8:05pm, 2/27/2016] +966 50 868 8519:

211 | P a g e
[8:13pm, 2/27/2016] +966 59 022 8446:
For prevalence studies... Cross sectional

[8:14pm, 2/27/2016] +966 59 022 8446: In groups and forward.... Case control

And backward cohort

[8:15pm, 2/27/2016] +966 50 868 8519: Case control. Rare diseases

Cohert. Incidence

[8:15pm, 2/27/2016] +966 59 022 8446: Means u Take groups for study.... Retrospective data...cohort

[8:16pm, 2/27/2016] +966 59 022 8446: Difference between incidence and prevalence[8:17pm,
2/27/2016] +966 50 868 8519: Retrospective already the events occured.You just analyse from
records.

Prospective you select the groups controlled& cases & follow themfor certain exposures

[8:19pm, 2/27/2016] +966 50 868 8519: You can simply define incidence as new cases in certain
period

[8:20pm, 2/27/2016] +966 50 868 8519: Prevalence include all cases diagnosed new & old

[8:20pm, 2/27/2016] +966 56 865 2520: right it is gievn in certain time

212 | P a g e
[8:25pm, 2/27/2016] +966 59 022 8446:
Condylomata

[8:25pm, 2/27/2016] +966 50 868 8519: You mean HPV ?

[8:25pm, 2/27/2016] +966 50 868 8519: There is 2 condylomata

[8:26pm, 2/27/2016] +966 50 868 8519: C.lata

&C.accuminata

The 1st is due to syphilis

The 2nd due to HPV

[8:27pm, 2/27/2016] +966 50 868 8519: Accuminata is warty like lesions

[8:27pm, 2/27/2016] +966 59 022 8446: It was not mention in question

[8:28pm, 2/27/2016] +966 50 868 8519: 1ryvSyphilis can cause hard painless nodule or ulcer but?

[8:30pm, 2/27/2016] +966 50 868 8519: I go with syphilis

Because herpes lesions are painful

[8:30pm, 2/27/2016] +966 50 868 8519: & usually vesicles or blisters

[8:31pm, 2/27/2016] +966 50 868 8519: But what relevance of (stable relationship?)

[8:32pm, 2/27/2016] +966 50 258 5374: Ithink it is primary syphylis painless nodule

[8:43pm, 2/27/2016] +966 50 868 8519: Race & location 're important clues in these questions

[8:43pm, 2/27/2016] +966 50 868 8519: Tropical

213 | P a g e
Subtropical

Asian

etc

[9:18pm, 2/27/2016] +966 50 868 8519:


Ok in 1ry syphilis the best is treponema pallidum imobilisation test

Looking under dark ground microscopy for motile organism.scrap from ulcer

[9:19pm, 2/27/2016] +966 50 868 8519: Overall the best test is TP haemoagglutination tet

&Flourescent

214 | P a g e
[9:21pm, 2/27/2016] +966 50 868 8519: VDRL can give false p+ve in case of other spirochetes or
chronic dieseases like TB etc

[9:23pm, 2/27/2016] +966 50 868 8519: Ciprofloxacin & flocloxacin are quinolones

Flucloxacillin is penicillin

[9:25pm, 2/27/2016] +966 50 868 8519: Genital Ulcers

[9:25pm, 2/27/2016] +966 50 868 8519: Ok 1st the race & locationIf pt from tropical or subtropical or
Asian suspect

Syphilis

Lymphogranuloma inguinale

Chancroid[9:29pm, 2/27/2016] +966 50 868 8519: Western countries

Herpes

HPV

[9:29pm, 2/27/2016] +966 50 868 8519: But this is not conclusive

[9:31pm, 2/27/2016] +966 50 868 8519: Sorrrry

Syphilis painless[9:38pm, 2/27/2016] +966 50 868 8519: Chancroid organism is hemophilus ducrei

[9:39pm, 2/27/2016] +966 50 868 8519: Syphilis hard indurated

[9:39pm, 2/27/2016] +966 59 717 4558: I think ragged margin in ducri

[9:39pm, 2/27/2016] +966 50 868 8519: Yes

[9:39pm, 2/27/2016] +966 50 868 8519: Chancroid soft

[9:39pm, 2/27/2016] +966 59 717 4558: yellow base

[9:40pm, 2/27/2016] +966 50 868 8519: Shape alone is not diagnostic

[9:40pm, 2/27/2016] +966 50 868 8519: Take whole picture

215 | P a g e
216 | P a g e
[10:08pm, 2/27/2016] +966 59 022
8446: Minor risk factors for SGA... Gtg

[10:09pm, 2/27/2016] +966 59 022 8446: If 3 or more needs monitoring

[10:10pm, 2/27/2016] +966 54 168 4046: What about aspirin she is 12 week. .primi old age and ivf. ..is
indication for aspirin

[10:12pm, 2/27/2016] +966 50 258 5374: Only 2 risk factors (age40+ IVF)

[10:14pm, 2/27/2016] +966 54 168 4046: No age should be greater than 40 years to be major risk
factor for sga. ..

[10:14pm, 2/27/2016] +966 54 168 4046: So 3 minor risk factor primi

Ivf

Age greater than 35 year .need monitoring

[10:15pm, 2/27/2016] +966 54 168 4046: Aspirin is for risk of pre-eclampsia. .not for risk of sga

[10:16pm, 2/27/2016] +966 54 168 4046: So b or d[10:18pm, 2/27/2016] +91 99107 73346: D

[10:18pm, 2/27/2016] +91 99107 73346: Age, primi,ivf

[10:19pm, 2/27/2016] +966 50 258 5374: Yes u r right d with 3 risk factors

217 | P a g e
Yes... So first go for uterine artery
Doppler them decide for further monitoring

218 | P a g e
Early home monitoring

219 | P a g e
Dvt, stop

PTE, continue

[11:10pm, 2/27/2016] +92 333


0313000: No dey r very different. Dey follow a theory

220 | P a g e
[11:11pm, 2/27/2016] +92 333 0313000: Enviorment or inherited

[11:11pm, 2/27/2016] +92 333 0313000: Iugr before 32. Suffers coz he is early iugr

[11:11pm, 2/27/2016] +92 333 0313000: A genetic cause or anomoly infection is always sought

[11:12pm, 2/27/2016] +92 333 0313000: Guidelines r nothing no bindings but just present we ve
evidence or not.n does they change every year

[11:12pm, 2/27/2016] +92 333 0313000: A baby who is iugr before his 32 the best place is not
utrrusBut here in question they are saying confirmed case of iugr...not sga. ..so I think writer of this
question is thinking in that way. ...so he decided to deliver baby at 33 week due to this because

answer is b.

[11:37pm, 2/27/2016] +966 50 258


5374: D

[11:38pm, 2/27/2016] +92 333 0313000: U know a tie between c n d

221 | P a g e
[11:38pm, 2/27/2016] +92 333 0313000: She has two children

[11:38pm, 2/27/2016] +92 333 0313000: If he abuses dem?

[11:38pm, 2/27/2016] +966 50 868 8519: Agree D

[11:38pm, 2/27/2016] +966 50 258 5374: Inform child protection for the other 2 children

222 | P a g e
[11:43pm, 2/27/2016] +92 333
0313000: We never give aunty to hysteroscopy

[11:43pm, 2/27/2016] +92 333 0313000: Iucd can predispose to infection. In next 3 months but

[11:44pm, 2/27/2016] +92 333 0313000: Yup . Hsg .chlamydia causes blocked tubes

[11:44pm, 2/27/2016] +92 333 0313000: Some say tuberculosis make dem bead like

[11:44pm, 2/27/2016] +966 59 022 8446: No for hysteroscopy

No for laproscopy

[11:44pm, 2/27/2016] +92 333 0313000: But if u disseminate chlamydia in abdomen

[11:44pm, 2/27/2016] +92 333 0313000: It causes fiz[11:44pm, 2/27/2016] +92 333 0313000: It causes
fiz

[11:44pm, 2/27/2016] +92 333 0313000: Hugs

[11:44pm, 2/27/2016] +92 333 0313000: Curts

[11:45pm, 2/27/2016] +92 333 0313000: Nothing for hysteroscopy nothing for lap till u do a blunder

[11:45pm, 2/27/2016] +92 333 0313000: Like opening some bowel[11:46pm, 2/27/2016] +966 50 868
8519: According to SIGN

[11:47pm, 2/27/2016] +966 50 868 8519: Answer is A

[11:48pm, 2/27/2016] +966 50 868 8519: Because they mention if dilated tubes during HSG

antibiotics re recommended

223 | P a g e
[11:52pm, 2/27/2016] +966 50 868
8519: B repair laparoscopically

[11:53pm, 2/27/2016] +966 50 258 5374: A

[11:53pm, 2/27/2016] +966 50 258 5374: Lapartomy and repair

[11:53pm, 2/27/2016] +966 59 022 8446: C

[11:54pm, 2/27/2016] +966 59 022 8446: Small tear if not bleeding? Can we wait and see

[11:54pm, 2/27/2016] +966 59 022 8446: Small puncture[11:55pm, 2/27/2016] +966 50 258 5374: No
bowl not like bladder small tear cause perionitis and sepsis

[11:55pm, 2/27/2016] +966 59 022 8446: It's puncture ... Not tear

[11:56pm, 2/27/2016] +966 59 022 8446: It can bleed heavily if u try to stich

[11:56pm, 2/27/2016] +966 59 022 8446: Might be wrongAnswer is none of above

224 | P a g e
[12:02am, 2/28/2016] +966 50 868
8519: A offer TVT

[12:04am, 2/28/2016] +966 54 168 4046: What is difference between colposuspension and tvt
success rate???

[12:08am, 2/28/2016] +966 54 168 4046: Evidence Base colpo 85 to 90 at 1 year

70 after 5 year

Tvt 90 after 1 year

85 after 5 year[12:08am, 2/28/2016] +966 54 168 4046: These are objectives cure rates which are not
much different

[12:09am, 2/28/2016] +966 50 868 8519: TVT is 1st line

[12:09am, 2/28/2016] +966 54 168 4046: Overall they are telling no difference. ..in dew hurts. ..right

[12:10am, 2/28/2016] +966 50 258 5374: I think Imisunderstand the question

[12:10am, 2/28/2016] +966 59 022 8446: 5 yrs

63

70...

[12:10am, 2/28/2016] +966 59 022 8446: Tvt

Colopo

[12:10am, 2/28/2016] +966 54 168 4046: She already have one procedure. ...so failed previous
surgery. ..can we offer her again same type of surgery where overall no difference

225 | P a g e
[12:11am, 2/28/2016] +966 50 868 8519: Other considerations

226 | P a g e
[12:13am, 2/28/2016] +966 50 868
8519: Yes.do urodynamic if in options

[12:14am, 2/28/2016] +966 54 168 4046: What indication of artifical valve as urethral bulking not in
optionWhat indication of artifical valve as urethral bulking not in option

227 | P a g e
228 | P a g e
[12:32am, 2/28/2016] +966 59 022
8446: IUCD... Best only

[12:34am, 2/28/2016] +966 54 168 4046: Yes you are right because a c d s

Is different

1 2 3 4 they use in wheel of this diagram to category pt

[12:35am, 2/28/2016] +966 54 168 4046: Answer is c

229 | P a g e
230 | P a g e
231 | P a g e
[12:57am, 2/28/2016] +966 59 022
8446: Ist week... EC

2 BD week... Barrie's

3 rd week .. No pill free interval

[12:57am, 2/28/2016] +966 54 168 4046: Yes should in second stage. .

[12:57am, 2/28/2016] +966 59 022 8446: Just before crowning

[12:59am, 2/28/2016] +966 54 168 4046: Pill answer is b. ...because it is assumed that 3rd pill already
taken. .he is asking what to do with pills. ..

[12:59am, 2/28/2016] +966 56 865 2520: Sorry. I have guests . they make me busy with dinner and
these stuf, already tired

Looks that ii missd lots of discussion


thanks to all

vWatts recalls 3

232 | P a g e
[10:24am, 2/28/2016] +91 99107
73346: There was one q about something stony dull to percussion

[10:27am, 2/28/2016] +91 99107 73346: Another about pneumothorax

[10:28am, 2/28/2016] +91 97916 19530: Questions on mental health??

[10:31am, 2/28/2016] +966 54 168 4046: Yes one is like schizophrenic mother ...postnatal mx.

Admit mother or baby together. .?

Let me see topic. .

[10:32am, 2/28/2016] +91 97916 19530: Claustrophobia treatment?

Anxiety disorder treatment

Admission after schizophrenia?

[10:34am, 2/28/2016] +91 97916 19530: Duration of zidovudine treatment to the child after birth in hiv
positive Mother?

233 | P a g e
[10:37am, 2/28/2016] +91 99107 73346: 4 wks

234 | P a g e
235 | P a g e
236 | P a g e
[12:51pm, 2/28/2016] +971 50 382
1209: Tazocin+ metronidazole would cover everything except MRSA

[12:51pm, 2/28/2016] +971 50 382 1209: Tazocin alone is not active against anaerobes

237 | P a g e
[12:52pm, 2/28/2016] +966 54 168 4046: I think clinda with metro more good as covering mrsa
also[12:52pm, 2/28/2016] +966 59 022 8446: No... Look at this pic[12:54pm, 2/28/2016] +971 50 382
1209: Yes

[12:54pm, 2/28/2016] +966 59 022 8446: Metro plus tazocin[12:55pm, 2/28/2016] +966 54 168 4046:
Tazo also covering anaerobes so mean only Tazo enough

[12:55pm, 2/28/2016] +966 59 022 8446: No... Tazovdif not cover anerobes

[12:56pm, 2/28/2016] +966 59 022


8446: Anerobes...

Ampicillin

Co amiclave

Clinda

Erythro

Metro

[12:56pm, 2/28/2016] +966 59 022 8446: All common antibiotics c

[12:56pm, 2/28/2016] +966 59 022 8446: Cover anerobes only

[12:56pm, 2/28/2016] +971 56 760 2703: Sepsis most common strep or stap which tazo is covering

Genta with renal compromise can wait

[12:57pm, 2/28/2016] +966 59 022 8446: Gentacin covers only staph and mrsa

Not GAs or GBS

238 | P a g e
[1:22pm, 2/28/2016] +966 59 022 8446: For MRSA... Vancomycin

[1:22pm, 2/28/2016] +91 97916 19530: Yes

[1:22pm, 2/28/2016] +966 59 022 8446: Or gentacin

[1:22pm, 2/28/2016] +966 59 022 8446: No other option[1:23pm, 2/28/2016] +966 59 022 8446: How
MRSA will clinically manifest

[1:22pm, 2/28/2016] +966 59 022 8446: For MRSA... Vancomycin

[1:22pm, 2/28/2016] +91 97916 19530: Yes

[1:22pm, 2/28/2016] +966 59 022 8446: Or gentacin

[1:22pm, 2/28/2016] +966 59 022 8446: No other option[1:23pm, 2/28/2016] +966 59 022 8446: How
MRSA will clinically manifest[1:23pm, 2/28/2016] +44 7464 366936: Metthicilline resistent steph
aureus

[1:23pm, 2/28/2016] +92 333 0313000: Hmmm. He loves diarrhea vomiting

[1:24pm, 2/28/2016] +92 333 0313000: He loves skin eating

[1:24pm, 2/28/2016] +92 333 0313000: He is a villon

[1:24pm, 2/28/2016] +92 333 0313000: But one question to u

[1:25pm, 2/28/2016] +966 59 022 8446: Is it toxic shock ?

[1:25pm, 2/28/2016] +966 59 022 8446: Same?

[1:25pm, 2/28/2016] +92 333 0313000: Wat is methi silli

[1:25pm, 2/28/2016] +966 59 022 8446:

[1:52pm, 2/28/2016] +92 333 0313000: Clinda is fastest

[1:52pm, 2/28/2016] +966 59 717 4558: d asad guide acc to organism plz

[1:52pm, 2/28/2016] +91 97916 19530: Mastitis or breast abscess?[1:53pm, 2/28/2016] +91 97916
19530: Fluxocloxacillin

239 | P a g e
[1:53pm, 2/28/2016] +966 59 717 4558: like flucloxa is good in mastitits and staph

[1:53pm, 2/28/2016] +966 59 717 4558: meropenum for GAS[1:56pm, 2/28/2016] +966 59 022 8446: So
far for pregnancy.... Sepsis

Tazo / imepenam / meropenem

[1:56pm, 2/28/2016] +92 333 0313000: Fastest is clinda

[1:56pm, 2/28/2016] +92 333 0313000: But it.s spectrum is narrow

[1:56pm, 2/28/2016] +966 59 022 8446: Now what's the difference for postpartum

[1:57pm, 2/28/2016] +92 333 0313000: So it needs a partner[1:56pm, 2/28/2016] +966 59 022 8446:
Now what's the difference for postpartum

[1:57pm, 2/28/2016] +92 333 0313000: So it needs a partner

[1:57pm, 2/28/2016] +966 59 022 8446: Hmmm

[1:57pm, 2/28/2016] +91 99107 73346: Pl start new topic, will post my notes in a short while

[2:07pm, 2/28/2016] +966 54 168 4046:


None of above

[2:08pm, 2/28/2016] +966 59 022 8446: Is This the rite answer???

[2:08pm, 2/28/2016] +966 54 168 4046: Yes.

240 | P a g e
[2:09pm, 2/28/2016] +966 54 168 4046: It's is about precaution during sacrocolpopexy not about

vault prolapse

241 | P a g e
[2:14pm, 2/28/2016] +966 59 022 8446:
Give oxytocin if pt is bleeding

[2:15pm, 2/28/2016] +966 59 022 8446: Means ... B

[2:18pm, 2/28/2016] +966 59 022 8446:


Now it's low grade or high grade

[2:18pm, 2/28/2016] +966 59 022 8446: Mild dyskaryosis is low grade

Moderate to high is high grade.

[2:19pm, 2/28/2016] +966 59 022 8446: Colposcopy is required but forgot duration again

[2:19pm, 2/28/2016] +966 54 168 4046: In new guidelines it's 2 week .in old one it's 4 week .so
answer is b.

242 | P a g e
[2:19pm, 2/28/2016] +92 333 0313000: Moderate stands with high now

Re-excision if:

243 | P a g e
1. Ca

2. cGIN

3. Above 40

Come back later if young less than 50

244 | P a g e
Yes alkylating agents are more toxic.

.c is correct

245 | P a g e
Repeat after 3 m

But if severe olgigo

Rep after 1 month

As in this q

246 | P a g e
247 | P a g e
C is answer

..tomorrow will continue. .. Answer is

248 | P a g e
[11:55pm, 2/28/2016] +966 54 168
4046: Clue here are follicular phase. ..and age .

[11:56pm, 2/28/2016] +966 54 168 4046: In follicular phase not significant. .

[11:56pm, 2/28/2016] +966 54 168 4046: In luteal phase it is significant

Recalls watts

Vulval questions:-

Silver scaly lesion . in the ext. Surface .also psoriases

Seborrhoeic dermatitis affect scalp also& associated with fungus malassesia

Old lady. Ulcerated hard raisrd edge 1 cm daimeter in labia maj: ca vulva

249 | P a g e
Old lady in institute satellite raised pigmented lesions bleeding underwear

what is answer

[: Ca vulva

Raise lesion

Melanoma

VIN

No agreed answer

1. Ca vulva

2. L.scler

3.behcet disease

Yes u are right

One of the two

: Raised ovious lesion ca vulva

[: Lichen sclerosis fused labia in old lady

And Behcet disease mouth ulcer

: Eyesight?

[ ????? This new to me tooo

Kamal what they mean by that

Syphilis?

It is occuring with behcet dis. Due to aneurysim formation.

Mouth ulcer can occure in planus too

[ Vulvo.vaginal gingival syndrome.

It seems like Behjet

[Amira. Where are the ectopic quest

250 | P a g e
[3:09am, 2/26/2016] +966 50 868 8519:
Painful vulval ulcer with reg lymph nodes progressed to sinuses pt from Africa or Asian

[ This Question in kalviani book am i right?!

[: Painful ulcer

Painless ulcer

.........

Pailess hard chancre is syphilis

Painful soft chancre is chancroid caused by haemophilus ducrei[3:13am, 2/26/2016] +966 50 868
8519: Painful soft chancre is chancroid caused by haemophilus ducrei

[3:13am, 2/26/2016] +92 336 2414031: Chancroid

[3:13am, 2/26/2016] +92 336 2414031: Painful

[3:13am, 2/26/2016] +92 336 2414031: With lymphadenitis

Amira for the 1st one repeat the scan after 7 days . g.s seen so we only need to see the viability.

The 2nd is laparscopy

Interstitial cystities

Filling provoke intense pain

Capacity is reduced

Urgent desire in less than 350

No contractions

251 | P a g e
No stress leakage

As far as i remember

[6:56am, 2/26/2016] +92 333 0313000: Anjali is asking about a famous condition written in the
poems.

Can't see

Can't pee

Can't climb on to tree.

Means eyes ,urethera and joints all are involved.

This is Rita. S syndrome.

[6:57am, 2/26/2016] +92 333 0313000: Sorry reiter.s

[7:06am, 2/26/2016] +92 333 0313000: Which country has highest Prevelance of obsterrical
cholestasis in pregnancy.

A. Uk

B.INDIA

C.PAKISTAN

D. CHILD SORRY CHILE

E.JAMAICA

[7:08am, 2/26/2016] +91 99107 73346: Chile[7:38am, 2/26/2016] +92 333 0313000: A young lady from
south America has just landed frim a commercial fkught .she has a midline incision due to some
obsterrical reason.she suddenly got pains and pv bleeding. You auscultated her and fetal heart
rate is normal. ...fine

What is your diagnosis man.

[7:50am, 2/26/2016] +91 99107 73346: What obs reason, fetus still inside?

252 | P a g e
[7:51am, 2/26/2016] +92 336 2414031: Gestational age?

[7:51am, 2/26/2016] +92 336 2414031: APH[8:43am, 2/26/2016] +966 54 168 4046: In CEMACH 2006 to
2008 commonest cause of death in hypertensive disease

1-Intracranial bleed

2-Pulmonary embolism

3-Renal failure???

[8:43am, 2/26/2016] +966 54 168 4046: Mbbrace most common neurological cause of death in
mbbrace 2014. ?

Epilepsy

CV thrombosis

[8:43am, 2/26/2016] +966 54 168 4046: Neonatal trauma bleeding lead to 3 hb after forceps delivery.
Bleeding not limited to sutures. .

Subgleal haemorrhage

Cephalhematoma???

[8:43am, 2/26/2016] +966 54 168 4046: Risk of recurrence of abruption

3 to 6

7 to 10

15 to 20 percent

Risk of transmission of disease of cystic fibrosis in 12 embryos with both parents being carrier.
How much embryo transferred will have disease. .?[8:43am, 2/26/2016] +966 54 168 4046: Incidence
of idiopathic polyhydrominos???

[9:03am, 2/26/2016] +91 99107 73346: Polyhydro 50- 60%

[9:13am, 2/26/2016] +966 54 168 4046: Asthma pt taking short acting b and steroid inhaler still
uncontrolled. ..

Long acting

Steroid

253 | P a g e
Leukotriene antagonist.

I will paste picture from tog article.

Wait. .

[9:13am, 2/26/2016] +61 424 973 543: Long acting

[9:13am, 2/26/2016] +966 53 551 4277: LABA

[9:14am, 2/26/2016] +966 53 619 0296: UAE was done for a 30 yrs old woman. % of those who
will require re-intervention?

[9:15am, 2/26/2016] +92 336 2414031: 25

[9:15am, 2/26/2016] +966 53 619 0296: because of age

[9:15am, 2/26/2016] +966 53 619 0296: if more than 40 10 %

254 | P a g e
[9:37am, 2/26/2016] +966 54 168 4046:
In asthma question was pt already on two inhalers ..now with sob. What next???

[9:37am, 2/26/2016] +966 59 022 8446: Add LABA

[9:38am, 2/26/2016] +92 336 2414031: Check step 4

[9:38am, 2/26/2016] +966 59 022 8446: Then lekotriens

[9:38am, 2/26/2016] +92 336 2414031: Leukotriene

[9:42am, 2/26/2016] +966 54 168 4046: But in discussion they were telling no option of long acting
so theophylline was the answer...It's always start by inhaled short acting b 2 agonist

The add inhaled steroids

Then

LABA

Then

Oral theophylline or lekotriens

255 | P a g e
Last oral steroids So

Laba inhaler should be next step. Agreed. .

Primary infertility case with female having all normal parameters and husband have azospermia.
Otherwise husband looks normal.

Kine filters

Killean

Haemochromatosis

Idiopathic

Answer is absence of vas

Sort of this but option were different. Main focus is husband looks normal so no disease like in
hypogonadisn. ..external genitalia should be also normal. .like that so idiopathic fits in it. Above us
tog 2013 investigation of male infertility

256 | P a g e
[10:11am, 2/26/2016] +966 54 168 4046: Sort of this but option were different. Main focus is husband
looks normal so no disease like in hypogonadisn. ..external genitalia should be also normal. .like
that so idiopathic fits in it. Above us tog 2013 investigation of male infertility

[10:13am, 2/26/2016] +61 424 973 543: Obstruction, excluding vasectomy, accounts for up to 41% of

causes of azoospermia.

TOG.

[10:13am, 2/26/2016] +61 424 973 543: So depends on the question, what to answer

[10:15am, 2/26/2016] +966 54 168 4046: Yes if external genitalia also normal then it's should be
obstructive cause excluding vasectomy. .as in idiopathic they write size of testes decrease. .

[10:16am, 2/26/2016] +61 424 973 543: Yes and also overall idiopathic is more common

Shoulder dystocia recurrence rate how many times higher than normal population. ?[10:57am,
2/26/2016] +966 54 168 4046: What is indication of 2 embryo transfer in ivf pregnancy. ????

[11:00am, 2/26/2016] +966 59 022 8446: Depends upon age

[11:00am, 2/26/2016] +966 59 022 8446: And quality of eggs

[11:01am, 2/26/2016] +966 54 168 4046: Above is stratog. ..it's for one embryo. .I am asking two
embryo indication. It's a recall question.

[11:01am, 2/26/2016] +966 59 022 8446: Ist and 2 BD cycle of ivf.... One

3 RS Alwyas 2

[11:01am, 2/26/2016] +966 59 022 8446: 2nd cycle with no good quality eggs and always in 3 RS
cycle 2 embryo transfer

[11:01am, 2/26/2016] +966 59 022 8446: From40 onwards....3

[11:01am, 2/26/2016] +966 59 022 8446: Sorry 2

[11:02am, 2/26/2016] +966 59 022 8446: Embryo for first cycle

257 | P a g e
[11:04am, 2/26/2016] +966 59 022 8446: Better sumamry

Can any one send me the values of different markers for hcg, AfP, uE for different trisomiesDr
Rabia

258 | P a g e
••The following markers are raised in Down's conceptions:

1- HCG

2- MSAFP

3- uE3

4- Inhibin A

5- PAPP-A

•• In Trisomy 18 the following serum markers are lowered:

A- MSAFP

B- uE3

C- HCG

D- Inhibin A

E- PAPP-A

Answers:

•To memorise Down's markers REMEMBER the H:

- H=High level=Hcg & inHibin.

-Other markers are low(msafp,uE3, & papp A).

- In Trisomy 18

MSAFP,hcg& uE3 are lowered. Remember these 3 markers are the basis for the triple test which
done in the 2nd trimester for later bookers at 15- 20 wks gestation.

259 | P a g e
Dr Rabia

••The following markers are raised in Down's conceptions:

1- HCG

2- MSAFP

3- uE3

4- Inhibin A

5- PAPP-A

•• In Trisomy 18 the following serum markers are lowered:

A- MSAFP

B- uE3

C- HCG

D- Inhibin A

E- PAPP-A

Answers:

•To memorise Down's markers REMEMBER the H:

260 | P a g e
- H=High level=Hcg & inHibin.

-Other markers are low(msafp,uE3, & papp A).

- In Trisomy 18 MSAFP,hcg& uE3 are lowered. Remember these 3 markers are the basis for the
triple test which done in the 2nd trimester for later bookers at 15- 20 wks gestation.

[12:02pm, 2/26/2016] +966 59 022


8446: Combined test is recommended in uk

[12:03pm, 2/26/2016] +966 50 868 8519: Yes combined test is 1st choice

[12:04pm, 2/26/2016] +966 50 868 8519: The figures re right

[12:04pm, 2/26/2016] +966 50 868 8519: Just add inhibin A which is high in T21

[12:05pm, 2/26/2016] +966 59 022 8446: And Papp A is low

[12:05pm, 2/26/2016] +966 50 868 8519: Yes

261 | P a g e
[12:05pm, 2/26/2016] +966 59 022 8446: Papp is always low with chromosome pblmShoulder
dystocia incidence is 0.58-0.7%

Recurrence 10 times higher than baseline

=10*0.58-0.7

It will come 5.8-7%

It is also mentioned 1-25%

All in the gtg

[12:02pm, 2/26/2016] +966 59 022 8446: Combined test is recommended in uk

[12:03pm, 2/26/2016] +966 50 868 8519: Yes combined test is 1st choice

[12:04pm, 2/26/2016] +966 50 868 8519: The figures re right

[12:04pm, 2/26/2016] +966 50 868 8519: Just add inhibin A which is high in T21

[12:05pm, 2/26/2016] +966 59 022 8446: And Papp A is low

[12:05pm, 2/26/2016] +966 50 868 8519: Yes

[12:05pm, 2/26/2016] +966 59 022 8446: Papp is always low with chromosome pblm[12:40pm,
2/26/2016] +966 54 168 4046: Combined test from 10 till 14 plus 1 week and quadruple from 14 plus
2 days

[12:41pm, 2/26/2016] +966 54 168 4046: Dr rabia it's antenatal nice 2012. .they remove triple. .also
change in starting and closing

Combined 10 to 14 plus 1 day

Quadruple 14 plus 2 to 20

[12:43pm, 2/26/2016] +966 59 022 8446: Ohhh... So?

Only 2 tests in uk

262 | P a g e
Combined and quadruple

[4:44pm, 2/26/2016] +966 54 168 4046: What are the chance of identifying ureteric injury during
laparoscopy and what is the

Risk of ureteric injury while doing lap for endometriosis. ..

[4:45pm, 2/26/2016] +91 99083 84860: One third

[4:47pm, 2/26/2016] +91 99083 84860: Ureteric injuries recognised during lap gynaec surgeries

[4:48pm, 2/26/2016] +91 99083 84860: 1-2% ?

[4:54pm, 2/26/2016] +966 54 168 4046: Answer must be in lap injuries. Tog articles. ..

[5:00pm, 2/26/2016] +91 99083 84860: Overall for lap gynaec surgeries 1-2%

For lap surgery in deep infiltrating endometriosis 21%

263 | P a g e
Recalls summary:

8 weeks: borderline and mild

4 weeks: moderate and severe

2 weeks: invasive and glandularDr asad if in untreated CIN1 neg cytology at 1 yr ...routine recall or
cytology reapt at 1 yr more if neg then routine recall

Development of squamous cell carcinoma (actual risk <5% lichen scleroses)

264 | P a g e
[6:35pm, 2/26/2016] +60 12-325 9818:
B

[6:36pm, 2/26/2016] +91 93456 49200: True less than 3.5 normal

More than 3.5 abnormalSee in that preterm new things...

1. Prophylactic cerclage from 16 till 34 ...in gtg 12 to 14 wks... and optional is progesterone.

2. Progesterone if casually detected short cervix...but now I doubt that the results of PROMISE trial
are out.

3. Erythro 1st line 2nd oral penicillin. ..its okay nt contrasting... in gtg no clearcut option... clinda is
an option fr IAP in GBS.

4. Rescue cerclage till 27+6 (from 16wks) nt like 26 as in gtg...everything here is from 16 weeks.

5. Suspected PTL nd less than 30 weeks ___

Nifedipine...consider if between 24 to 25+6 weeks and

.....offer if 26 till 33+6 weeks.

265 | P a g e
6. Suspected PTL from 30 weeks ....

Diagnose it by TVS ..if Cx less =15mm

(^15mm go home)

If nt tvs then FFN

(^50 abnormal...if till 50 ffn go home)

Ffn plus tvs not 2 be used together...save nhs money

[6:47pm, 2/26/2016] +249 11 545 7595: Follow up post abortion is not recommended if the
termination is successful and confirmed. It is advised if there are concerns about completeness of
the TOP...eg TOP in women with multiple uterine fibroid

[6:47pm, 2/26/2016] +249 11 545 7595: U r right D is the answeNuchal fold thickness is a parameter
that is measured in a second trimester scan (at ~18-22 weeks) and it should not be confused with
nuchal translucency (which is measured in the first trimester).

Pathology

266 | P a g e
The proposed aetiology of increased nuchal thickness is as result of congenital heart disease and
lymphatic obstruction.

Associations

aneuploidy

the trisomies

Turner syndrome

congenital heart disease

[6:48pm, 2/26/2016] +91 93456 49200: Amira in quest 55

They jus mention >5 is abn

So its true

Tats y A is answer

[6:48pm, 2/26/2016] +249 11 545 7595:

267 | P a g e
I 've v bad internet that I could not follow you for many hours.Really it 's so depressive.

Pictures 're not downloading

I 've v bad internet that I could not follow you for many hours.Really it 's so depressive.

Pictures 're not downloading

268 | P a g e
269 | P a g e
270 | P a g e
Delay smear till 3 months post
parturm.

D is the correct answer

271 | P a g e
[8:04pm, 2/26/2016] +249 11 545 7595:
no 4 A

[8:04pm, 2/26/2016] +249 11 545 7595: Aspirin 300 mg

GYNE
statistics: [some of them came in obs]
• EMQ :What type of test should be used:

Options: Almost all types of tests were given


Questions:
1. We want to find the relationship between Um.A doppler & pre-eclamsia, after other
factors like age, parity, BMI are taken into consideration. If two groups are similar in
age , etc u use T-test, but if he said we take variants or adjust for co-factors eg PG in
consideration analysis of co-variance…..Read Tog article on statistics.

2. To find the relationship between maternal BMI & fetal birth wt. this example is written
in tog oct2013 Answer is Pearson correlation
3. To look for the different outcomes of pregnancy in relation to BMI.
For multiple factors wt one dependent variable logistic regression
• Name of the study:
4. To measure the outcome of breech delivery. Women were assigned into 2 groups:
vaginal & c/s ; & followed up. Two groups RCT intention to treat analysis (busy spr)

272 | P a g e
Options:
oncology:
• EMQ: Counselling about 5 rys survival rate

Options : Different figures


Qeustions: (May be changed next exams)
5. Stage 2b cx 58% I think options had 60% important as management changes
worsening of stage suregery cant be used so important (source: read choices NHS
and busy spr statistics)

6. 1a ovary (i think it was specifically mucinous). 94% options were….85% read NHS
choices
7. 1b endometrium 75% this stage is imp as more than half of myometrium invaded we
need radio survival rate changes significantly (read busy spr notes epidemiology
and sats of ca ovary and NHS choices)
• SBA:
8. An old woman with ?1cm mass in clitoris. What should be the next step to reach a
diagnosis?

Options included exisional biopsy/incisional from centre surface/incisional from


margin/wide local exision + LND , keye punch biopsy
Source guideline on vulva : keye punch biopsy mentioned in strat OG
Incisional biopsy from margin is written in guideline, u have to take edge and normal
and abnormal region never excisional
9. A woman undergone evac of delayed miscarrige. Histopath. --> Complete mole. What
is the risk of developing chorio carcinoma? 2-3% written in epidemiology and stats

Options: 1:20, 1:40 , 1:60 , 1:80 , 1:100, 1:200[15% is invasive mole but he is asking
chorioca ]
10. what is the % of bilaterality of dermoid. 15% (past papers and green top?) always
choose upper one as books give 10-15% so choose 15%

Options: I think started with 5%,10%, 15% then bigger figures; 15% up to 40%.

273 | P a g e
11. Management option after treatment[LLETZ] for CIN 3 with biopsy showing unclear
margins? Repeat LLETZ

[? test of cure + smear after 6 m]


12. Management option after treatment[LLETZ], age 52, biopsy showed CIN1+cGIN
reaching excitional margins. CIN 3 if unclear margin repeat cone biopsy
Options: cone biopsy, TAH, Radical hysterectomy,& others i can’t remember.
Ethics/clinical goverannce
• EMQ: Options were many. We recalled these:
Woman wtitten consent
Woman to sign consent & assessing that she understands what she's consenting to
Girl or woman verbal consent
Can’t proceed as the consent requirements not complete
Encourage girl to tell parents (Gillick/frasers’s were not in the options)

Questions:
13. Emergency c/s, mother refuse to sign consent. Father insist for c/s
Mother’s consent is more important Woman to sign consent & assessing that she
understands what she's consenting to

14. A 15 yr girl came with boy friend (15 yrs) for TOP [GA ?17 wks] & they look
aware/understanding. Her consent is important as 15 and fraser competent, no
evidence of abuse as boy just 17, first step is to encourage to tell her parents, if she
refuses go to next option take patients consent
15. A 14 yrs girl came with partner for ERPOC.from whom will you take consent take
consent from girl
encourage to tell parents is first step in Fraser competent (but doesn’t mean u will take
consent from mother) and in this case its missed not TOP
Options:

274 | P a g e
• EMQ:

Options: Too many. Unfortunately can’t recall most of them.


Questions:
16. I think a patient died & the coroner request the copy pt medical records. Option list
included full access, restricted access & ?deny access , advice from legal advisor of
your hospital , were in the option list. Government legal investigator for case of death
is coroner…… you will give notes u cannot refuse, u don’t have the right. U chose u
will give copy of file no need to give unrestricted access to notes as he only asked
for copy of notes. If in case of a death we call coroner first to see scene and if he
allows then we can remove tubings etc.
17. A woman who underwent TVT 3 yrs ago now have symptoms recurred & she asks for
her medical records. [full or restricted access to patient records, ask legal advisor of
hospital for help??] u give copy to patient as its pts right. (according to patient ‘s
right act) ( check patient information ) u have to provide copy after 40 days if 40 days
have passed since procedure so u need time to find records and make a copy but if
recent can be given earlier
18. A woman diagnosed with severe dyskaryosis, colposcopy recommended. She moved
house & no letter reached her to come back to follow up ( can't remember options). I
think later on she came with cancer. (failure of pt to recall for colpo is serious and
requires an incident report review and the person is responsibility is of GP and is a
clinical governance issue so has to be informed to clinical governance…. Next step
is to carry out root cause analysis , I the question mentions that incident review was
done then we will do root analysis…. If question didn’t mention review then we will
do report to governance …….) read safe actions in cervical screening program
document GP is responsible and root analysis will be done to prevent this in future
by clinical governance
Options were : incident report review , report in national governance, pt safety events, to
inform responsibility disclaimer, root cause analysis
19. doctor was carrying a study has took the study papers home [papers with private
patient info]. The study papers were stolen. (what u are going to do?) breach of
confidentiality of patient…… police needs to be involved but the hospital will do that
you will not contact police. You cannot remove patient’s notes from hospital even for
an audit you do in hospital u can’t remove notes from hospital at all. U cannot even
access computer without password u can’t even put USB … u will inform clinical
director of hospital ….. who will tell legal advisor and then they will inform police
……it’s a clinical governess issue so root cause analysis done to make sure it
doesn’t happen again .

There were options like “report never event”, “report to information handling
committee” risk management committee , inform GMC

275 | P a g e
20. A study carried out to compare suture material used &seniority of surgeon in
suturing 3rd 4th degree perineum tear & compare it against rcog guideline. Audit
• EMQ: Unexpected pathology during surgery.

Options:
Abandon procedure.
Take biopsy from X & abandon procedure
Remove X from from Y
Other options like -i think- Remove X, Remove X &Y, Contact next of kin........
21. A young girl who was entered as acute appendicitis. Appendix was found normal but
a 3 cm dermoid cyst[X] was found on Rt ovary[Y]. pt signed for appendicectomy and
laparotomy. complicated dermoid cyst as pt had acute abdomen so has to remove X
from Y.if option included continued operation as patient consented if appendix not
mentioned to be normal and accidently discovered dermoid, u will not operate as not
consented for
22. A 40 yrs woman during lap assisted vag hysterectomy and had a previous history of
left ovary removed was found to have a dermoid cyst[X] on Rt ovary[Y], both
attached to pelvic side wall by adhesions . do TAH as planned but do nothing else
leave cyst in so answer is continue operation as consent don’t remove dermoid.

• EMQ
23. A woman came at 41 wks GA. You offered her IOL but she denied. I would offer her
sweeping of membranes if not in ption then go for pt wishes
Options: u still do induction, go with her wishes

[?give her patient inforamtion leaflet]


fertility/andrology
24. Couples, 1ry infertility for 30 months i think, all tests normal. This Q came as SBA &
EMQ Options: CC, IVF, advise to try naturally for another 6 m,.......
Fertility guideline says in unexplained 2yrs u go for IVF

26. Male previously fertile. Now azoospermic. Low FSH & testosterone
Anabolic steroids ( source TOG article)

276 | P a g e
options : klienfilter, anabolic steroids, Kallman’s

27. A woman with infertility, workup showed blocked tubes with hydrosalpings.
Do salpingectomy first or tubal connection blocked and then IVF(infertility guideline)

[?Salpingectomy followed by IVF. Also came as SBA & as a part of an EMQ].

29. From the following figures, what hormonal profile is consistent with WHO class 3
ovarian disorders?
HighFSH , low E?

[High FSH, high LH , low E, normal PRL]


30. A 19-year-old woman was seen in the gynaecology clinic with a history of excessive
growth of facial hair, needing to wax every 2-3 weeks. Her menstrual periods last 3-4
days every 3-4 months. There is no change in her voice. Her BMI is 28 kg/m2.
Examination shows Ferriman-Gallwey grade 2-3 hirsutism over chest and abdomen.
A pelvic ultrasound showed no abnormality. Her day two hormone tests showed LH
level 7.4IU/L, FSH level 5.2IU/L, serum testosterone level 2.3nmol/l, SHBG 24 nmol/L.

What is the most likely diagnosis?


PCO as 2 Rotterdam criteria biochemical and clinical hyperandrogenism,and
oligomennorhea. LH: FSH ratio reversed, test <5
PCO, Idiopathic, Cushing, Androgen producing tumour
*This is a repeat question, but in the previous exam they put a cycle pattern of 7-8
days every 24-35 days. means he canceled oligomenorrhea so not PCO so idiopathic
hirsutism ( idiopathic).
31. A woman came with hirsutism + virilization. Testosterone 7. No DHEA OR OTHER
INVEST were given, but mentioned to have central obesity
Cushing coz test >7 think of ovarian or adrenal tumor

[?cushing]
32. A woman came with hirsutism, irregular periods, LH was higher than FSH.
Testosterone was ? 6.5 . They asked about next test to help reach a diagnosis.

277 | P a g e
ultrasound

[Options included U/S , TFT, DHEA, 17(OH)P, ? dexameth. Suppression test ]


• EMQ: Choose the most likely diagnosis.

Options: can’t remember well but include


OHSS. (or manage OHSS according to unit protocol)
Ectopic / Heterotropic [each as a separate option]
Acute appendicitis
Torsion / Ruptured cyst [each as a separate option]
Bowel perforation
 Incomplete miscarriage
Laparotomy laparoscopy (?or lap for detorsion)/, Expectant mng,.....
33. Surrogate woman, came from Spain [or somewhere] after replacing 2 embryos , came
with severe right iliac Fossa pain. Mild pv bleeding. Tachycardia and hypotension. I
think Ultrasound showed single gestational sac
Acute appendicitis if pain was on LIF it would be heterotropic but would show on scan
34. A woman 2 days after egg collection (large number collected,? 25), came with severe LIF pain,
vomiting, lightheadedness. Egg collection done after two days travelled to UK and woke up in
morning with severe iliac fossa pain and felt lighted and abdominal pain torsion as localized pain
(LIF), in perforation pain would not be localized, vomiting is a common symptom in torsion
U/S showed ?“spider web”.
No bowel perforation as doesn’t present this early it has to be something in ovary but
spider web shows pseudocysts

[? OHSS/ some said Torsion]


Pt had left salpingectomy before marriage for ectopic, had sperm donation, slight vag
bleeding, no free fluid. Intrauterine empty sac
35. I am not sure but i think there was another acute presentation, with U/S showing ?
doughnut sign or something
Ectopic as doughnut sign is of ectopics
• EMQ or SBA[not sure] : Male factor infertility: investigations?

278 | P a g e
36. A healthy male with slightly smaller testes scenario was given where semen analysis
was normal [lower normal], the only abnormality mentioned was dysuria or turbid
urine after coitus.what will be invg?small to avg testes, mild gynecomastia
Retrograde ejaculation as turbid urine investigation would be urine analysis (will show
sperms) ,if cause uncontrolled DM treat it otherwise no specific treatment and o
problem
37. Another scenario, the man has fatigability & erection problems.(i think the options
included something about “electric wave analysis for ejaculate”. Deeply sorry for the
poor recall.) Previous 2 normal reports two kids test is 4, and now feels tired and
cannot remain erected FSH 0.1, LH0.3, Serum test 0.4
Answer serum prolactin
Read TOG article on male infertility
38. A third scenario, i think with oligospermia. He has mild gynecomastia. healthy male
motility 40%
Whenever there is an abnormality in semen analysis do after 3 months but if azospermia
repeat sooner so answer repeat semen analysis

NERVE /vascular .INJ


• EMQ: Identify the most likely injured nerve (TOG article nerve injuries)
39. After forceps, woman came later with urinary & fecal incontinence

[pudendal neuropathy]
40. After VD, woman can't flex knee or extend hip+ absent knee jerk

[? Femoral]
41. After surgery in lithotomy, a woman has foot drop & sensory loss in lateral surface of
leg. [Common peroneal wasn't in the options, but Lumbar-peroneal neuropathy was
there.]

• SBA

279 | P a g e
42. During TVT, the surgeon accedently hit the inferior epigastric artery ( &NOT
OBTURATOR). From wich artery does it arise?

[ext. iliac ]
urodynamics:
• EMQ: Choose the appropriate Mng:

Can't remember much but options included:


PFMT
Bladder retraining
PFMT & Bladder retraining
Bladder diary
Clean intermittent self cath.
Many options including specific drugs & surgeries. I don’t remember if urodynamics
were there or not.
43. A woman presents with urgency, freq, nocturia & STRES Inc.

[?diary]
44. Another woman who i think her symptoms were pure stress UI.

[PFMT]
45. A woman with multiple sclerosis + voiding diff

[?CISC] self ctheterisation


• EMQ: urodynamic interpretation: what is the likely diagnosis

Options included DO, USI, Interstitial cystis, &-i think- chronic cystitis
46. Residual 70. 1st & max desire both reduced. Normal voiding velocity. I think pressure
rise during void was ? Normal. If pressure rise during void and flow dec obstructive
problem voiding difficulty,…….if another Q if rise in pressure on filling….for non
comliance
47. There was another Q. Can’t remember

280 | P a g e
• SBA
48. woman with recurrent UTI + Pain : cystoscopy: --> multiple small hemorrhagic areas
Interstitial cystitis (TOG)

Options: Interstitial cystitis, transitional cell ca, superficial cystitis or something.


49. Of women with OAB, what % will also have urge incontinence?
STRAT OG has answer
In one Q capacity ecreased , flow rate decreased
In another question same as above voiding difficulties multiple sclerosis, self
ctheterisation is treatment ( TOG article)

Infections
• EMQ: What is the diagnosis

Options: Primary syphilis, 2ndry syphilis, Chancroid, Vulval candidiasis, Lichen


planus/sclerosis, Vulval cancer, Melanoma, others.
50. A young lady, came from trip to china. C/O vulval lesion. O/E : 2 symmetrical painless
ulcers with raised edges on labia majora. Chancroid , Strat Og says its painless in
women

[we argued about chancroid-kissing ulcers. But painless?]


51. An elderly 82 yrs woman at nursery home c/o blood staining of underwares. NO
SYMPTOMS. O/E small pigmented lesion with rolled edge on Lt labium majus, with
small satellite lesions around. VIN has satellite lesions, age
82,precancerous…..melanoma or VIN

[?candidiasis,? cancer]
52. A young lady came from somewhere, c/o mass, o/e: sessile growth on perineum ,
painless, no itching. wart

[wart]
Others

281 | P a g e
53. Young lady with PMS mainly psychlogical (& her partner prosecutig her for assaulting
him). Her GP advised exercise or something. First step is to diagnose PMS…….by
daily record of severity of symptoms if management asked…….

Options included all steps of managing PMS, & also referal to social worker
54. what is the % of U perforation after evac (?of missed ab?) 5:1000 ( consent for
surgical evacuation has this incidence)

[? 5:1000]
55. what is the absolute C/I to UAE from the following?

This Q came twice. & the only option came twice was adenomyosis....HINT OR
DISTRACTOR? Other options wrere age below 35 , IUCD in situ, active pelvic
infection, Asymptomatic fibroid,uterine anamolies. [Pregnany was NOT there]. Acute
PID, if pt does not accept hysterectomy as known complication , asymptomatic
fibroid are absolute C/I
56. A woman on stable relationship for past 23 years & on IUCD for last 12 years now she
had abd. Pain.U/S showed a mass beside right ovary. Sulphur granules intraop.
actinomycosis

[?Actinomycetoma?, can’t remember the other options]


57. What is the 1 st line managment of menorrhagia? 30 yrs old doesn’t want to prevent
pregnancy so option would be non-hormonal treatment tranexmic acid

This Q came twice. The difference was in age &-in the first Q, they specifically
referred to NICE. IUS if mentions NICE
[We didn’t think of a difference: both IUS?]
58. A long history but at the end: ovarian tumour + pathology report of Call hexener
bodies.

[granulosa cell tumour]


• EMQ: What sign do you expect to find. Options:
bilateral adnexal masses
Longitudenal vaginal septum.
Retroverted uterus

282 | P a g e
Thickenings in POD [?or tuero sacral lig]
Urethral caurencle
Other options i can’t remember
59. A young girl with recurrent UTI. Now also has dsyparunia (recent TOG about uretheral
diverticulum)

[Most said urethral caruncle no as that’s asymptomatic and incidentally discovered


in post menopausal. Why not long.septum?]
60. A PCO woman taking clomifine .

[?adnexal masses] as usually ovary not palpable unless hyperstimulation


61. One with most predictive diagnosis of endometriosis rectal vag wall painful
indurations

[? R/V uterus, thickenings, tender uteroscaral ligament(could be PID), rectovaginal


wall painful indurations, adenexal mass(could be any cause)]
• SBA
62. A 36 yrs old lady with previous term healthy baby, now had 3 recurrent miscarriages
at 6,7,8 wks. What invest. are you going to do? antiLupus antibodies

Options : APLa, thrombopillia screen, pelvic U/S for anatomy, other 2 options i can’t
remember but did’t include karyo as far as i remember.
nd
63. A woman with PH of two 2 TM miscarriages, presented at 18 wks,no contractions
but o/e you find membranes bulging into vagina

[? Admission + cerclage]
64. EMQ: sexually active, C/O frothy-greenish V. discharge [?TV] trichomonas vaginalis
65. EMQ: sexually active, C/O malodorous fishy discharge. [?Gardnella] or BV
66. What is the recurrence rate of bacterial vaginosis? 50 % in one year
67. There was a question, where an obese diabetic lady on metformin undergone TAH for
HMB. After...6.hrs post surgery,c/o abd pain and increasing dysnea you were called
by the concerned nurse. The patient’s MOEWS chart was given. We were asked about
the underlying pathology.drain had 750 ml blood and in urine analysis no ketones.
Options icnluded sepsis, DKA, Hypovolemia, Hypoxia. Blood press 160/100.Options

283 | P a g e
were pul embolism , sepsis,overdose morphine) pulse 120 (morphine 3 doses in 3
hrs), temp normal, sat 100%(pul embolism unlikely), BP inc so excludes
hypovolemia, DKA excluded as no ketones on urine D/R…….
68. There was a question about a woman who did an HSG, can’t remember details but i
think came later with features of PID. There was an option of “Manage PID according
to protocol” .
Left salp previously and now presented wt LIF mass Egg collection done after two days
travelled to UK and woke up in morning with severe iliac fossa pain and felt lighted
and abdominal pain
and pain
69. An 18 yrs girl came with 1ry A. Normal 2ndry sexual development. Very short
vagina{?the ygave the length}. Ovaries present on scan.

[?MRKH]
70. A 16 yrs girl came with 1ry A. Heavy exercise. BMI 18. FSH 34. Primary ovarian failure(
not wt related as FSH would be low wt exercise)

[Options included: Wt related A, primary POF, wait for another 2 yrs]


71. What is the emergency CC option for an epileptic lady who denied IUCD?

[?levoenlle 3g]
72. The concentration/efficacy of the following drug is reduced by taking COCs

Options included the common AEDs


[?lamotrigine]
73. What is the commonest type of ureteric inj in laparoscopic surgery?

[?transection]
• EMQ : Antibiotics prophylaxis choice: ALL CASES HAS SOME SORT OF CARDIAC DISEASE

Options included many endocarditis prophylaxis, Erythromycin/doxycycline pre & post


surgery. NO cefuroxime or Augmentin as single doses , but there was cefazolin single
dose
74. TAH + Ant[?or may be post] repair.

284 | P a g e
75. Diagnostic Hysteroscopy + cystoscopy.
76. There was a question about a lady who came 5 wks post IVF conception with mild
bleeding. U/S showed intrauterine empty GS of 21 mm. I think they asked about the
diagnosis rather than next step.
OBS
• EMQ: for the following scenarios, what is the risk of maternal morbidity/ mortality
(low/significant/ high) & the risk of the baby having cong.heart disease(<10%, 15-25% , 50
%: )
77. mother with minor anomaly, with previous baby with CHD.
78. mother with pul.HTN
79. mother with PH of PPCM. In this pregnancy the echo shows structurally normal heart
with mild systolic dysfunction.
• EMQ: PPH: Options were many but included:
Bimanual compress
Carpoprost
Ergometrin[one option was I/M, other was slow I/V]
Synto [one option was bolus, other as infusion]
Explore lower genital tract under good light
Explore in theatre. + many other options
80. Heavy bleeding, HD compromise, placenta complete, catheter in. Resuscitate WAS
NOT AN OPTION.
81. Moderate bleeding after delivering 4.1 kg baby .placenta complete. Uterus well
contracted.
82. Heavy bleeding. Placenta complete. catheter+ bimanual +oxytocin already given.
Patient has mild asthma.
• EMQ: what is the single most imp. Parameter you want to know:

Options were BP, CBC, RBG,Coag profile, GCS, O2 sat.


83. You are about to undertake hysterectomy for massive I.O bleeding. The anesthetic
team are managing to keep stable BP.
84. After delivery, a lady developed a grand mal fit.

285 | P a g e
85. After c/s , pt became hypotensive. No vaginal bleeding but ooze from wound.
• EMQ: Most probable underlying cause:

Options included: anaphylaxis, Amniotic fluid embolism, Air& Fat embolism, Malignant
hyperthermia, tension pneumo thorax.
86. During c/s, pt developed generalized rash, hypotension.
87. During c/s , pt became short of breath, hypotensive, low O2 sat( & also CO2)
• EMQ: What is underlying cause:
88. before emergency c/s, pt complained of headache + visual disturb. BP ?160/110 i
think. Smooth section & viable fetus but pt didn't awake from anesthesia & died.
Autopsy: IVH + conation of ? Cerebellar vermis. In the options, there was "acute
hydrocephalus due to herniation of medulla" or something like that.
89. An obese bus driver suddenly collapsed & died
• EMQ: 2nd stage & heart dis. The classification given was not NYHA ,but that mentioned in the
FSRH GL (CC & cardiac dis).

Options: Aim for VD, give oxytocin, deliver with forceps, deliver with ventouse, C. section
90. Class 2, symptomless, in 2nd stage but pushing well, head +3.
91. 2nd woman was class 3 ,in 2nd stage, head + 2. I think she has some symptoms.
DERMA:
92. Woman with purpuric rash. Cant' remember GA. rash disappeared but woman still
itchy. LFT:( normal bilirubin, normal AST, high ALP , slightly low albumin[32 instead
of 35]. What further testing?

Options: serum bile acids, virology screen, caog.profile, repeat LFT, no further test
needed.
93. Mother with purpuric rash involving abd. straie. What feature is with good prognosis?
(?Sparing umbilicus,....)
94. Mother with purpuric rash. Biopsy showed immune complex deposition. What’s the
likely diagnosis?

Options: they gave the D/D of purpuric rash in pregnancy.


INFECTIONS:

286 | P a g e
95. A pregnant lady developed chickenpox rash. Phoned on same day.

[? Acyclovir].
96. A pregnant lady developed chickenpox rash, phoned 3 days later .

[? avoid contact with preg ladies OR no further actions? OR U/S after 5 wks?].
97. A pregnant lady came with rash + joint pain. RETESTING( that how it was written)
showed her infected with parvo- not rubella. GA was 2nd TM . What action?

[? U/S follow up]


98. There was a question about congenital malaria. A woman has malaria in pregnancy.
What should you do?

Options after delivery: 1)Blood film for baby at birth then wkly for 4 wks 2) placental
films; if neg for malaria, no further action.
Labour
• EMQ: IPC

Options: Re-assess after 15m/30m/1h/2h/4h [each as a separate option], Forceps delivery,


Ventouse [2 cup types given]delivery, C/S, Reassess in theatre for forceps or c/s,...
nd
99. PG, effective epidural, good progress, 1h passive 2 stage & then actively pushing
for 90m. head at spines & CTG normal.
nd
100.Parous lady, in 2 stage for 90m, now pushing for 30m. Head ? OL at spines.
nd
101.A woman after 20 m in 2 stage tells you that she can’t push more & demadns c/s.
She allows you to examine her. Head? OP at spines.
Headache
102.What is the most common finding in CVT?

[? headache]
103.Preconeption counseling of pt with migraine. The options were combinations of ↑,
↓,unchanged migraine attacks & ↓, ↑ or unchanged risk of pre-eclampsia.

[PE ↑. Attacks ?unchanged]


287 | P a g e
104.A question about headache where the lady described it as the worst ever.

[?SAH].
105.Headache developing post partum + focal neurological signs. MRI showed filling
defect

[?CVT, others said Posterior reversible encephalopathy syndrome]


OTHERS:
106.what is the chance of survival without disability for a baby born at 24 wks.
107.what is the most abnormal karyo to a/w truncus arteriosus? [Options: trisomy 13 /21/
deletion of q22, others ].
108.Mother with type 1 DM came with PPROM. What is the more common cause of death
for this baby

[ prematurity or lung hypoplasia?].


109.in what country is obstetric cholestasis most common?

[?Cheli]
110. A woman with PH of recurrent DVT, now on thromboprophylaxis with Rivaroxaban -
when to stop pre op? 24 hrs , 5 days , 7 days , 4 weeks .

After we did some search, it appeared to be an oral heparin, given daily. Needs to be
stopped 24 hrs prior to surgery. This was a web search. Please check again
111. What is the length of the presenting diameter in Face presentation [9.5 cm]
112. A midwife ot sure about presentation called you. You palpate malar eminence + ?
alveolar margin. [?Face]
st
113. A pregnant women with breast lump. 1 invest will be: (options were u/s-
mammogram- chest x ray).
114. A woman found to have asymptomatic GBS bacteruria. Management plan?

[?treat now and IPAP in labour].


115. What is the rate of spont. Reversion to breech after ECV in PG

288 | P a g e
[? 5%]
116. A woman & her partner both are CF carriers. What is the chance of an affected
baby?

[1:4]
117. From the following, what is NOT a risk factor for twins?

Options: advanced age, PH of monozygotic twins, IVF, others.


118. What is the chance of success of Mc robert with suprapubic pressure to effect
delivery?

[? 90%]
119. From the following, what is a major risk factor for SGA? [can’t remember options]
120. What is the best measure of GA at 84 mm diameter ?GS. [ options: CRL, HC, BPD].
• EMQ: HSV: can’t remember options
121. A woman came in labour, tells you that her partner was recently treated for HSV.
122. A woman came in labour. Was treated 2 wks ago for recurrent HSV.
123. A woman came in labour, was treated ? 2 wks ago for 1ry HSV. After counseling;
she’s willing for VD.
• EMQ : Management

Options: Augment with oxytocin, IPAP, Erythromycin, I/M steroids, Expectant Mng,....
124. GA 22wks, Known GBS carrier, now PPROM ,? Features of ch.amnitis
125. GA 28 wks. Threatened PTL. Speculum cx 3 cm with intact membranes. Already
given full dose I/M steroids 2 wks ago.
126. What of the following conditions has more association with a male fetus

Options included AFLP, obstetric cholestasis,.......


• EMQ : The causative organism. (GAS, GBS, HSV, CT, Staph,... )
127. A neonate developed eye infection(stick eyes) after 1 wk from delivery.

289 | P a g e
[CT]
128. A mother who had a vulval soreness, gave birth to a neonate who latter developed
signs of sepsis & he had vesicles on his back.

[?HSV]
129. What is the anti-D level above which there should be referral for fetal medicine unit?
[was it “above which” or “at which”?]
130. Blood level of which antiepileptic drug may particularly decrease in pregnancy ( plus
phenytoin may need monitoring)
Carbamazepine
Lamotrigine
Valproate sodium
131. Patient on carbamazepine comes for emergency contraception
Levonorgestrel 1.5 mg
Levonorgestrel 3 mg
132. 32 years presents with flat reddish brown lesion on the vulva with satellite
lesions
Melanoma
VIN
Candidiasis
Lichen sclerosis

133. 15 years old girl accompanied with her 15 years old boy friend 17 weeks
pregnant asking for termination
Encourage her to tell her parents
Take written consent
Discuss with child protection lead
134. At 24 weeks delivery what is the percentage of babies that survive:
20%, 30% , 40%, 60%

290 | P a g e
135. Cancer endometrium with parametruim involved in surgery 5 year survival
rate is
70%
60%
50%
136. Both partners carrier of cystic fibrosis offer:
PGD
Amniocentesis
Advice against pregnancy
137. Obstetric cholestasis which vitamin to be given
Vitamin k
Vitamin c
Vitamin d
Folic acid
138. P2 DVT in previous pregnancy start LMWH
ASAP
At 12 weeks
At 24 weeks
139. 33 years previous difficult forceps delivery, now fully dilated OP position:
Cs
Trial of forceps in the theatre
140. Suture material for 3rd and 4th perineal tear
Polydioxanone PDS 2 zero
Polydioxanone PDS 3 zero

141. Suture material for b-Lynch

142. MCDA twin from when to do us

291 | P a g e
16 ,18,24 weeks

143. MCDA twin DVP one twin < 1.5, other > 10 cm
TTT
TRAP

144. PMS first line ttt


New OCP
LNG IUS
Progesterone
145. Patient with cystic fibrosis died at 36 days postpartum with
respiratory infection
Direct maternal death
Indirect maternal death
146. Woman refusing cs her husband insists to save the baby
Do cs
Follow the woman’s wish
147. Malig Growth involving hood of the clitoris
Local excision
Wide local excision + ipsilateral groin LN dissection
Wide local excision + bilateral groin LN dissection
148. Waht drug you choose for an asthamtic lady with post partum HTN?

i. [? Nifidipine. There was an ACEi in the options but not enalapril/captopril]


149. What is the definition of perinatal mortality rate?

i. [? After 24wks GA, up to 7days/1000 or 10.000?]


150. There was an EMQ with 3 scenarios of APH.

292 | P a g e
151. There was a question about the mode of delivery in HIV mother on HAART whose
last VL was 150

i. [?? The 2011 NICE c/s guideline says don’t offer c/s if VL <400. 2014 BHIVA
said “consider” c/s].
.

Rivaroxaban: Drug information


Mechanism of Action

Inhibits platelet activation and fibrin clot formation via direct, selective and reversible inhibition of
factor Xa (FXa) in both the intrinsic and extrinsic coagulation pathways. FXa, as part of the
prothrombinase complex consisting also of factor Va, calcium ions, factor II and phospholipid,
catalyzes the conversion of prothrombin to thrombin. Thrombin both activates platelets and
catalyzes the conversion of fibrinogen to fibrin.

Pregnancy Risk Factor

C (show table)

Pregnancy Implications

Adverse events were observed in animal reproduction studies. Data are insufficient to evaluate the
safety of oral factor Xa inhibitors during pregnancy; use during pregnancy should be avoided
(Guyatt, 2012). Use may increase the risk of pregnancy related hemorrhage. Clinicians should note
that the anticoagulant effect cannot be easily monitored or readily reversed. Prompt clinical
evaluation is warranted with any unexplained decrease in hemoglobin, hematocrit or blood
pressure, or fetal distress. Pregnancy planning should be discussed if use is needed in women of
reproductive potential. Use during pregnancy is contraindicated in the Canadian labeling.

Lactation

Excretion in breast milk unknown/not recommended

Breast-Feeding Considerations

It is not known if rivaroxaban is excreted into breast milk. Due to the potential for serious adverse
reactions in the nursing infant, the decision to discontinue rivaroxaban or to discontinue breast-
feeding during therapy should take into account the benefits of treatment to the mother; use of

293 | P a g e
alternative anticoagulants is preferred (Guyatt, 2012). Use in breast-feeding mothers is
contraindicated in the Canadian labeling.

European guidelines recommend waiting at least 22-26 hours following the last
rivaroxaban dose when using prophylactic dosing (eg, 10 mg once daily) before
catheter placement or lumbar puncture (Gogarten, 2010). When higher doses are
used (eg, 20 mg once daily), some suggest avoidance of neuraxial procedures for at
least 48 hours (Rosencher, 2013).
Surgery and invasive procedures: Discontinue rivaroxaban at least 24 hours prior to
procedure. Some have recommended to discontinue rivaroxaban 3 days prior to a
procedure in patients with a CrCl ≥50 mL/minute or 5 days prior in patients with a
CrCl <50 mL/minute (Wysokinski, 2012). The risk of bleeding should be weighed
against the urgency of the procedure; reinitiate rivaroxaban when adequate
hemostasis has been achieved unless oral therapy cannot be administered, then
consider administration of a parenteral anticoagulant.
Question: 11
A 27 year old woman presents with painless vaginal bleeding six weeks after her last period. Her
urine pregnancy test was positive 10 days ago. A transvaginal ultrasound scan shows a gestation
sac containing a yolk sac and a fetal pole of 6.3mm. Fetal cardiac activity is absent. Her serum
BHCG level is 1500 iu/l.
Lead – in Select the correct description of the scan findings from the list below
ENHANCED REVISION PROGRAMME www.rcog.org.uk/erp
Single Best Answer
Options: a) Missed miscarriage b) Pregnancy of unknown location c) Pregnancy of uncertain
viability d) Partial mole e) Complete miscarriage
ENHANCED REVISION PROGRAMME www.rcog.org.uk/erp
Single Best Answer
Options: a) Missed miscarriage b) Pregnancy of unknown location c) Pregnancy of uncertain
viability d) Partial mole e) Complete miscarriage
ENHANCED REVISION PROGRAMME www.rcog.org.uk/erp
Single Best Answer
Question: 12 A 27 year old woman presents with painless vaginal bleeding six weeks after her last
period. Her urine pregnancy test was positive 10 days ago. A transvaginal ultrasound scan shows a
gestation sac containing a yolk sac and a fetal pole of 6.3mm. Fetal cardiac activity is absent. Her
serum BhCG level is 1500 iu/l.

294 | P a g e
Lead – in Select the most appropriate management plan from the list below.
ENHANCED REVISION PROGRAMME www.rcog.org.uk/erp
Single Best Answer
Options: a) Advise expectant management b) Offer surgical evacuation of products of conception c)
Offer medical management of miscarriage d) Re scan in seven days e) Re scan by a more
experienced sonographer
ENHANCED REVISION PROGRAMME www.rcog.org.uk/erp
Single Best Answer
Options: a) Advise expectant management b) Offer surgical evacuation of products of conception c)
Offer medical management of miscarriage d) Re scan in seven days e) Re scan by a more
experienced sonographer
ENHANCED REVISION PROGRAMME www.rcog.org.uk/erp
Single Best Answer
Question: 13 A 23 year old woman attends the early pregnancy unit at 7 weeks gestation with
vaginal spotting, left iliac fossa pain and one episode of diarrhoea. A transvaginal scan has shown
a 2x4mm echolucent area in the uterine cavity with no yolk sac or fetal pole. There is a moderate
amount of free fluid in the pelvis. The serum BhCG level is 2300 iu/l. Her observations are:
temperature 36.5oC, blood pressure 100/65mmHg, pulse 120 bpm, oxygen saturation 98%,
respiratory rate 24 per minute, pain score moderate.
Lead – in Select the most appropriate initial management from the options below.
ENHANCED REVISION PROGRAMME www.rcog.org.uk/erp
Single Best Answer
Options: a) Repeat quantitative BhCG in 48 hours b) Rescan in seven days c) Intramuscular
methotrexate d) Laparoscopy e) Laparotomy
ENHANCED REVISION PROGRAMME www.rcog.org.uk/erp
Single Best Answer
Options: a) Repeat quantitative BhCG in 48 hours b) Rescan in seven days c) Ontramuscular
methotrexate d) Laparoscopy e) Laparotomy
ENHANCED REVISION PROGRAMME www.rcog.org.uk/erp
Single Best Answer
Question: 14 A 34 year old woman presents with painless vaginal bleeding at 8 weeks gestation in
her third pregnancy. She has a three year old child and her last pregnancy resulted in a
spontaneous miscarriage at 7 weeks gestation six months ago. She is known to be rhesus negative.

295 | P a g e
Lead – in Which of the following statements is correct?
ENHANCED REVISION PROGRAMME www.rcog.org.uk/erp
Single Best Answer
Options: a) Complete bed rest for two weeks is recommended. b) Anti D isn’t required c) Low
molecular weight heparin should be commenced and continued till 12 weeks gestation d) There is
an increased risk of congenital abnormalities if the pregnancy continues e) She should commence
75mg aspirin daily until 12 weeks gestation
ENHANCED REVISION PROGRAMME www.rcog.org.uk/erp
Single Best Answer
Options: a) Complete bed rest for two weeks is recommended b) Anti D isn’t required c) Low
molecular weight heparin should be commenced and continued till 12 weeks gestation d) There is
an increased risk of congenital abnormalities if the pregnancy continues e) She should commence
75mg aspirin daily until 12 weeks gestation
ENHANCED REVISION PROGRAMME www.rcog.org.uk/erp
Single Best Answer
Question: 15 A 35 year old woman has been fully investigated after her third first trimester
miscarriage and all of the results are normal.
Lead – in
Which of the following interventions have been shown to be effective in reducing the risk of further
miscarriage in unexplained recurrent miscarriage:
ENHANCED REVISION PROGRAMME www.rcog.org.uk/erp
Single Best Answer
Options: a) Laparoscopic ovarian drilling b) Weekly BhCG injections c) Support and counselling d)
Cervical cerclage e) Progesterone supplements
ENHANCED REVISION PROGRAMME www.rcog.org.uk/erp
Single Best Answer
Options: a) Laparoscopic ovarian drilling b) Weekly BhCG injections c) Support and counselling d)
Cervical cerclage e) Progesterone supplements
ENHANCED REVISION PROGRAMME www.rcog.org.uk/erp
Single Best Answer

296 | P a g e
Question: 16 A 59 year old woman presents with prolapse. On straining the cervix descends
outside the vaginal opening. The patient is companioning of frequency and urgency of micturition.
The prolapse is also causing obstruction to intercourse.
Lead – in You should recommend:
ENHANCED REVISION PROGRAMME www.rcog.org.uk/erp
Single Best Answer
Options: a) Pelvic floor physiotherapy b) Pelvic floor repair c) Shelf pessary d) Urodynamic studies
e) Vaginal hysterectomy and pelvic floor repair
ENHANCED REVISION PROGRAMME www.rcog.org.uk/erp
Single Best Answer
Options: a) Pelvic floor physiotherapy b) Pelvic floor repair c) Shelf pessary d) Urodynamic studies
e) Vaginal hysterectomy and pelvic floor repair
ENHANCED REVISION PROGRAMME www.rcog.org.uk/erp
Single Best Answer
Question: 17 A 52 year old woman with advanced vault prolapse would like to consider different
surgical options.
Lead – in Your counselling should include the following:
ENHANCED REVISION PROGRAMME www.rcog.org.uk/erp
Single Best Answer
Options: a) Colpocleisis is a suitable option b) Erosion is a rare complications if mesh is used c)
Iliococcygeal fixation is recommended by NICE d) Objective and subjective outcomes for
sacrospinous fixation are poor e) Sacral colpopexy has a superior outcome to vaginal
sacrospinous fixation
ENHANCED REVISION PROGRAMME www.rcog.org.uk/erp
Single Best Answer
Options: a) Colpocleisis is a suitable option b) Erosion is a rare complications if mesh is used c)
Iliococcygeal fixation is recommended by NICE d) Objective and subjective outcomes for
sacrospinous fixation are poor e) Sacral colpopexy has a superior outcome to vaginal
sacrospinous fixation
ENHANCED REVISION PROGRAMME www.rcog.org.uk/erp
Single Best Answer

297 | P a g e
Question: 18 A 49 year old woman presents complaining of urgency, frequency of micturition and
stress incontinence. Her symptoms did not improve with pelvic floor physiotherapy. Uro-dynamics
studies results: • Maximum bladder capacity – 300mls • Detrusor pressure reached 20cm /H20
during filling • Stress leak was reproduced at detrusor pressure of 27cm/H20 Normal voiding
cystometry
Lead – in Your counselling should include the following:
ENHANCED REVISION PROGRAMME www.rcog.org.uk/erp
Single Best Answer
Options: a) Ambulatory uro-dynamics b) Anticholinergics c) Colposuspension d) Conservative
management e) Mid-urethral tape (TVT)
ENHANCED REVISION PROGRAMME www.rcog.org.uk/erp
Single Best Answer
Options: a) Ambulatory uro-dynamics b) Anticholinergics c) Colposuspension d) Conservative
management e) Mid-urethral tape (TVT)
ENHANCED REVISION PROGRAMME www.rcog.org.uk/erp
Single Best Answer
Question: 19 A 55 year old woman presents to the clinic complaining of urgency, frequency,
nocturia and stress incontinence. Her BMI is 28 and she smokes 20 cigarettes a day. The patient
has kept bladder diary which shows diurnal frequency 10 times, nocturia 3 times, voided volumes
50-200mls and many leaking episodes.
Lead – in The most likely diagnosis is:
ENHANCED REVISION PROGRAMME www.rcog.org.uk/erp
Single Best Answer
Options: a) Mixed urinary incontinence b) Over active bladder c) Over active detrusor d) Neurogenic
bladder e) Stress incontinence
ENHANCED REVISION PROGRAMME www.rcog.org.uk/erp
Single Best Answer
Options: a) Mixed urinary incontinence b) Over active bladder c) Over active detrusor d) Neurogenic
bladder e) Stress incontinence
ENHANCED REVISION PROGRAMME www.rcog.org.uk/erp
Single Best Answer

298 | P a g e
Question: 20 A 45 year old woman presents complaining of urgency, frequency of micturition and
stress incontinence. She has to wear pads during the day. Her symptoms failed to respond to pelvic
floor physiotherapy. Uro-dynamics showed: • Bladder capacity – 400mls • Detrusor pressure
reached 3cm/H20 during filling • Stress incontinence was positive at detrusor pressure of cm/H20 •
Completely voided with average flow rate 23mls/s
Lead - in Based on the urodynamic results, you should advise:
ENHANCED REVISION PROGRAMME www.rcog.org.uk/erp
Single Best Answer
Options: a) Anticholinergics b) Autologous rectus fascial sling c) Continue conservative
management d) Exclude vesico-vaginal fistula e) Mid urethral tape (TVT)
ENHANCED REVISION PROGRAMME www.rcog.org.uk/erp
Single Best Answer
Options: a) Anticholinergics b) Autologous rectus fascial sling c) Continue conservative
management d) Exclude vesico-vaginal fistula e) Mid urethral tape (TVT)
ENHANCED REVISION PROGRAMME www.rcog.org.uk/erp
Single Best Answer
Question: 21
Interpret the above intrapartum electronic fetal heart rate tracing using the definitions and
classifications recommended in the Intrapartum Care document produced by NICE 2007. The
pattern did not change during 120 minutes of observation.
Lead – in Which is single most accurate description of the tracing?
ENHANCED REVISION PROGRAMME www.rcog.org.uk/erp
Single Best Answer
Options: a) Abnormal b) Suspicious c) Non-reassuring d) Sleep pattern e) Normal variant
ENHANCED REVISION PROGRAMME www.rcog.org.uk/erp
Single Best Answer
Options: a) Abnormal b) Suspicious c) Non-reassuring d) Sleep pattern e) Normal variant
ENHANCED REVISION PROGRAMME www.rcog.org.uk/erp
Single Best Answer
Question: 22

299 | P a g e
Interpret the above intra-partum electronic fetal heart rate tracing using the definitions and
classifications recommended in the Intra-partum Care document produced by NICE 2014. Lead – in
Which is single most accurate description of the tracing?
ENHANCED REVISION PROGRAMME www.rcog.org.uk/erp
Single Best Answer
Options: a) Non-reassuring b) Four features are reassuring c) Abnormal d) Recurrent late
decelerations e) Fetal anaemia
ENHANCED REVISION PROGRAMME www.rcog.org.uk/erp
Single Best Answer
Options: a) Non-reassuring b) Four features are reassuring c) Abnormal d) Recurrent late
decelerations e) Fetal anaemia
ENHANCED REVISION PROGRAMME www.rcog.org.uk/erp
Single Best Answer
Question: 23
Interpret the above intra-partum electronic fetal heart rate tracing using the definitions and
classifications recommended in the Intra-partum Care document produced by NICE 2014. Lead – in
Which is single most accurate description of the tracing?
ENHANCED REVISION PROGRAMME www.rcog.org.uk/erp
Single Best Answer
Options: a) Normal baseline rate b) Reduced variability c) More than two non-reassuring features d)
Hypoxic trace e) Abnormal
ENHANCED REVISION PROGRAMME www.rcog.org.uk/erp
Single Best Answer
Options: a) Normal baseline rate b) Reduced variability c) More than two non-reassuring features d)
Hypoxic trace e) Abnormal
ENHANCED REVISION PROGRAMME www.rcog.org.uk/erp
Single Best Answer
Question: 24
Interpret the above intra-partum electronic fetal heart rate tracing using the definitions and
classifications recommended in the Intra-partum Care document produced by NICE 2014.
Lead – in Which is single most accurate description of the tracing?

300 | P a g e
ENHANCED REVISION PROGRAMME www.rcog.org.uk/erp
Single Best Answer
Options: a) Fetal tachycardia b) Two non-reassuring features c) Recurrent late decelerations d)
More than one abnormal feature e) Fetal hypoxia
ENHANCED REVISION PROGRAMME www.rcog.org.uk/erp
Single Best Answer
Options: a) Fetal tachycardia b) Two non-reassuring features c) Recurrent late decelerations d)
More than one abnormal feature e) Fetal hypoxia

301 | P a g e

You might also like